summaryrefslogtreecommitdiffstats
diff options
context:
space:
mode:
authorUlli Kehrle <ulli.kehrle@rwth-aachen.de>2017-12-17 20:14:11 +0100
committerUlli Kehrle <ulli.kehrle@rwth-aachen.de>2017-12-17 20:14:11 +0100
commit5bae51d51824c2d960bc15d332ec69bd5df515bf (patch)
tree99da1ebb3bdab7b5be337e9a3556bd0680705958
parent07a2a40a36c3d114950cd54ae293b9b11352f9bb (diff)
downloadfunkana-5bae51d51824c2d960bc15d332ec69bd5df515bf.tar.gz
funkana-5bae51d51824c2d960bc15d332ec69bd5df515bf.tar.xz
Quellcode neu strukturiert, Zusätzliche Version für kleine Bildschirme hinzugefügt
-rw-r--r--.gitignore3
-rw-r--r--Makefile11
-rw-r--r--ch01-lineare-struktur.tex296
-rw-r--r--ch02-topologie.tex566
-rw-r--r--ch03-topologisch-lineare-raeume.tex1480
-rw-r--r--ch04-unitaere-raeume.tex447
-rw-r--r--ch05-hahn-banach.tex370
-rw-r--r--funkana-ebook.tex68
-rw-r--r--funkana.tex35
-rw-r--r--inhalt.tex972
-rw-r--r--latexmkrc4
-rw-r--r--motivation.tex68
-rw-r--r--pdf/funkana.pdfbin654737 -> 0 bytes
-rw-r--r--ref.bib153
-rw-r--r--skript.cls67
15 files changed, 4029 insertions, 511 deletions
diff --git a/.gitignore b/.gitignore
index 0583d8e..83c0976 100644
--- a/.gitignore
+++ b/.gitignore
@@ -1,3 +1,4 @@
auto/*
build/*
-*.pdf \ No newline at end of file
+*.pdf
+gitHeadInfo.gin \ No newline at end of file
diff --git a/Makefile b/Makefile
index a2933d0..f653328 100644
--- a/Makefile
+++ b/Makefile
@@ -1,4 +1,13 @@
-all: pdf/funkana.pdf
+n: all publish
+
+all: pdf/funkana.pdf \
+ pdf/funkana-ebook.pdf
pdf/funkana.pdf: funkana.tex inhalt.tex skript.cls
latexmk funkana.tex
+
+pdf/funkana-ebook.pdf: funkana-ebook.tex inhalt.tex skript.cls
+ latexmk funkana-ebook.tex
+
+publish: all
+ scp -r pdf/* hrnz.li:public_html/files/funkana/
diff --git a/ch01-lineare-struktur.tex b/ch01-lineare-struktur.tex
new file mode 100644
index 0000000..9f151c6
--- /dev/null
+++ b/ch01-lineare-struktur.tex
@@ -0,0 +1,296 @@
+\chapter{Die lineare Struktur}
+\section{Der lineare Raum}
+Sei im Folgenden stets $\K = ℝ$ oder $\K = ℂ$. Zunächst die
+\begin{definition}[Vektorraum]
+ Sei $\K$ ein Körper. Eine Abelsche Gruppe $(X,+)$ zusammen mit einer Abbildung
+ \[
+ \cdot : \K × X → X
+ \]
+ heißt $\K$-Vektorraum, falls für alle $\alpha , β ∈ \K$ und $x, y ∈ X$ gilt:
+ \begin{enumerate}[label=(V\arabic*)]
+ \item $\alpha x+y) = \alpha x + βy$
+ \item $(\alpha +β)x = \alpha x + βx$
+ \item $(\alpha β)x = \alpha (βx)$
+ \item $1 \cdot x = x$
+ \end{enumerate}
+\end{definition}
+
+\begin{bemerkung-nn}
+ Je nachdem, ob $\K = ℂ$ oder $\K = ℝ$ gilt, heißt $X$ ein \emph{komplexer} oder ein \emph{reeller} Vektorraum.
+\end{bemerkung-nn}
+
+\begin{bemerkung-nn}
+ Eine nichtleere Teilmenge $Y ⊂ X$ ist bereits dann ein linearer Raum, falls aus $\alpha , β ∈ \K$, $x, y ∈ Y$ bereits $\alpha x + βy ∈ Y$ folgt, also $Y$ abgeschlossen unter den Vektorraumoperationen ist.
+ $Y$ heißt dann \emph{linearer Teilraum} oder auch \emph{linearer Unterraum}.
+\end{bemerkung-nn}
+
+\begin{bemerkung-nn}
+ Zu jeder Teilmenge $M ⊂ X$ bildet die Menge aller Linearkombinationen von je endlich vieler Elemente einen linearen Teilraum von $X$.
+ Dieser heißt die \emph{lineare Hülle} von $M$ oder der \emph{Aufspann} von $M$
+ \[
+ \lspan M = \left\{ x ∈ X: ∃ l ∈ ℕ, \alpha _1,…,\alpha _l ∈ \K, m_1,…,m_l ∈ M \text{ mit } \sum_{i=1}^l \alpha _i m_i = x \right\}.
+ \]
+\end{bemerkung-nn}
+
+\begin{bemerkung-nn}
+ $M = \{x_\lambda \}_{\lambda ∈ \Lambda } ⊂ X$ heißt \emph{Basis} oder \emph{Hamel-Basis} von $X$, falls $M$ \emph{linear unabhängig}, das heißt,
+ $0 ∈ X$ lässt sich nur auf triviale Art und Weise als Linearkombination endlich vieler der $x_\lambda $ schreiben, und $\lspan M = X$ ist.
+\end{bemerkung-nn}
+
+\begin{bemerkung-nn}
+ Besitzt $X$ eine Basis von $n < \infty $ Elementen, dann heißt $n$ die \emph{Dimension} von $X$ und wir schreiben $\dim X = n$.
+ Andernfalls heißt $X$ \emph{unendlich-dimensional} ($\dim X = \infty $).
+\end{bemerkung-nn}
+
+\begin{bemerkung-nn}
+ Seien $X_1, X_2 ⊂ X$ lineare Teilräume. Dann ist
+ \[
+ X_1 + X_2 \coloneq \left\{ \alpha x_1 + βx_2: \alpha , β ∈ \K, x_1 ∈ X_1, x_2 ∈ X_2 \right\}
+ \]
+ ebenfalls ein linearer Teilraum.
+ Falls $X_1 ∩ X_2 = \{ 0\}$, schreiben wir $X_1 \oplus X_2$ und nennen die Summe \emph{direkt}.
+\end{bemerkung-nn}
+
+\begin{bemerkung-nn}
+ Sei $Y$ ein linearer Teilraum von $X$. Definiere die Äquivalenzrelation $\sim$ auf $X$ durch
+ $x \sim y \Leftrightarrow x - y ∈ Y$.
+ Dann wird die Menge der Äquivalenzklassen mit vertreterweiser Addition und Multiplikation auch ein $\K$-Vektorraum.
+ Wir schreiben für diesen Vektorraum $X/Y$.
+\end{bemerkung-nn}
+
+\begin{satz}\label{01-vr-besitzt-basis}
+ Jeder lineare Raum besitzt eine (Hamel-)Basis.
+\end{satz}
+\begin{proof}
+ Folgt unmittelbar aus \cref{01-basisergaenzungssatz}.
+\end{proof}
+
+\begin{satz}[Basisergänzungssatz]\label{01-basisergaenzungssatz}
+ Sei $M ⊂ X$ eine linear unabhängige Teilmenge.
+ Dann gibt es eine Basis $B$ von $X$ mit $M ⊂ B$.
+\end{satz}
+\begin{proof}
+ Sei $P$ die durch Inklusion geordnete Menge aller linear unabhängigen Teilmengen von $X$, die $M$ umfassen.
+ Wegen $M ∈ P$ ist $P$ nichtleer.
+ Für jede totalgeordnete Teilmenge $T ⊂ P$ ist $\bigcup T ∈ P$, also $T$ durch $\bigcup T$ beschränkt.
+ Nach Zorn's Lemma besitzt $P$ ein maximales Element $B$.
+ Wäre $B$ keine Basis, gäbe es ein $x ∈ X \setminus \lspan B$.
+ Aber dann wäre $B ∪ \{x\}$ ebenfalls linear unabhängig im Widerspruch zur Maximalität von $B$.
+\end{proof}
+
+\section{Beispiele}
+\begin{beispiel}
+ Der $ℝ^n$ ist ein linearer Raum über dem Körper $ℝ$. Der $ℂ^n$ ist sowohl ein $ℂ$- als auch ein $ℝ$-Vektorraum.
+\end{beispiel}
+
+\begin{beispiel}
+ Sei $[a,b] ⊂ ℝ$, $a < b$. Dann ist
+ \[
+ C[a,b] = \{x: [a,b] → \K, x \text { ist stetig}\}
+ \]
+ ein $\K$-Vektorraum mit $\dim C[a,b] = \infty $.
+ Zum Beispiel sind die Monome $(t^k)_{k ∈ ℕ}$ ein unendliches linear unabhängiges System, jedoch keine Basis.
+ Tatsächlich ist jede Basis dieses Raumes überabzählbar.
+\end{beispiel}
+
+\begin{beispiel}[Folgenräume]
+ Es ist
+ \[
+ \ell^p = \{ x = (ξ_n)_{n ∈ ℕ} , ξ_n ∈ \K, \sum_{n=1}^∞ |ξ_n|^p < ∞ \}
+ \]
+ für $0 < p < ∞$ ein linearer Raum.
+ Die Menge der Einheitsvektoren $\{e_i\}_{i ∈ ℕ}$ mit $e_i = (0,…,0,1,0,…)$ ist eine unendliche linear unabhängige Teilmenge, aber ebenfalls keine Basis.
+
+ Genauso ist
+ \[
+ \ell^∞ = \left\{ x = (ξ_n)_{n ∈ ℕ} : ξ_n ∈ \K, \sup_{n=1} |ξ_n| < ∞ \right\}
+ \]
+ ein überabzählbar"=dimensionaler linearer Raum mit den Unterräumen
+ \[
+ c = \left\{ (ξ_n)_{n ∈ ℕ} ∈ \ell^∞: \lim_{n → ∞} ξ_n \text{ existiert}\right\}
+ \]
+ und
+ \[
+ c_0 = \left\{ (ξ_n)_{n ∈ ℕ} ∈ \ell^∞: \lim_{n → ∞} ξ_n = 0 \right\}.
+ \]
+\end{beispiel}
+
+\begin{beispiel}[Lebesgue-integrierbare Funktionen].
+ Sei $M ⊂ ℝ$ messbar und $0 < p < ∞$.
+ Dann ist
+ \[
+ \L^p(M) = \left\{f : M → ℝ, f \text { messbar}, ∫_M |f|^p \dd μ < ∞ \right\}
+ \]
+ ein unendlich"=dimensionaler linearer Raum.
+ Offenkundig ist $\mathcal N \coloneq \{ f: M → ℝ,\; f = 0$ fast überall $\}$ ein Unterraum von $\L^p(M)$, also auch
+ \[
+ L^p(M) = \L^p(M)/\mathcal N
+ \]
+ ein linearer Raum.
+\end{beispiel}
+
+\section{Lineare Abbildungen}
+\begin{definition}
+ Seien $X, Y$ lineare Räume über $\K$. $A: X → Y$ heißt \emph{linear}, falls für alle $x_1, x_2 ∈ X$ und $\alpha , β ∈ \K$ gilt:
+ \[
+ A(\alpha x_1 + βx_2) = \alpha A(x_1) + βA(x_2).
+ \]
+ $A: X → \K$ heißt \emph{lineares Funktional}.
+ Für $A$ linear heißt $R(A) = \im A = \{A(x): x ∈ X\}$ der \emph{Bildraum} von $A$ und $N(A) = \ker A = \{ x ∈ X: A(x) = 0\}$ der \emph{Kern} von $A$.
+\end{definition}
+
+\begin{bemerkung}
+ Sei $A: X → Y$ linear.
+ \begin{enumerate}
+ \item Sei $M ⊂ X $ ein linearer Unterraum. Dann ist $A(M) ⊂ Y$ wieder ein linearer Unterraum und es gilt $\dim A(M) \le \dim M$ mit Gleichheit bei Injektivität.
+ \item Es gilt
+ \[
+ A \text{ injektiv} \Longleftrightarrow N(A) = \{ 0\}.
+ \]
+ Allgemeiner ist
+ \[
+ X/(N(A)) \cong \im A.
+ \]
+ \item
+ Falls $\dim X = \dim Y = n < \infty $, dann ist $A$ genau dann injektiv, wenn $A$ surjektiv ist.
+ \item
+ $A: X → Y$ ist linear und bijektiv genau dann, wenn es eine lineare Umkehrabbildung $A^{-1}: Y → X$ gibt.
+ \item
+ Falls so ein $A: X → Y$ linear und bijektiv existiert, nennen wir $X$ und $Y$ \emph{linear isomorph.}
+ $A$ heißt dann ein \emph{linearer Isomorphismus}.
+
+ Nur falls $\dim X = \dim Y < \infty $ sind $X$ und $Y$ auch „topologisch“ isomorph.
+ In diesem Fall erhält man die Prototypen $ℝ^n$ und $ℂ^n$ für endlich-dimensionale Vektorräume und andere gibt es nicht (die sie auch als Topologische Räume isomorph sind).
+ \end{enumerate}
+\end{bemerkung}
+
+\begin{beispiel-nn}
+ $X = \{ x: [a,b] → ℝ, x, \dot x, \ddot x \text{ stetig},\; x(a) = \dot x(a) = 0\}$ ist ein linearer Raum.
+ Sei $Y = C[a,b]$ und $A: X → Y$ gegeben durch
+ \[
+ (Ax)(t) \coloneq \ddot x(t) + c_1 (t) \dot x (t) + c_2 (t) x(t), \quad t ∈ [a,b], c_1,c_2 ∈ C[a,b].
+ \]
+ Dann ist $A$ linear, weil differenzieren linear ist und $A$ ist injektiv:
+ Zunächst ist $x = 0$ eine Lösung der linearen Differentialgleichung zweiter Ordnung $Ax = 0$.
+ Die Theorie der Differentialgleichungen sagt uns, dass diese Differentialgleichung eine eindeutige Lösung des Anfangswertsproblems ist.
+
+ $A$ ist aber auch surjektiv: Sei $y ∈ Y$ gegeben, dann suchen wir $x ∈ X$ mit $Ax = y$.
+ Also wollen wir eine inhomogene Differentialgleichung zweiter Ordnung lösen.
+ Auch diese ist nach der Theorie von gewöhnlichen Differentialgleichungen eindeutig lösbar.
+
+ Also ist $A$ bijektiv, das heißt, es gibt eine lineare Abbildung $A^{-1}: Y → X$.
+ Diese Inverse ist in der Regel schlecht anzugeben.
+ Einen einfacheren Spezialfall dazu wird in der Übung behandelt.
+\end{beispiel-nn}
+
+\begin{beispiel-nn}
+ Sei $X = Y = C[a,b]$, $A: X → X$ gegeben durch
+ \[
+ (Ax)(t) \coloneq ∫_a^b k(s,t) x(s) ds, \quad t ∈ [a,b],
+ \]
+ wobei $k : [a,b] × [a,b] → ℝ$ stetig und gegeben ist.
+ Dann ist $A$ linear, da das Integral linear ist.
+ Auch ist, wenn $\lambda ∈ ℝ$ ein Parameter ist, die Abbildung
+ \[
+ (A_\lambda x)(t) \coloneq \lambda x(t) - (Ax)t), \quad t ∈ [a,b]
+ \]
+ linear.
+ Die Probleme $Ax = y$ (bei gegebenem $y ∈ Y$ und gesuchtem $x ∈ X$) oder $A_\lambda x = 0$ (gesucht ist $\lambda ∈ ℝ$ und eine nichttriviale Lösung $x ∈ X \setminus \{ 0\}$)
+ heißen Integralgleichungen erster und zweiter Ordnung.
+\end{beispiel-nn}
+
+\begin{beispiel-nn}
+ Sei $X = C[a,b]$, $A : X → ℝ$ mit
+ \[
+ Ax = x(t_0),
+ \]
+ wobei $t_0 ∈ [a,b]$ fest gewählt sei.
+ Eine andere lineare Abbildung $A: X → ℝ$ ist gegeben durch
+ \[
+ Ax = ∫_a^b x(t) dt
+ \]
+ Dann sind beide Abbildungen $A$ linear und nicht injektiv, aber surjektiv.
+\end{beispiel-nn}
+
+\begin{beispiel-nn}
+ Sei $X = \ell^2$, $A: X → X$. Für $x = (ξ_n)_{n ∈ ℕ}$ sei
+ \[
+ Ax = (0,ξ_1, ξ_2, \dots) ∈ \ell^2.
+ \]
+ $A$ heißt (Rechts-)Shiftoperator und ist linear und injektiv, jedoch nicht surjektiv.
+ Solche Abbildungen gibt es für $\dim X = \dim Y < \infty $ nicht.
+\end{beispiel-nn}
+
+\section{Duale Räume}
+$A: X → \K$ sei ein lineares Funktional, $X$ ein linearer Raum. Wir verwenden ein neues Symbol (statt $A$)
+\[
+ x': X → \K = \begin{cases} ℝ \\ ℂ \end{cases} \text{ linear}.
+\]
+Wir schreiben nun
+\[
+ x'(x) =: \langle x, x' \rangle = \langle x, x' \rangle_{X × X^f} ∈ \K.
+\]
+Wir setzen
+\[
+ X^f \coloneq \left\{ x': x' \text{ ist lineares Funktional auf } X \right\}.
+\]
+Hierbei sollte man nicht $x'$ nicht mit der Ableitung von $x$ verwechseln.
+Auch ist $\langle -, - \rangle_{X × X^f}$ kein Skalarprodukt.
+
+Der Raum $X^f$ wird auf natürlicher Weise zum linearen Raum mit
+\[
+ (\alpha x_1' + βx_2')(x) \coloneq \alpha x_1'(x) + βx_2'(x), \quad x ∈ X, x_1', x_2' ∈ X^f, \alpha , β ∈ \K.
+\]
+So ist
+\[
+ \langle -,- \rangle_{X×X^f}: X × X^f → \K
+\]
+bilinear.
+\begin{definition}
+ $X^f$ heißt der \emph{algebraische Dualraum} zu $X$.
+ $X^{ff} \coloneq (X^f)^f$ heißt der \emph{biduale Raum} zu $X$.
+\end{definition}
+
+\begin{beispiel-nn}
+ $X^{ff}$ liefert die kanonische Abbildung
+ \[
+ J: X → X^{ff}, \; x ↦ J(x) = x''
+ \]
+ mit
+ \[
+ \langle x', x'' \rangle \coloneq \langle x, x' \rangle \quad ∀x' ∈ X^f.
+ \]
+ Damit ist $x'': X^f → \K$ linear wohldefiniert.
+\end{beispiel-nn}
+
+\begin{definition}
+ Der lineare Raum $X$ heißt \emph{algebraisch reflexiv}, falls $J$ bijektiv ist (und damit $X$ linear isomorph zu $X^{ff}$) ist.
+\end{definition}
+
+\begin{bemerkung}
+ $X$ ist genau dann algebraisch reflexiv, wenn $\dim X < \infty $ ist.
+
+ Im Fall $\dim X < \infty $ lässt sich leicht eine duale Basis angeben:
+ Sei dazu $M \coloneq \{x_1,…,x_n\}$ eine Basis von $X$. Dann wird durch
+ \[
+ \langle x_i, x_k' \rangle \coloneq \delta _{i,k}
+ \]
+ und linearer Fortsetzung die Menge $ M \coloneq \{x_1',…,x_n'\} ⊂ X^f$ erklärt.
+ Dann ist $M'$ eine Basis von $X'$, die die \emph{duale Basis} von $M$ genannt wird.
+ Tatsächlich ist $X^f$ im Falle $\dim X = \infty $ wesentlich größer.
+ Man wählt deshalb eine (neue) Defintion des Dualraums:
+\end{bemerkung}
+
+\begin{definition}[Dualraum]
+ Zu einem linearen Raum $X$ ist
+ \[
+ X' \coloneq \left\{ x' : X → \K, x' \text{ linear und stetig} \right\} ⊂ X^f
+ \]
+ der Dualraum von $X$.
+\end{definition}
+Um Allerdings von Stetigkeit reden zu können, müssen wir zunächst \emph{Topologien} einführen.
+
+%%% Local Variables:
+%%% mode: latex
+%%% TeX-master: "funkana-ebook"
+%%% End:
diff --git a/ch02-topologie.tex b/ch02-topologie.tex
new file mode 100644
index 0000000..b1e79f6
--- /dev/null
+++ b/ch02-topologie.tex
@@ -0,0 +1,566 @@
+\chapter{Topologie}
+\section{Topologische Räume}
+\begin{definition}
+ Sei $X$ eine Menge und $\mathcal T ⊂ \Pot X$ eine Menge von Teilmengen von $X$.
+ $\mathcal T$ heißt eine \emph{Topologie} auf $X$, falls $\mathcal T$ unter endlichen Durchschnitten und beliebigen Vereinigungen abgeschlossen ist.
+ Insbesondere muss $\mathcal T$ $\emptyset$ als leere Vereinigung und $X$ als leeren Schnitt enthalten.
+ $(X,\T)$ heißt dann \emph{topologischer Raum}. Die Elemente von $\T$ heißen \emph{offene Mengen}
+\end{definition}
+\begin{beispiele-nn}
+ \begin{enumerate}[label=(\alph*)]
+ \item
+ Für alle Mengen $X$ ist $\T = \{ ∅, X\}$ eine Topologie auf $X$, die sogenannte \emph{indiskrete Topologie}, \emph{gröbste Topologie} oder auch \emph{Klumpentopologie}.
+ \item
+ Für alle Mengen $X$ ist $\T = \Pot X$ eine Topologie, die sogenannte \emph{diskrete Topologie} oder \emph{feinste Topologie} auf $X$.
+ \item
+ In Analysis I wird eine Menge $U ⊂ ℝ$ für offen erklärt, wenn es zu jedem $x ∈ U$ ein $\epsilon > 0$ gibt, so dass für alle $ y ∈ ℝ$ mit $|x - y| < \epsilon $ auch $y ∈ U$ gilt.
+ Aus der Analysis ist bekannt, dass die so definierten offenen Mengen den Axiomen genügen.
+ Diese Topologie $\Tnat$ wird \emph{natürliche Topologie} genannt.
+ \item
+ Sei $X$ eine beliebige Menge. Die \emph{cofinite Topologie} auf
+ $X$ wird definiert als
+ \[
+ \Tcof = \{ Y ⊂ X: Y = ∅\; \text{oder}\; \complement_X Y\, \text{ist endlich}\}
+ \]
+ \item
+ Der \emph{Sierpinski-Raum} ist die Menge $\{0,1\}$ versehen mit der Topologie $\{ ∅, \{0\}, \{0,1\}\}$.
+ \end{enumerate}
+\end{beispiele-nn}
+
+\begin{definition}
+ Sei $M ⊂ X$.
+ \begin{enumerate}
+ \item
+ $M$ heißt \emph{abgeschlossen}, wenn $X \setminus M$ offen ist.
+ \item
+ $U ⊂ X$ heißt \emph{Umgebung von $A$}, wenn es eine offene Menge $V$ gibt mit $A ⊂ V ⊂ U$. Wir setzen
+ \[
+ \U_A \coloneq \U_A (\T) \coloneq \{ U ⊂ X : U\; \text{Umgebung von $A$}\}.
+ \]
+ $\U_A$ heißt \emph{Umgebungssystem} oder \emph{Umgebungsfilter} von $A ⊂ X$.
+ Für $x ∈ X$ setzen wir $\U_x \coloneq \U_{\{x\}}$. $x$ heißt dann \emph{innerer Punkt} von $U$ für alle $U ∈ \U_x$.
+ \item
+ $x ∈ X$ heißt \emph{Häufungspunkt} von $M$, falls jede Umgebung von $x_0$ ein $y ∈ M$ enthält mit $y \ne x$.k
+ \item
+ Das \emph{Innere von M} ist
+ \[
+ M^\circ \coloneq \bigcup \left\{ U ∈ \T: U ⊂ M \right\}
+ \]
+ die größte offene Menge, die in $M$ enthalten ist.
+ \item
+ Der \emph{Abschluss von} M ist
+ \[
+ \cl M \coloneq \bigcap \left\{ U ⊂ M: U \text{ abgeschlossen} \right\}
+ \]
+ die kleinste abgeschlossene Menge, die $M$ enthält.
+ \item
+ $M$ heißt \emph{kompakt}, falls jede offene Überdeckung von $M$ eine endliche Teilüberdeckung besitzt.
+ \item
+ $M$ heißt \emph{dicht}, falls $\cl M = X$.
+ \item
+ $M$ heißt \emph{nirgends dicht}, falls $(\cl M)^\circ = \emptyset$.
+ \end{enumerate}
+\end{definition}
+\begin{bemerkung}
+ \begin{enumerate}
+ \item $M^\circ ⊂ M ⊂ \cl M$.
+ \item
+ $M^\circ$ ist die Menge der inneren Punkte von $M$.
+ \item
+ $M$ ist genau dann abgeschlossen, wenn $M = \cl M$.
+ \end{enumerate}
+\end{bemerkung}
+
+
+\begin{definition}[Hausdorff-Raum]
+ Sei $(X,\T)$ eine topologischer Raum.
+ Für alle $x,y \in X$ mit $x \neq y$
+ existieren $U \in \U_x, V \in \U_x$ mit $U \cap V = \emptyset$.
+ Dann heißt $(X,\T)$ Hausdorff-Raum bzw. genügt dem $T_2$-Axiom.
+\end{definition}
+
+\begin{beispiele-nn}
+ \begin{enumerate}
+ \item
+ Ein Pseudometrischer Raum $(X,d)$ ist Hausdorff"=Raum genau dann, wenn
+ $d$ eine Metrik ist.
+ \item
+ Der Sierpinski"=Raum $(\{0,1\}),\{\emptyset, \{0\}, \{0,1\}\})$ ist kein Hausdorff"=Raum.
+ \item
+ Sei $X = \prod_{i ∈ I} X_i$ ausgestattet mit dem Produkt $\T$ der Topologien
+ $(T_i)_{i ∈ I}$. $(X,\T)$ ist hausdorffsch genau dann, wenn alle $(X_i,
+ \T_i)$ hausdorffsch sind.
+ \item
+ Ist $(X,\T)$ ein Hausdorff Raum und $Y ⊂ X$, dann ist auch $(Y,\T|Y)$ hausdorffsch.
+ \end{enumerate}
+\end{beispiele-nn}
+
+\begin{definition}[Konvergenz]
+ Eine Folge $\{x_{n}\}_{n \in \N} \subset X$ heißt konvergent gegen $x_{0} \in X$,
+ falls zu jeder Umgebung $U \in \U_{x_{0}}$ ein $n_{0} \in \N$ existiert,
+ sodass $x_{n} \in U$ für alle $n \geq n_{0}$.
+\end{definition}
+\begin{bemerkung-nn}
+ Man überlegt sich leicht, dass der Grenzwert $x_{0}$ in der Regel nicht eindeutig ist.
+ Bsp: In $\T=\{X,\emptyset\}$ konvergiert jede Folge gegen jeden Punkt.
+ Ist $(X,\T)$ jedoch ein Hausdorff-Raum, so ist jeder Grenzwert eindeutig.
+\end{bemerkung-nn}
+\begin{beweis}
+ Seien $x_{0} \neq x'_{0}$ Grenzwerte von $(x_{n})_{n \in \N} \subset X$.
+ Dann existieren disjunkte Umgebungen $U,U' ∈ \U_{x_0}$.
+ Weiterhin gibt es ein $n_{0} \in \N$, so dass $x_{n} \in U$ für alle $n \geq n_{0}$
+ und $n_0' \in \N$, so dass $x_{n} \in U'$ für alle $n \geq n_0'$.
+ Also gilt $x_{\max\{n_0,n'_0\}} \in U \cap U'$
+ Das ist ein Widerspruch zur Disjunktheit der Umgebungen.
+\end{beweis}
+
+\begin{definition}[Häufungspunkt]
+ $x_{0} \in X$ heißt Häufungspunkt von $\{x_{n}\}_{n \in \N} \subset X$,
+ falls zu jeder Umgebung $U \in \U_{x_{0}}$ und für alle $k \in \N$
+ ein $n \geq k \in \N$ existiert, so dass $x_{n} \in U$.
+\end{definition}
+\begin{beispiel-nn}
+ Wir betrachten $\R$ mit der natürlichen Topologie.
+ $(x_n)_{n ∈ ℕ}$ mit $x_n=(-1)^n$ hat zwei Häufungspunkte $\pm 1$.
+ Die Menge aller Folgenglieder $M=\{x_{n}:n \in \N\}=\{-1,1\}$ hat als Menge jedoch keine Häufungspunkte.
+\end{beispiel-nn}
+\begin{bemerkung-nn}
+ Für die indiskrete Topologie ist jeder Punkt in $X$ Häufungspunkt jeder Folge.
+\end{bemerkung-nn}
+
+\begin{definition}[Stetigkeit]
+ Seien $(X, \T_X)$ und $(Y, \T_Y)$ topologische Räume, $f: X → Y$.
+ \begin{enumerate}
+ \item
+ Sei $x_0 ∈ X$.
+ $f$ heißt \emph{stetig in $x_0$}, falls für jede Umgebung $V$ von $f(x_0)$ das Urbild $f^{-1}(V)$ eine Umgebung von $x_0$ ist.
+ \item
+ $f$ heißt \emph{stetig}, falls für alle $V \in \T_{Y}$ gilt, dass $f^{-1}(V) \in \T_{X}$.
+ \end{enumerate}
+\end{definition}
+\begin{bemerkung-nn}
+ $f$ ist genau dann stetig, wenn $f$ in jedem Punkt stetig ist.
+\end{bemerkung-nn}
+
+\begin{definition}[Homöomorphismus]
+ Ist $f : (X,\T_{X}) \rightarrow (Y,\T_{Y})$ bijektiv, stetig,
+ und $f^{-1} : (Y,\T_{Y}) \rightarrow (X,\T_{X})$ auch stetig,
+ dann heißt $f$ (und $f^{-1}$) \emph{Homöomorphismus}.
+ $X$ und $Y$ heißen \emph{homöomorph}, falls so ein Homöomorphismus
+existiert.
+\end{definition}
+
+\begin{definition}[Basis von Topologien und Umgebungen]
+ \begin{enumerate}
+ \item
+ Eine Familie $B \subset \T$ heißt Basis der Topologie in $(X,\T)$, falls $T= \{\bigcup M: M \subset B\}$.
+ \item
+ Eine Familie $B \subset \U_{x}$ von $x \in X$ heißt Umgebungsbasis des Punktes $x$, falls für alle $U \in \T, x \in U$ existiert ein $V \in B$ mit $x \in V \in U$.
+ \end{enumerate}
+\end{definition}
+\begin{beispiel-nn}
+ Für die natürliche Topologie auf $\R^n$ ist eine Basis der Topologie gegeben durch
+ ${B_{\eps}(x): x \in X, \eps > 0}$
+ mit den offenen Kugeln $B_{\eps}(x)={y \in R^n : \norm{x-y}<\eps}$.
+ Sei $x \in \R^n$ fest.
+ Dann ist ${B_{1/n}(x):n \in \N}$ eine abzählbare Umgebungsbasis von x
+\end{beispiel-nn}
+
+\begin{definition}[Relativtopologie oder Spurtopologie]
+ $M \subset \T$ eines topologischen Raumes $(X,\T)$ lässt sich in natürlicher Weise
+ zu einem topologischen Raum machen, nämlich mit $\T' \coloneq \{M \cap V : V \in \T\}$.
+\end{definition}
+\begin{bemerkung-nn}
+ $M = M \cap X \in \T'$ da $X \in \T$, d.h. $M$ ist offen in der Spurtopologie.
+ Achtung: $M$ muss nicht offen in $X$ sein.
+\end{bemerkung-nn}
+
+\begin{definition}
+ Seien zwei Topologien $\T_{1},\T_{2}$ auf X gegeben.
+ Wir sagen $\T_{1}$ ist feiner als $\T_{2}$, falls $\T_{1} \supset \T_{2}$.
+ Wir sagen $\T_{1}$ ist gröber als $\T_{2}$, falls $\T_{1} \subset \T_{2}$.
+ Wir sagen die Topologien sind gleich, falls $\T_{1}=\T_{2}$.
+\end{definition}
+\begin{bemerkung-nn}
+ Sei $\T_{1}$ feiner als $\T_{2}$.
+ Die feinere Topologie $\T_{1}$ enthält mehr offene Mengen,
+ und damit zu jedem Grenzwert $x_{0}$ weniger konvergte Folgen.
+
+ Man zeigt leicht:
+ $\T_{1}$ ist feiner als $\T_{2}$ $\Longleftrightarrow$
+ Für alle $x \in X$ gilt: Seien $B_{1} \subset T_{1},B_{2} \subset T_{2}$ Umgebungsbasen von $x$,
+ dann gilt für alle $U \in B_{1}$, dass ein $V \in B_{2}$ existiert mit $V \subset U$.
+\end{bemerkung-nn}
+
+\begin{beispiel-nn}
+ Folgende Topolgien auf $\R^n$ sind gleich.
+ $\T_{1}$ sei die Topologie, die durch die Kugeln
+ $B_{\eps}(x)=\{y \in R^n : \norm{x-y}<\eps\}$ erzeugt wird.
+ $\T_{2}$ sei die Topologie, die durch die Quader
+ $B_{\eps}(x)=\{y \in R^n : \max_{1 \leq i \leq n} |y_{i}-x_{i}|<\eps\}$ erzeugt wird.
+\end{beispiel-nn}
+
+\begin{definition}[Produkttopologie]
+ Seien $(X,\T_{X}),(Y,\T_{Y})$ topologische Räume.
+ Dann ist die Familie von Mengen
+ \[
+ \{U_{X} \times U_{Y} : U_{X} \in \T_{X}, U_{Y} \in \T_{Y} \} \subset \Pot{X \times Y}
+ \]
+ eine Basis der Topologie $\T_{X \times Y}$ im kartesischen Produkt $X \times Y$.
+ Bemerkung: Es genügt auch wenn $U_{X},U_{Y}$ über Basen von $\T_{X},\T_{Y}$ genommen werden.
+\end{definition}
+
+\section{Metrische Räume}
+\begin{definition}[Pseudometrik, Metrik]
+\label{defi:metrik}
+ Sei $X$ eine Menge. $d: X × X → \R$ heißt \emph{Pseudometrik}, wenn $d$ den
+ folgenden Axiomen genügt:
+ \begin{enumerate}[series=metrik,label=(\textbf{M.\arabic*}),ref=M.\arabic*]
+ \item \label{defi:metrik:m1}
+ Für alle $x, y ∈ X$ gilt $d(x,y) \ge 0$ und $d(x,x) = 0$.
+ \item \label{defi:metrik:m2:symmetrie}
+ \emph{Symmetrie:} Für alle $ x, y ∈ X$ gilt $d(x,y) = d(y,x)$.
+ \item \label{defi:metrik:m3:dreiecksungleichung}
+ \emph{Dreiecksungleichung:} Für alle $x, y, z ∈ X$ gilt $d(x,y)
+ \le d(x,y) + d(z,y)$.
+ \end{enumerate}
+ $d$ heißt \emph{Metrik}, falls es zusätzlich
+ \begin{enumerate}[resume=metrik,label=(\textbf{M.\arabic*}),ref=M.\arabic*]
+ \item \label{defi:metrik:m4:posdef}
+ $d(x,y) = 0 \implies x = y$
+ \end{enumerate}
+ erfüllt. $(X,d)$ heißt dann (pseudo-)metrischer Raum. Zu $x ∈ X$ und $r > 0$
+ definieren wir die \emph{offene Kugel um $x$ mit Radius $r$} als
+ \[
+ B_r(x) \coloneq B^d_r(x) \coloneq \{ y ∈ X: d(x,y) < r\}.
+ \]
+ Die Menge
+ \[
+ \cl{B_r}(x) \coloneq \{y ∈ X: d(x,y) \le r\}
+ \]
+ heißt \emph{abgeschlossene Kugel}.
+\end{definition}
+\begin{satz}
+ Sei $(X,d)$ pseudometrischer Raum. Dann wird durch
+ \[
+ U ∈ \T_d :\Longleftrightarrow ∀ x ∈ U ∃ ε > 0: B_ε(x) ⊂ U
+ \]
+ eine Topologie $\T_d$ definiert, die \emph{von $d$ induzierte Topologie} auf
+ $X$. Die Kugeln $B_r(x)$ für $x ∈ X$ und $r > 0$ sind offen bezüglich dieser
+ Topologie. Ein topologischer Raum $(X,\T)$ heißt
+ \emph{(pseudometrisierbar}, wenn es eine (Pseudo-)Metrik $d$ mit $\T =
+ \T_d$ gibt.
+\end{satz}
+\begin{proof}
+ Sei zunächst $\mathfrak M ⊂ \T_d$ und $x ∈ \bigcup \mathfrak M$. Dann gibt es
+ $x ∈ M ∈ \mathfrak M$, da $M$ offen ist, enthält $M$ einen Ball $B_ε(x)$. Somit auch
+ $B_ε(x) ⊂ M ⊂ \bigcup \mathfrak M$. Da dies für alle $x ∈ \bigcup \mathfrak M$ gilt, ist $\bigcup \mathfrak M$ offen.
+
+ Sei nun $\mathfrak M ⊂ \T_d$ endlich und $x ∈ \bigcap \mathfrak M$. Dann
+ gibt es zu jedem $M_i ∈ \mathfrak M$ ein $ε_i > 0$ mit $B_{ε_i}(x) ⊂ M_i$. Somit ist
+ für $ε \coloneq \min\limits_{i} ε_i $ auch $B_ε(x) ⊂ \bigcup \mathfrak M$.
+ Also ist $\T_d$ eine Topologie.
+
+ Jetzt zu $B_r(x) ∈ \T_d$. Sei dazu $y ∈ B_r(x)$, also $δ \coloneq r - d(x,y) > 0$
+ Sei $z ∈ B_δ(y)$, also $d(z,y) < δ$. Mit der Dreiecksungleichung ist dann
+ $d(x,z) < d(x,y) + d(y,z) = d(x,y) + r - d(x,y) = r$. Also $y ∈ B_δ(y) ⊂ B_r(x)$.
+\end{proof}
+\begin{bemerkung-nn}
+ Die abgeschlossene Kugel $\cl B_r (x)$ ist im Allgemeinen nicht der Abschluss der offenen Kugel $B_r(x)$, aber es gilt immer
+ \[
+ \cl{B_r(x)} ⊂ \cl{B_r}(x).
+ \]
+\end{bemerkung-nn}
+\begin{satz}
+ Sei $(X,d)$ ein metrischer Raum.
+ Dann genügt $(X,\T_d)$ dem $T_2$-Axiom.
+\end{satz}
+\begin{proof}
+ Seien $x \ne y ∈ X$. Dann ist $δ \coloneq d(x,y) > 0$.
+ Dann sind $B_{δ/2}(x)$ und $B_{δ/2}(y)$ disjunkte Umgebungen von $x$ bzw $y$:
+ Sei $z ∈ B_{δ/2}(x)$. Dann ist
+ \[
+ d(z,y) \ge d(y,x) - d(x,z) > δ - \frac δ 2 = \frac δ 2.
+ \]
+\end{proof}
+\begin{lemma-nn}[Eigenschaften metrischer Räume]
+ Sei $(X,d)$ ein metrischer Raum.
+ \begin{enumerate}
+ \item Jeder Punkt $x ∈ X$ besitzt eine abzählbare Umgebungsbasis
+ \[
+ \{ B_{1/n} (x), n ∈ ℕ\}.
+ \]
+
+ \item
+ Es gilt
+ \[
+ \lim_{n \to \infty } x_n = x \; \Longleftrightarrow \; \lim_{n→\infty } d(x,x_n) = 0.
+ \]
+ \item
+ Es ist $x_0 ∈ M$ genau dann ein innerer Punkt von $M ⊂ X$, wenn ein $\epsilon > 0$ existiert mit $B_\epsilon (x_0) ⊂ M$.
+ \item
+ $M$ ist nirgends dicht in $X$ genau dann, wenn es zu jeder Kugel
+$B_\epsilon (x_0)$ mit $x_0 ∈ X, \epsilon > 0$ eine Kugel $B_\delta (x_1) ⊂
+B_\epsilon (x_0)$ mit $B_\delta(x_1) ∩ M = \emptyset$ gibt.
+ \item
+ Seien $(X,d_X)$ und $(Y,d_Y)$ metrische Räume.
+ Dann ist auch $(X×Y,d_{X×Y})$ ein metrischer Raum vermöge der Metrik
+ \[
+ d_{X×Y}((x_1,y_1),(x_2,y_2)) \coloneq \max\{d_x(x_1,x_2),d_y(y_1,y_2)\}
+ \]
+ oder auch mit
+ \[
+ d_{X×Y}((x_1,y_1),(x_2,y_2)) \coloneq \sqrt{d_x^2(x_1,x_2)+d_y^2(y_1,y_2)}.
+ \]
+ Tatsächlich induzieren diese beiden Metriken die gleiche Topologie (nämlich die Produkttopologie)
+ \item
+ Homöomorphismen $f: X → Y$ (für metrische Räume $X, Y$), die die Metrik respektieren, das heißt
+ \[
+ d_X(x_1,x_2) = d_Y(f(x_1),f(x_2)) \quad ∀x_1, x_2 ∈ X
+ \]
+ heißen \emph{Isometrien}.
+ \item
+ Ein metrischer Raum muss im allgemeinen keine lineare Struktur haben.
+ Man betrachte hierzu die Menge $X \coloneq \{1,2,3,4,5,6\}$ mit der diskreten Metrik.
+ Diese kann keine Vektorraumstruktur haben, da $|X| = 6$ keine Primzahlpotenz ist.
+ \end{enumerate}
+\end{lemma-nn}
+\begin{proof}
+Der Beweis wird aufgrund seiner Trivialität den Lesern zur Übung überlassen, da er wirklich nur Einsetzen der Definitionen ist.
+\end{proof}
+
+Nun ein paar Charakterisierungen von kompakten Mengen in metrischen Räumen.
+\begin{satz}
+ Im metrischen Raum $(X,d)$ sind äquivalent:
+ \begin{enumerate}
+ \item
+ $K ⊂ X$ ist kompakt (überdeckungskompakt)
+ \item
+ Jede Folge in $K$ besitzt mindestens einen Häufungspunkt in $K$ (abzählbar kompakt)
+ \item
+ Jede Folge in $K$ besitzt eine konvergente Teilfolge mit Grenzwert in $K$ (folgenkompakt)
+ \end{enumerate}
+\end{satz}
+
+\begin{bemerkung}
+ Der Satz gilt so im allgemeinen Hausdorff-Raum \emph{nicht}.
+ Für „$(b) \Rightarrow (a)$“ benötigt man zusätzlich das zweite Abzählbarkeitsaxiom, also die Existenz einer abzählbaren Basis der Topologie.
+ Für „$(b) \Rightarrow (c)$“ benötigt man das erste Abzählbarkeitsaxiom, also die Existenz von abzählbaren Umgebungsbasen für jeden Punkt.
+\end{bemerkung}
+
+\section{Vollständigkeit in metrischen Räumen und der Satz von Baire}
+\begin{definition}
+ Eine Folge $(x_n)_{n ∈ ℕ} ⊂ X$ in $(X,d)$ heißt \emph{Cauchy-Folge}, falls zu jedem $\epsilon > 0$ ein $N = N(\epsilon )$ existiert mit $d(x_m,x_n) < \epsilon $ für alle $n,m \ge N$.
+\end{definition}
+
+\begin{lemma}
+ Jede konvergente Folge $(x_n)_{n ∈ ℕ} ⊂ X$ ist auch eine Cauchy-Folge.
+\end{lemma}
+\begin{proof}
+ Sei etwa $\lim_{n→∞} x_n = x$. Sei $ε > 0$.
+ Da $(x_n)_{n ∈ ℕ}$ gegen $x$ konvergiert, gibt es $N ∈ ℕ$ mit $d(x_n,x)< ε/2$ für alle $n ≥ N$, also mit der Dreiecksungleichung
+ \[
+ ∀n,m ≥ N: d(x_n,x_m) ≤ d(x_n,x) + d(x, x_m) < \frac ε 2 + \frac ε 2 = ε.
+ \]
+\end{proof}
+
+\begin{definition}
+ Der metrische Raum $(X,d)$ heißt \emph{vollständig}, falls jede Cauchy-Folge in $(X,d)$ konvergiert.
+\end{definition}
+
+Nicht jeder metrische Raum braucht vollständig zu sein (man betrachte hierfür z.B. $ℚ$ und die Folge der Partialsummen der Dezimalbruchentwicklung von $\sqrt 2$),
+jedoch lässt sich jeder metrische Raum zu einem vollständigen Erweitern.
+
+\begin{satz}
+ Jeder metrische Raum $(X,d)$ lässt sich in einen bis auf Isometrie eindeutig bestimmten kleinsten vollständigen metrischen Raum $(\tilde X, \tilde d)$ einbetten.
+ Dieser Raum $(\tilde X, \tilde d)$ heißt die Vervollständigung von $(X,d)$.
+\end{satz}
+\begin{proof}
+ Zwei Cauchyfolgen $(x_n)_{n ∈ ℕ}$ und $(y_n)_{n ∈ ℕ}$ seien äquivalent, falls $d(x_n,y_n) \xrightarrow[n → \infty ]{} 0$.
+ Hierdurch ist eine Äquivalenzrelation definiiert. Sei $[(x_n)_{n ∈ ℕ}]$ die vom Repräsententaten $(x_n)_{n ∈ ℕ}$ erzeugte Klasse. Man setzt
+ \[
+ \tilde X \coloneq \{ [ (x_n)_{n ∈ ℕ}] : (x_n)_{n ∈ ℕ} \text{ ist Cauchy-Folge in }(X,d)\}
+ \]
+ und
+ \[
+ \tilde d([(x_n)_{n ∈ ℕ}],[(y_n)_{n ∈ ℕ}]) \coloneq \lim_{n → \infty } d(x_n,y_n).
+ \]
+ Dann ist $(d(x_n,y_n))_{n ∈ ℕ}$ eine Cauchy-Folge in $ℝ$, da
+ \[
+ |d(x_n,x_m) - d(y_m,y_m) | \le \underbrace{d(x_n,x_m)}_{→ 0} + \underbrace{d(y_n,y_m)}_{→ 0}.
+ \]
+ Da $ℝ$ bekanntlich vollständig ist, existiert somit der Grenzwert.
+ Ferner ist $\tilde d$ Repräsentatenunabhängig, also wohldefiniert:
+ Seien $(\tilde x_n)$ und $(\tilde y_n)$ andere Repräsentaten. Dann ist
+ \[
+ d(x_n,y_n) \le \underbrace{d(x_n,\tilde x_n)}_{→ 0} + d(\tilde x_n,\tilde y_n) + \underbrace{d(\tilde y_n, y_n)}_{→ 0}.
+ \]
+ Die umgekehrte Ungleichung ergibt sich aus Vertauschung der Rollen. Man rechnet leicht nach, dass $(\tilde X, \tilde d)$ ein vollständiger Raum ist.
+ Wir können $(X,d)$ durch die entsprechenden konstanten Folgen isometrisch in $\tilde X$ einbetten.
+\end{proof}
+
+\begin{bemerkung-nn}
+ Wendet man diese Technik auf $ℚ$ mit der natürlichen Metrik an, dann erhält man $(ℝ,d)$ als vollständige Hülle.
+ Man beachte jedoch, dass dies nicht für die Konstruktion von $ℝ$ ausreicht, da hier schon die Existenz von $ℝ$ verwenden wird -- Aber das funktioniert größtenteils analog.
+\end{bemerkung-nn}
+
+
+\begin{satz}[Schachtelsatz]\label{schachtelsatz}
+ Sei $(X,d)$ ein vollständiger metrischer Raum und seien
+ $(x_n)_{n * ℕ} ⊂ X$ und $(r_n)_{n ∈ ℕ} ⊂ (0,\infty ) $ Folgen mit der Eigenschaft
+ \begin{enumerate}
+ \item $\cl B_{r_{n+1}}(x_{n+1}) ⊂ B_{r_n} (x_n)$
+ \item $\lim_{n \to \infty } r_n = 0$.
+ \end{enumerate}
+ Dann gibt es genau ein $x_0 ∈ X$ mit $x_0 ∈ \bigcap_{n ∈ ℕ} \cl B_{r_n} (x_n)$.
+\end{satz}
+\begin{proof}
+
+ Für $p ∈ ℕ$ beliebig gilt
+ \[
+ \cl B_{r_{n+p}} (x_{n+p}) ⊂ \cl B_{r_n} (x_n).
+ \]
+ Also
+ \[
+ d(x_{n+p},x_n) \le r_n \xrightarrow[n → \infty ]{} 0.
+ \]
+ Damit ist $(x_n){n ∈ ℕ}$ eine Cauchyfolge und damit konvergiert gegen ein $x_0 ∈ X$.
+ Außerdem gilt
+ \[
+ d(x_p,x_n) \le \underbrace{d(x_0, x_{n+p})}_{→ 0 (p → \infty )} + \underbrace{d(x_{n+p},x_n)}_{ \le r_n}.
+ \]
+ Damit folgt für $p → \infty $
+ \[
+ d(x_0, x_n) \le r_n \quad ∀ n ∈ ℕ
+ \]
+ also $x_0 ∈ \bigcap_{n ∈ ℕ} \cl B_{r_n}(x_n)$.
+ Für die Eindeutigkeit sei $\tilde x_0$ ebenfalls in $\bigcap_{n ∈ ℕ} \cl B_{r_n}(x_n)$.
+ Dann folgt
+ \[
+ d(x_0,\tilde x_0) \le \underbrace{d(x_0,x_n)}_{\le r_n} + \underbrace{d(x_n, \tilde x_0)}_{\le r_n} \le 2r_n \xrightarrow[n → \infty ]{} 0.
+ \]
+ Doch damit war bereits $x_0 = \tilde x_0$.
+\end{proof}
+
+\begin{definition}
+ Eine Teilmenge $M$ eines metrischen Raumes $(X,d)$ heißt \emph{von erster Kategorie} oder \emph{mager}, falls sie
+ die Vereinigung abzählbar vieler in $X$ nirgends dichter Mengen ist. Andernfalls heißt $M$ \emph{von zweiter Kategorie}.
+\end{definition}
+
+Der folgende Satz wird beim Beweis mehrerer fundamentaler Sätze benötigt, z.B beim Prinzip der gleichmäßigen Beschränktheit oder dem Open-Mapping-Theorem.
+
+
+\begin{satz}[Baire]\label{baire}
+ Jede nichtleere offene Menge eines vollständigen metrischen Raumes $(X,d)$ ist von zweiter Kategorie (insbesondere $X$ selbst)
+\end{satz}
+\begin{proof}
+ Sei $ \emptyset \ne M ⊂ X$ offen. Wir nehmen umgekehrt an, $M$ wäre von erster Kategorie, das heißt
+ \[
+ M ⊂ \bigcup_{n ∈ ℕ} M_n
+ \]
+ mit $M_n ⊂ X$ nirgends dicht. Wähle $x_0 ∈ M$. Da $M$ offen ist, gibt es ein $r = r_0 > 0$ mit $B_{r_0}(x_0) ⊂ M$.
+ Da $M_1$ nirgends dicht ist, gibt es $r_1 > 0$ und $x_1 ∈ X$ mit
+ \[
+ B_{r_1}(x_1) ⊂ B_{r_0/2} (x_0)
+ \]
+ und $B_{r_1}(x_1) ∩ M_1 = \emptyset$.
+ Analog finden wir, da $M_2$ nirgends dicht ist, $r_2 > 0$ und $x_2 ∈ X$ mit
+ \[
+ B_{r_2}(x_2) ⊂ B_{r_1/2} (x_1)
+ \]
+ und $B_{r_2}(x_2) ∩ M_2 = \emptyset$.
+ Durch Fortsetzen dieses Schemas finden wir eine Folge $(x_n)_{n ∈ ℕ} ⊂ X$ und Radien $(r_n)_{n ∈ ℕ} ⊂ (0,\infty )$ mit $r_n \le r/2^n \xrightarrow[n → \infty ]{} 0$.
+ Damit sind alle Voraussetzungen von \cref{schachtelsatz} erfüllt. Folglich existiert genau ein
+ \[
+ \tilde x ∈ \bigcap_{n ∈ ℕ} B_{r_n} (x_n) ⊂ B_r(x_0) ⊂ M.
+ \]
+ Aber $\tilde x \not\in M_n$ für alle $n ∈ ℕ$ Folglich ist auch $\tilde x$ nicht in $\bigcup_{n ∈ ℕ} M_n = M$. Das ist ein Widerspruch. Also ist $M$ von zweiter Kategorie.
+\end{proof}
+
+% \begin{satz}[Satz von Baire]\label{44-baire}
+% Sei $(X,\T)$ ein vollständig metrisierbarer oder lokalkompakter Hausdorff\hyp{}Raum
+% \begin{enumerate}
+% \item
+% Sei $(U_n)_{n ∈ ℕ}$ eine Folge offener, dichter Teilmengen von $X$.
+% Dann ist auch $\bigcap_{n ∈ ℕ} U_n ⊂ X$ dicht.
+% \item
+% Sei $(A_n)_{n ∈ ℕ}$ eine Folge nirgends dichter Teilmengen. Dann ist
+% $\bigcup_{n ∈ ℕ} A_n \ne X$.
+% \item
+% Sei $(A_n)_{n ∈ ℕ}$ eine Folge abgeschlossener Teilmengen mit
+% $\bigcup_{n ∈ ℕ} A_n = X$. Dann gilt für mindestens ein $n ∈ ℕ$, dass $A_n^\circ
+% \ne \emptyset$.
+% \end{enumerate}
+% \end{satz}
+% \begin{proof}
+% \begin{enumerate}
+% \item
+% Sei $W ⊂ X$ offen und nichtleer. Zu zeigen: $\bigcap_{n ∈ ℕ} U_n ∩ W \ne
+% \emptyset$. Sei zunächst $X$ vollständig metrisierbar durch die Metrik
+% $d$. Da $U_1 ∩ W$ offen und nichtleer nach Annahme gibt es $x_1 ∈ U ∩ W$
+% und $0 < r_1 < 1$ mit $B_{r_1}(x_1) ⊂ U_1 ∩ W$. Wir wählen nun induktiv
+% Punkte $x_n ∈ X$ und Zahlen $0 < r_n < 1$ (für $n \ge 2$) mit folgenden
+% Eigenschaften:
+% \begin{enumerate}[label=(\roman*)]
+% \item
+% $0 < r_n < \frac 1 n$
+% \item
+% $\cl{B_{r_n}(x_n)} ⊂ U_n ∩ B_{r_{n-1}} (x_{n-1})$
+% \end{enumerate}
+% Dazu beachte man, dass $U_n ∩ B_{r_{n-1}}(x_{n-1})$ nichtleer und offen
+% ist, also existiert $x_n$ und $\frac 1 n > \epsilon > 0$ mit $B_\epsilon (x_n) ⊂ U_n ∩
+% B_{r_{n-1}} (x_{n-1})$ und $r_n = \frac \epsilon 2$ ist wie gewünscht. Für $m
+% \ge n$ impliziert (ii), dass $x_m ∈ B_{r_n}(x_n)$ und aus (i) folgt,
+% dass die Folge $(x_n)_{n ∈ ℕ}$ damit eine Cauchyfolge ist. Damit
+% konvergiert $(x_n)_{n ∈ ℕ}$ gegen ein $x ∈X$. Sei nun $N ∈ ℕ$ und $m >
+% N$. Dann folgt aus $x_m ∈ B_{r_N}(x_N)$, dass
+% \begin{align*}
+% x &= \lim_{m → \infty } x_m ∈ \cl{B_{r_N}(x_n)} ⊂ U_N ∩ B_{r_{N-1}}(x_{N-1}) \\
+% & ⊂ U_N ∩ B_{r_1}(x_1) ⊂ U_N ∩ W,
+% \end{align*}
+% also $x ∈ \bigcap_{n ∈ ℕ} U_N ∩ W$.
+
+% Sei Nun $X$ lokalkompakt. Da $U_1 ∩ W$ offen und nichtleer ist, gibt es
+% $x ∈ U_1 ∩ W$, und es ist $U_1 ∩ W ∈ \U_x$. Wähle $B_1 ∈ \U_x$ kompakt
+% mit $B_1 ⊂ U_1 ∩ W$. Wir konstruieren nun sukzessive kompakte Mengen
+% $B_k$ (zu $k \ge 2$) mit folgenden Eigenschaften:
+% \begin{enumerate}[label=(\roman*)]
+% \item
+% $B_k ⊂ B_{k-1}$
+% \item
+% $\emptyset \ne B_k^\circ ⊂ B_k ⊂ U_k$.
+% \end{enumerate}
+% Ist $B_{k-1}$ gegeben, so ist wegen $B_{k-1}^\circ \ne \emptyset$ und
+% der Dichtheit von $U_k$ der Schnitt $B_{k-1}^\circ ∩ U_k$ offen und
+% nichtleer. Es gibt also ein $x ∈ B_{k-1}^\circ ∩ U_k$ und damit auch
+% eine kompakte $x$-Umgebung $B_k ⊂ U_k ∩ B_{k-1}$.
+% Die Familie $(B_k)_{k ∈ ℕ}$ nichtleerer Teilmengen der kompakten Menge
+% $B_1$ ist absteigend, besitzt also die endliche Durschnittseigenschaft.
+% Da $X$ hausdorffsch und die $B_k$ kompakt sind, ist zudem jedes $B_k$
+% abgeschlossen, somit folgt
+% \[
+% \emptyset \ne \bigcap_{k ∈ ℕ} B_k ⊂ \bigcap _{k ∈ ℕ}U_k
+% \]
+% sowie
+% \[
+% \bigcap_{k ∈ ℕ} B_k ⊂ B_1 ⊂ W.
+% \]
+% Insgesamt also
+% \[
+% \emptyset \ne \bigcap_{k ∈ ℕ} B_k ⊂ \bigcap_{k ∈ ℕ} U_k ∩ W.
+% \]
+% \item
+% Für $n ∈ ℕ$ Sei $U_n = \complement_X \cl{A_n}$. Dann ist $U_n$ offen und
+% dicht. Mit Teil (a) folgt, dass auch $\bigcap_{n ∈ ℕ U_n }$ dicht, und
+% somit insbesondere nicht leer, ist. Also ist $\bigcup_{n ∈ ℕ} A_n
+% ⊂ \bigcup_{n ∈ ℕ} \cl{A_n} = (\bigcap_{n ∈ ℕ} U_n)^\complement \ne X$.
+% \item
+% Das ist eine direkte Konsequenz aus (b).
+% \end{enumerate}
+% \end{proof}
+
+
+%%% Local Variables:
+%%% mode: latex
+%%% TeX-master: "funkana-ebook"
+%%% End:
diff --git a/ch03-topologisch-lineare-raeume.tex b/ch03-topologisch-lineare-raeume.tex
new file mode 100644
index 0000000..d58b433
--- /dev/null
+++ b/ch03-topologisch-lineare-raeume.tex
@@ -0,0 +1,1480 @@
+\chapter{Topologische lineare Räume}
+Erklärtes Ziel dieses Kapitels wird sein, die beiden Strukturen aus den vorherigen beiden Kapiteln, also die Topologie und den linearen Raum zusammenzuführen.
+\begin{definition}
+ Ein linearer Raum $X$ über dem Körper $\K$ mit Topologie $\T$ heißt \emph{topologischer linearer Raum}, falls die Vektorraumoperationen ($+ : X×X → X$ und $\cdot: \K×X → X$) stetig sind.
+\end{definition}
+
+\begin{bemerkung-nn}
+ Stetigkeit der Vektorraumoperationen sollte als minimales Kompatibilitätskriterium der beiden Strukturen gefordert werden.
+ Tatsächlich ist es im Allgemeinen gar nicht erfüllt. Erst im normierten Raum bekommt man diese Stetigkeit geschenkt.
+\end{bemerkung-nn}
+
+\section{Normierte Räume}
+\begin{definition}
+ Sei $X$ ein linearer Raum über $\K$. Die Abbildung $\norm\cdot: X → [0,\infty )$
+ heißt \emph{Norm} auf $X$, falls für alle $x, y ∈ X, \alpha ∈ K$ gilt:
+ \begin{enumerate}
+ \item $\norm x = 0 \Longleftrightarrow x = 0$ (Definitheit)
+ \item
+ $\norm{\alpha x} = |\alpha | \norm x$ (Homogenität)
+ \item
+ $\norm{x+y} \le \norm x + \norm y$ (Dreiecksungleichung)
+ \end{enumerate}
+ $(X,\norm\cdot)$ heißt dann \emph{normierter Raum}.
+\end{definition}
+
+\begin{bemerkung}
+ Durch $d(x,y) \coloneq \norm{x-y}$ wird ein normierter Raum auch ein metrischer, also insbesondere auch ein topologischer Raum.
+ Diese induzierte Topologie auf $(X, \norm\cdot)$ heißt \emph{Normtopologie}.
+
+ Ohne die lineare Struktur macht der normierte Raum gar keinen Sinn, da für die Definition einiger der Normaxiome die Vektorraumoperationen verwendet werden.
+\end{bemerkung}
+
+\begin{beispiele}
+ \begin{enumerate}
+ \item
+ Betrachte den $ℝ^n$ mit $\norm x _{p} \coloneq \left( \sum_{i=1}^n |x_i|^p \right)^{1/p}$ mit $1 \le p < \infty $ ist ein normierter Raum,
+ genauso wie mit $\norm{x}_{\infty } \coloneq \max_{1 \le i \le n} |x_i|$.
+ Insbesondere gibt es im $ℝ^n$ überabzählbar viele verschiedene Normen.
+ Wir werden jedoch später sehen, dass diese Normen alle die gleiche Topologie erzeugen.
+ \item
+ Der Raum aller stetigen Funktionen auf einem kompaktem Intervall $C[a,b]$ mit $\norm{x}_{\infty } \coloneq \max_{t ∈ [a,b]} |x(t)|$ ist ein normierter Raum.
+ Außerdem wird durch
+ \[
+ \norm x \coloneq ∫_a^b |x(t)| dt
+ \]
+ ebenfalls eine Norm definiert.
+ \item
+ Sei $\Omega ⊂ ℝ^n$ offen und beschränkt. Dann wird $C(\cl{\Omega})$ mit
+ \[
+ \norm{x}_{\infty } \coloneq \max_{t ∈ \cl \Omega} |x(t)|
+ \]
+ auch zu einem normierten Raum.
+ \item
+ $L^p(\Omega) = \L^p(\Omega)/\mathcal N$, wobei $\mathcal N = \{ f: \Omega → \R, f(t) = 0 \text{ fast überall}\}$ ist mit
+ \[
+ \norm x \coloneq \left(∫_{\Omega} |x(t)|^p dt \right)^{1/p}
+ \]
+ ein normierter Raum, wobei $1 \le p < \infty $.
+ \item
+ $\ell^p$ mit
+ \[
+ \norm x _{p} \coloneq \left( \sum_{i=1}^n |x_i|^p \right)^{1/p}
+ \]
+ ist ebenfalls ein normierter Raum, wobei $1 \le p < \infty $.
+ \end{enumerate}
+\end{beispiele}
+
+\begin{lemma}
+ Sei $(X, \norm\cdot)$ ein normierter Raum. Dann sind die Abbildungen $+$, $\cdot$ und $\norm\cdot$ stetig.
+\end{lemma}
+\begin{proof}
+ Für beliebige Folgen $(x_n)_{n ∈ ℕ},(y_n)_{n ∈ ℕ} ⊂ X, (\alpha _n)_{n ∈ ℕ}$ mit $\lim x_n = x$, $\lim y_n = y$, $\lim \alpha _n = \alpha $ gelten
+ \[
+ \norm{(x_n + y_n) - (x+y)} \le \norm{x-x_n} + \norm{y -y_n}
+ \]
+ sowie
+ \[
+ \norm{\alpha _nx_n - \alpha x} \le |\alpha _n| \norm{x_n-x} + \norm{x} |\alpha _n - \alpha |
+ \]
+ und
+ \[
+ |\norm{x_n} - \norm{x}| \le \norm{x_n - x}
+ \]
+ nach der umgekehrten Dreiecksungleichung.
+ Folglich sind die zu betrachtenden Abbildungen alle folgenstetig, und, da metrische Räume stets dem ersten Abzähhlbarkeitsaxiom genügen, auch stetig.
+\end{proof}
+
+\begin{korollar}
+ Jeder normierte Raum versehen mit der Normtopologie ist ein topologischer linearer Raum.
+ Deshalb ist auch keine Unterscheidung zwischen normierten Räumen und normierten topologischen linearen Räumen nötig.
+\end{korollar}
+
+\section{Topologische lineare Räume}
+\begin{bemerkung-nn}
+ Hierbei sei stets die Topologie von $X×X$ die Produktopologie, bei den Körpern $\K = \begin{cases} ℝ \\ ℂ \end{cases}$ die übliche Topologie.
+ Wir schreiben im Folgenden für Mengen $M, M_1, M_2 ⊂ X$ und $\alpha ⊂ \K$ nun
+ \[
+ M_1 + M_2 \coloneq s(M_1,M_2) \coloneq \{x+y : x ∈ M_1, y ∈ M_2\},
+ \]
+ \[
+ A \cdot M \coloneq m(A,M) \coloneq \{ \alpha x: \alpha ∈ A, x ∈ M\}.
+ \]
+\end{bemerkung-nn}
+
+\begin{lemma}
+ Hat der topologische Raum $(X,\T)$ auch eine lineare Struktur, so sind äquivalent:
+ \begin{enumerate}
+ \item Die Addition $s$ ist stetig.
+ \item
+ Für beliebiges $x, y ∈ X$ gilt: Zu jeder Umgebung $O_{x+y} ∈ \T$ existieren Umgebungen $O_x ∈ \T$ von $x$ und $O_y ∈ \T$ von $y$ mit $O_x + O_y ⊂ O_{x+y}$
+ \end{enumerate}
+\end{lemma}
+\begin{proof}
+ $s$ ist stetig in $(x,y)$ genau dann, wenn zu jeder Umgebung $O_{x,y} ∈ \T_X$
+ von $(x,y)$ existiert eine Umgebung $U ⊂ \T_{X×X}$ von $(x,y)$ mit $s(U) ⊂ O_{x+y}$.
+ Nach Definition der Produkttopologie existieren dann Umgebungen $O_x ∈ \U_x$ und $O_y ∈ \U_y$ mit $O_x × O_y ⊂ U$.
+ Damit ist
+ \[
+ O_x + O_y = s(O_x, O_y) = s(O_x × O_y) ⊂ s(U) ⊂ O_{x+y}.
+ \]
+ Analog zeigt man die entsprechende Aussage für die skalare Multiplikation:
+\end{proof}
+\begin{lemma}
+ Hat der topologische Raum $(X,\T)$ auch eine lineare Struktur, so sind äquivalent:
+ \begin{enumerate}
+ \item Die Addition $m$ ist stetig.
+ \item
+ Für beliebiges $\alpha ∈ \K, x ∈ X$ gilt: Zu jeder Umgebung $O_{\alpha x} ∈ \T$ existieren Umgebungen $O_x ∈ \T$ von $x$ und $O_\alpha ∈ \T$ von $y$ mit $O_\alpha × O_x ⊂ O_{\alpha x}$.
+ \end{enumerate}
+\end{lemma}
+
+Betrachtet man insbesondere die Stetigkeit am Punke $\alpha =0$ und $x ∈ X$ beliebig, dann gilt also:
+Für jede Umgebung $O_0 ∈ \U_0 ⊂ X$ existiert eine Umgebung $O_x ∈ \U_x$ und ein $r > 0$, so dass
+\[
+ ∀β: |β| <r: βO_x ⊂ O_0.
+\]
+Unmittelbar daraus erhalten wir folgendes Korollar:
+\begin{korollar}
+ Im topologischen Raum $(X,\T)$ gilt für $x ∈ X$ beliebig und $(β_n)_{n ∈ ℕ} ⊂ ℝ$
+ \[
+ β_n \xrightarrow[n → \infty ]{} 0 \implies β_nx \xrightarrow[n → \infty ]{} 0.
+ \]
+\end{korollar}
+
+\begin{definition-nn}
+ \begin{enumerate}
+ \item
+ Zu $x_0 ∈ X$ fest definieren wir den Translationsoperator
+ \[
+ T_{x_0} \coloneq X → X, x ↦ x + x_0.
+ \]
+ \item
+ Zu $\alpha _0 ∈ \K^*$ fest definieren wir den Multiplikationsoperator
+ \[
+ M_{\alpha _0} \coloneq X → X, x ↦ \alpha _0\cdot x.
+ \]
+ \end{enumerate}
+\end{definition-nn}
+
+\begin{lemma}
+ Die Translationsoperatoren und Multiplikationsoperatoren sind Homöomorphismen.
+\end{lemma}
+\begin{proof}
+ Das ist klar.
+\end{proof}
+
+\begin{korollar}[Invarianzprinzip]
+ Im topologischen linearen Raum $(X,\T)$ ist die Topologie bereits durch die offenen Umgebungen von $0 ∈ X$ bestimmt. Alle anderen offenen Mengen entstehen durch Translation.
+\end{korollar}
+\begin{proof}
+ Das ist klar.
+\end{proof}
+
+\section{Metrische lineare Räume und Quasi-normierte Räume}
+\begin{definition}
+ Eine Metrik $d: X × X → ℝ$ auf einem linearen Raum $X$ heißt \emph{translationsinvariant}, falls gilt:
+ \[
+ ∀x,y,z ∈ X: d(x,y) = d(x+z, y+z),
+ \]
+ oder äquivalent dazu:
+ \[
+ ∀x,y ∈ X: d(x,y) = d(x-y, 0).
+k \]
+\end{definition}
+\begin{bemerkung-nn}
+ Ohne lineare Struktur macht das gar keinen Sinn!
+\end{bemerkung-nn}
+
+\begin{definition}
+ Ein metrischer Raum $(X,d)$ mit linearer Struktur und translationsinvarianter Mertik $d$ heißt \emph{metrischer linearer Raum}, falls
+ die Vektorraumoperationen stetig sind (in der von der Metrik induzierten Topologie).
+\end{definition}
+
+
+\begin{lemma}
+ Im metrischen Raum $(X,d)$ mit linearer Struktur und translationsinvarianter metrik, dann ist die Addition immer stetig.
+\end{lemma}
+\begin{proof}
+ Es genügt, da in metrischen Räumen Folgenstetigkeit und Stetigkeit äquivalent sind, zu zeigen, dass $\lim d(x_n + y_n, x + y ) = 0$, sofern $\lim d(x_n,x) = 0$ und $\lim d(y_n,y) = 0$.
+ Dazu ist
+ \[
+ d(x_n+y_n,x+y) \le d(x_n+y_n) + d(x+y_n, x+y) = d(x_n,x) +d(y_n,y) \xrightarrow[n → \infty ]{} 0.
+ \]
+\end{proof}
+
+\begin{beispiel-nn}
+ Sei $X = C(a,b)$ mit der Metrik
+ \[
+ d(x,y) \coloneq \min\{ 1, \sup_{t ∈ (a,b)} |x(t)-y(t)|\}.
+ \]
+ Dann ist $d$ eine translationsinvariante Metrik, aber $X$ ist kein linearer Raum, da die Skalarmultiplikation nicht stetig ist.
+\end{beispiel-nn}
+Für die Stetigkeit der skalaren Multiplikation im Punkt $(\alpha ,x) ∈ \K × X$ hat man (nach dem $\epsilon -\delta$ -Kriterium)
+\[
+ ∀\epsilon > 0 ∃ \delta > 0 ∃ r> 0 ∀β ∈ \K ∀y ∈ X:
+ \begin{rcases}
+ |β - \alpha | < r \\
+ d(x,y) < \delta
+ \end{rcases}
+ \implies d(βy,\alpha x) < \epsilon
+\]
+
+
+\begin{lemma}
+ \label{lemma-metrischer-linearer-raum-charak}
+ Sei $(X,d)$ ein metrischer Raum mit linearer Struktur und mit einer translationinvarianten Metrik.
+ Dann ist $X$ mit der von $d$ erzeugten Topologie ein \emph{metrischer linearer Raum} genau dann, wenn für alle $\alpha ∈ \K, x ∈ X$ und beliebige Nullfolgen $(x_n)_{n ∈ ℕ} ⊂ X, (\alpha _n)_{n ∈ ℕ)} ⊂ \K$ gilt
+ \begin{gather*}
+ \alpha x_n \xrightarrow[n → \infty ]{} 0 \\
+ \alpha x_n \xrightarrow[n → \infty ]{} 0 \\
+ \alpha _nx_n \xrightarrow[n → \infty ]{} 0
+ \end{gather*}
+\end{lemma}
+\begin{proof}
+ „$⇒$”: Skalare Multiplikation ist im metrischen linearen Raum stetig, also folgen die Aussagen sofort.
+
+ „$⇐$“: Wegen der Äquivalenz von Stetigkeit und Folgenstetigkeit ist zu zeigen
+ \[
+ \begin{rcases}
+ \alpha _n \xrightarrow[n → \infty ]{} \alpha ∈ \K \\
+ x_n \xrightarrow[n → \infty ]{} x ∈ X
+ \end{rcases}
+ \implies \alpha _n x_n \xrightarrow[n → \infty ]{} \alpha x.
+ \]
+
+ Sei dazu $z_n \coloneq x_n - x ∈ X$, $γ_n \coloneq \alpha _n - \alpha ∈ \K$. Dann ist
+ \[
+ γ_n z_n + γ_n x + \alpha z_n = (\alpha _n - \alpha )(x_n-x) + (\alpha _n-\alpha ) x + \alpha (x_n-x)
+ = \alpha _n x_n - \alpha ×.
+ \]
+ Somit ist
+ \begin{align*}
+ d(\alpha _nx_n,\alpha x) &= d(\alpha nx_n - \alpha x,0) = d(γ_nz_n + γnx + \alpha z_n, 0) \\
+ &\le \underbrace{d(γ_nz_n,0)}_{→ 0} + \underbrace{d(γ_nx, 0)}_{→ 0} +
+\underbrace{d(\alpha z_n, 0)}_{→ 0} \xrightarrow{n → \infty } 0.
+ \end{align*}
+ Da die Addition ohnehin immer stetig ist, sind wir fertig.
+\end{proof}
+
+
+\begin{definition}
+ Eine Abbildung $|\cdot|: X → [0,\infty )$ heißt \emph{Quasi-Norm} auf dem Linearen
+ Raum $X$, falls gilt:
+ \begin{enumerate}[label=(Q\arabic*)]
+ \item
+ $|x| \ge 0$ für alle $x ∈ X$ und $|x| = 0$ genau dann, wenn $x = 0$.
+ \item
+ $|-x| = |x|$ für alle $x ∈ X$
+ \item
+ $|x+y| \le |x| + |y|$ für alle $x,y ∈ X$
+ \item
+ $|\alpha x_n| \xrightarrow[n → \infty ]{} 0$ für $\alpha ∈ \K$, falls $|x_n| → 0$
+ \item
+ $|\alpha _nx| \xrightarrow[n → \infty ]{} 0$ für $x ∈ X$, falls $|\alpha _n| → 0$
+ \item
+ $|\alpha _nx_n| \xrightarrow[n → \infty ]{} 0$ falls $|x_n| → 0$ und $|\alpha _nx_n| → 0$
+ \end{enumerate}
+ $(X,|\cdot|)$ heißt dann \emph{quasi-normierter} Raum.
+\end{definition}
+
+\begin{bemerkung}
+ Jeder normierte Raum ist auch ein quasi-normierter Raum.
+\end{bemerkung}
+
+\begin{satz}
+ \begin{enumerate}
+ \item
+ Ist $|\cdot|$ eine Quasi-Norm auf $X$, so wird durch $d(x,y) \coloneq |x-y|$ eine translationsinvariante Metrik definiert, welche $X$ zu einem metrischen linearen Raum macht.
+ \item
+ Ist $(X,d)$ ein metrischer linearer Raum mit translationsinvarianter Metrik $d$, so ist
+ $(X,|\cdot|)$ mit $|x| \coloneq d(x,0)$ ein quasi-normierter Raum.
+ \end{enumerate}
+\end{satz}
+\begin{proof}
+ Das folgt direkt aus den Axiomen und \cref{lemma-metrischer-linearer-raum-charak}.
+\end{proof}
+
+
+Speziell für die Anwendung sehr wichtige metrische lineare Räume werden von Semi-Normen erzeugt.
+
+\begin{definition}
+ Sei $X$ ein linearer Raum.
+ Eine Abbildung $p: X → ℝ$ heißt \emph{Semi-Norm} oder \emph{Halbnorm}, falls folgendes gilt:
+ \begin{enumerate}[label=(S\arabic*)]
+ \item
+ $∀x ∈ X: p(x) \ge 0$ (positiv)
+ \item
+ $∀ x ∈ X, \alpha ∈ \K: p(\alpha x) = |\alpha | p(x)$ (Homogenität)
+ \item
+ $∀ x, y ∈ X: p(x+y) \le p(x) + p(y)$ (Dreiecksungleichung)
+ \end{enumerate}
+ $(X,p)$ heißt dann \emph{semi-normierter} Raum.
+\end{definition}
+
+\begin{beispiel-nn}
+ $\L^p(\Omega)$ ist ein semi-normierter Raum.
+\end{beispiel-nn}
+
+\begin{bemerkung}
+ Jeder semi-normierte Raum $(X,p)$ erzeugt einen normierten Raum $(X/N,p)$, wobei $N = \{ x ∈ X: p(x) = 0\}$ ein linearer Unterraum ist.
+\end{bemerkung}
+
+\begin{satz}
+ \label{satz-abzaehlbares-prod-seminormierter-raeume}
+ Es seien $p_n: X → ℝ, n ∈ ℕ$ abzählbar viele Semi-Normen auf einem linearen Raum mit der Eigenschaft
+ \begin{equation}
+ p_n(x) = 0 \text{ für alle n ∈ ℕ } \implies x = 0. \label{eq:seminorm-folge-blub}
+ \end{equation}
+ Dann ist
+ \[
+ d(x,y) \coloneq \sum_{n = 1}^\infty 2^{-n} \frac{p_n(x-y)}{1+p_n(x-y)}
+ \]
+ eine translationsinvariante Metrik auf $X$, welche $X$ zum metrischen linearen Raum macht.
+\end{satz}
+
+\begin{bemerkung}
+ $p_n: X → ℝ$ sind auf $(X,d)$ stetig. Das folgt aus (für $x_i → x_0$ in $X$)
+ \[
+ |p_n(x_i) - p_n(x_0)| \le p_n(x_i-x_0) \xrightarrow{} 0
+ \]
+ und einer Übungsaufgabe.
+\end{bemerkung}
+
+\begin{satz}
+ \label{satz-umgebungsbasis-produkt-von-seminorm}
+ Sei $(X,d)$ der in \cref{satz-abzaehlbares-prod-seminormierter-raeume} gegebene metrische lineare Raum (mit der von der Metrik erzeugten Topologie).
+ Dann bilden die Mengen ($\epsilon _n > 0$)
+ \[
+ U (p_n,\epsilon _n) \coloneq \bigcup B^{p_n}_{\epsilon _n}(0)
+ = \{ x ∈ X: p_n(x) < \epsilon _n\}
+ \]
+ und deren endliche Durchschnitte eine Umgebungsbasis von $0 ∈ X$
+\end{satz}
+
+\begin{bemerkung-nn}
+ Nach dem Invarianzprinzip ist damit durch $\bigcup B^{p_n}_{\epsilon _n}$ die ganze Topologie bestimmt.
+ Mit anderen Worten: Die Topologie welche über die Metrik bestimmt ist, ist dieselbe wie die, welche von den
+ $U(p_n,\epsilon _n)$ und endlichen Schnitten davon erzeugt wird.
+\end{bemerkung-nn}
+\begin{proof}[\cref{satz-umgebungsbasis-produkt-von-seminorm}]
+ Zunächst ist $U (p_n,\epsilon _n) ∈ \T$:
+ Sei $n ∈ ℕ$ und $\epsilon _n > 0$ fest und $y ∈ U(p_n,\epsilon _n)$ beliebig gegeben.
+ Dann ist $p_n(y) < \epsilon _n$. Dann wähle $ρ = ρ(y) > 0$, so dass $p_n(y) + ρ < \epsilon _n$.
+ Dann gilt für $r \coloneq 2^{-n} \frac{ρ}{1+ρ} > 0$:
+ \[
+ x ∈ B_r(y) \implies p_n(x+r) < ρ.
+ \]
+ Dazu ist
+ \[
+ \frac{p_n(x-y)}{1+p_n(x-y)} \le 2^n \underbrace{d(x,y)}_{< r} < 2^n r = \frac{ρ}{1+ρ},
+ \]
+ also $p_n(x-y) < ρ$. Mit diesem $r$ gilt $B_r(y) ⊂ U(p_n,\epsilon _n)$:
+ Sei $x ∈ B_r(y)$. Dann gilt
+ \[
+ p_n(x) \le \underbrace{p_n(x-y)}_{< ρ} + p_n(y) < p_n(y) + ρ = \epsilon _n
+ \]
+ wie gewünscht.
+
+
+ Sei $ B_r(0), r > 0$ gegeben.
+ Wähle $n_0 ∈ ℕ$ mit
+ \[
+ \sum_{n=n_0}^\infty 2^{-n} < \frac r 2.
+ \]
+
+ mit $\epsilon \coloneq \frac r 2 $ gilt dann
+ \[
+ \bigcap_{n=1}^{n_0} U(p_(,\epsilon ) ⊂ B_r(0).
+ \]
+ Sei dazu $x ∈ \bigcap_{n=1}^{n_0} U(p_n,\epsilon )$ beliebig.
+ Dann ist
+ \[
+ d(x,0) \le \sum_{n=1}^{n_0} 2^{-n} \frac{p_n(x)}{1+p_n(x)} + \sum_{n=n_0}^\infty 2^{-n} < \epsilon \sum_{n=1}^{n_0} 2^{-n} + \frac r 2 < \epsilon + \frac r 2 = r,
+ \]
+ somit also $x ∈ B_r(0)$.
+\end{proof}
+
+\begin{bemerkung}
+ Die Mengen $U(p_n,\epsilon _n)$ und deren endlichen Schnitte sind konvexe Mengen, das heißt
+ \[
+ x, y ∈ U(p_n,\epsilon _n),\alpha ∈ [0,1] \implies \alpha x+(1-\alpha )y ∈ U(p_n,\epsilon _n)
+ \]
+\end{bemerkung}
+\begin{proof}
+ Es ist
+ \[
+ p_n(\alpha x + (1-\alpha )y) \le |\alpha | \underbrace{p_n(x)}_{< \epsilon _n} + |1-\alpha |\underbrace{p_n(y)}_{< \epsilon _n} = \epsilon _n.
+ \]
+\end{proof}
+
+Also besitzt der in \cref{satz-abzaehlbares-prod-seminormierter-raeume} gewonne metrische lineare Raum $(X,d)$ eine Umgebungsbasis von $0$, die nur aus konvexen elementen besteht.
+
+\begin{definition}
+ Ein topologischer linearer Raum $(X,\T)$, in dem jedes $x ∈ X$ eine Umgebungsbasis besitzt, die nur aus konvexen Mengen besteht, heißt \emph{lokalkonvex}.
+\end{definition}
+
+\begin{satz}
+ Sei $X$ ein linearer Raum mit Semi-Normen $p_i, i ∈ I$, wobei $I$ eine beliebige Indexmenge ist, mit der Eigenschaft
+ \[
+ p_i(x) = 0 \text { für alle } i ∈ I \implies x = 0.
+ \]
+ Dann sind die Mengen
+ \[
+ U(p_i,\epsilon _i) = \{ x ∈ X: p_{(x) < \epsilon _i}\}, \quad \epsilon _i > 0, i ∈ I
+ \]
+ und deren endliche Schnitte eine konvexe Umgebungsbasis von $0 ∈ X$.
+ Die dadurch gewonne Topologie $\T$ macht $X$ zu einem \emph{lokalkonvexen Hausdorff"=Raum}.
+\end{satz}
+
+\section{Beispiele}
+Wir werden die unten angegebenen Beispiele auch gleich auf Vollständigkeit untersuchen.
+
+\begin{definition}
+ \begin{enumerate}
+ \item
+ Ein metrischer linearer Raum $(X,d)$ der vollständig ist, heißt \emph{Fréchet"=Raum}.
+ \item
+ Ein normierter Raum $(X,\norm\cdot)$, der vollständig ist, heißt \emph{Banach"=Raum}.
+
+ \end{enumerate}
+\end{definition}
+
+\begin{beispiel-nn}[$\ell^p$-Räume]
+ \begin{enumerate}
+ \item
+ $(\ell^p,\norm\cdot_p)$, $1 \le p < \infty $ ist normierter Raum mit
+ \[
+ \norm x _p = \left( \sum_{i=1}^\infty |x_i|^p \right)^{1/p}.
+ \]
+ \item
+ $(\ell^\infty ,\norm\cdot_\infty)$, ist normierter Raum mit $\norm x _\infty = \sup_{i ∈ ℕ} |x_i|$.
+ \item
+ $(\ell^p,|\cdot|_p = \norm\cdot_p^p)$, $0 \le p < 1$ ist quasi-normierter Raum.
+ \end{enumerate}
+\end{beispiel-nn}
+
+\begin{bemerkung}
+ Für $0 < p < q \le \infty $ gilt $\ell^p ⊂ \ell^q ⊂ \ell^\infty $.
+\end{bemerkung}
+\begin{beweis}
+ Sei $x ∈ \ell^p$ mit $|x| = 1 = \sum_{i ∈ ℕ} |x_i|^p$.
+ Dann ist für alle $i ∈ ℕ$ $|x_i|^p \le 1$, also auch $|x_i| < 1$.
+ Dann folgt auch $\sum_{i ∈ ℕ} |x_i|^q < 1$, also $x ∈ \ell^q$ und $\sup_{i ∈ ℕ} |x_i| \le 1$, also $x ∈ \ell^\infty $.
+\end{beweis}
+
+
+\begin{satz}
+ Für $1 \le p \le \infty $ ist $(\ell^p,\norm\cdot_p)$ ein Banachraum.
+ Für $0 < p < \infty $ ist $(\ell^p,|\cdot|_p)$ ein Fréchet-Raum.
+\end{satz}
+\begin{proof}
+ Nur für $1 \le p < \infty $.
+ Sei dazu $(x_n)_{n ∈ ℕ} ⊂ \ell^p$ eine Cauchy-Folge, also
+ $x_n = (ξ_k^n)_{k ∈ ℕ}$ und für jedes $\epsilon > 0$ gibt es ein $n_0$ mit
+ \[
+ ∀n,m > n_0: \norm{x_n-x_m}_p = \left( \sum_{k=1}^\infty |ξ_k^n-ξ_k^m|^p \right)^{1/p} < \epsilon .
+ \]
+ Sei $k_0 ∈ ℕ$ beliebig. Dann ist $(ξ_k^n)_{n ∈ ℕ}$
+ eine Cauchy-Folge in $\K$, besitzt also einen Grenzwert $ξ_{k_0}$.
+ Setze nun $x \coloneq (ξ_k)_{k ∈ ℕ} ∈ \K^\infty = s$. Wir vermuten $x$ als Grenzwert unserer Cauchy-Folge.
+ Also müssen wir zeigen, dass $x ∈ \ell^p$, und dass unsere Folge tatsächlich gegen $x$ konvergiert.
+
+ Es gilt
+ \[
+ \norm{x_n}_! \le \underbrace{\norm{x_n-x_{n_0}}}_{< \epsilon } + \norm{x_{n_0}} \quad \forall n \ge n_0
+ \]
+ Deshalb existiert ein $M > 0$ mit $\norm{x_n}_p < M$ für alle $n ∈ ℕ$, also
+ \[
+ \sum_{k=1}^N |ξ_k^n|p < \sum_{k =1}^\infty |ξ_k^n|^p \le M^p < \infty .
+ \]
+ Also haben wir
+ \[
+ \sum_{k=1}^N |ξ_k^p| \le M^p \quad ∀ n ∈ ℕ,
+ \]
+ also durch Grenzwertbildung $N → \infty $ auch $\norm{x}_p^p \le M^p$ bzw. $x ∈ \ell^p$.
+
+
+ Ferner haben wir
+ \[
+ \sum_{k=1}^N |ξ_k^n-ξ_k^m|^p < \epsilon ^p \quad ∀ N ∈ ℕ, n, m \ge n_0(\epsilon ).
+ \]
+ Für $n → \infty $ folgt
+ \[
+ \sum_{k=1}^N |ξ_k-ξ_k^m|^p < \epsilon ^p \quad ∀N ∈ ℕ, m \ge n_0,
+ \]
+ und mit $N → \infty $
+ \[
+ \sum_{k=1}^\infty |ξ_k-ξ_k^m|^p < \epsilon ^p \quad ∀m \ge n_0,
+ \]
+ also die Konvergenz.
+\end{proof}
+\begin{beispiel-nn}
+ Betrachte den Folgenraum $S = \K^\infty = \{x = (ξ_n)_{n ∈ ℕ}, ξ_n ∈ \K\}$.
+ Dann ist
+ \[
+ p_n(x) \coloneq |ξ_n|, \quad p_n: \K^\infty → ℝ
+ \]
+ eine abzählbare Familie von Halbnormen mit
+ \[
+ p_n(x) = 0 ∀n ∈ ℕ \implies x = 0 ∈ \K^\infty
+ \]
+ Nach \cref{satz-abzaehlbares-prod-seminormierter-raeume} folgt, dass $(\K^\infty , d)$ mit
+ \[
+ d(x,y) \coloneq \sum_{n ∈ ℕ} 2^{-n} \frac{p_n(x-y)}{1+p_n(x-y)}
+ \]
+ ein metrischer linearer Raum ist.
+ Der Konvergenzbegriff entspricht gerade der komponentenweisen Konvergenz, das heißt, für eine Folge $(x_k)_{k ∈ ℕ}$ mit $x_k = (ξ^k_n)_{n ∈ ℕ}$ gilt
+ \begin{align*}
+ x_k \xrightarrow[k→\infty ]{} 0
+ &\gdw d(x_n,0) \xrightarrow[k→\infty ]{} 0 \\
+ &\gdw p_n(x_k) \xrightarrow[k→\infty ]{} ∀ n ∈ ℕ \\
+ &\gdw |ξ_n^k| \xrightarrow[k→\infty ]{} 0 ∀ n ∈ ℕ.
+ \end{align*}
+
+ Wir fragen uns nun, ob auf dem $\K^\infty $ auch eine Topologie existiert, so dass der induzierte Konvergenzbegriff der der gleichmäßigen Konvergenz in allen Komponenten entspricht.
+ Also
+ \[
+ x_k \xrightarrow[k → \infty ]{\text{glm}} 0 ∈ \K^\infty \gdw ∀\epsilon > 0 ∃ k_0 ∈ ℕ: |ξ_n^k| < \epsilon ∀ k \ge k_0 ∀n ∈ ℕ.
+ \]
+ Wenn $\K^\infty $ ein topologischer linearer Raum sein soll, ist das nicht möglich. Notwendig wäre, dass für eine Folge $x  ∈ \K^\infty $
+ \[
+ \alpha _k \xrightarrow[k → \infty ]{} 0 \text{ in } \K \implies \alpha _k x \xrightarrow[k→\infty ]{} \text{ in } X = \K^\infty .
+ \]
+ Wähle dazu die Nullfolge $(\alpha _k)_{k ∈ ℕ} = (1/k)_{k ∈ ℕ} ⊂ ℝ$, $x= (n)_{n ∈ ℕ} ⊂ X$. Dann ist
+ \[
+ \alpha _k x = (n/k)_{n ∈ ℕ} ∈ \K^\infty
+ \]
+ zwar eine Nullfolge in $\K^\infty$ ist, diese Konvergenz ist jedoch nicht gleichmäßig in $n$.
+ Man kann zeigen, dass $\K^\infty $ mit $d$ vollständig, also ein Fréchet-Raum, ist.
+ Ist $\K^\infty $ auch normierbar?
+ Also gibt es auf $\K^\infty $ eine Norm, welche die gleiche Topologie erzeugt wie die $d$?
+ Auch das ist nicht möglich:
+\end{beispiel-nn}
+\begin{lemma}
+ \label{lemma-s-metrikkugeln-enthalten-unterraeme}
+ In $(\K^\infty ,d)$ gilt:
+ \begin{enumerate}
+ \item
+ $B_1(0) = \K^\infty $
+ \item
+ Betrachte den linearen Unterraum $M_{n_0} \coloneq \{ x = (ξ_n)_{n ∈ ℕ}$ mit $ξ_n = 0$ für $n = 1,…,n_0 \}$.
+ Dann gibt es für jeden Radius $r > 0$ ein $n_0 ∈ ℕ$, so dass $M_{n_0} ⊂ B_{r}(0)$.
+ Das heißt, jede noch so kleine Metrikkugel enthält einen nichttrivialen Unterraum.
+ \end{enumerate}
+\end{lemma}
+\begin{proof}
+ \begin{enumerate}
+ \item
+ Das ist trivial.
+ \item
+ Sei $r > 0$ gegeben.
+ Wähle nun $n_0$, so dass $\sum_{n=n_0+1}^\infty 2^{-n} < r$.
+ Dann gilt
+ \[
+ ∀ x ∈ M_{n_0}: d(x,0) =
+ \sum_{n ∈ ℕ} 2^{-n} \frac{p_n(x)}{1+p_n(x)} =
+ \sum_{n=n_0}^\infty 2^{-n} \frac{p_n(x)}{1+p_n(x)} \le
+ \sum_{n=n_0}^\infty 2^{-n} < r.
+ \]
+ \end{enumerate}
+\end{proof}
+
+Wäre nun die Topologie auf $(\K^\infty ,d)$ nun auch von einer Norm erzeugt, dann wären die Normkugeln
+\[
+ B_r^{\norm\cdot}(0) = \{ x ∈ \K^\infty : \norm x < \tilde r \}
+\]
+auch eine Umgebungsbasis der Null.
+Das heißt insbesondere würden wir zu jedem $\tilde r$ ein $r$ finden, so dass $0 ∈ B_r^d(0) ⊂ B_r^{\norm\cdot} (0)$.
+Mit \cref{lemma-s-metrikkugeln-enthalten-unterraeme} folgt also
+\[
+ M_{n_0} ⊂ B_r(0) ⊂ B_r^{\norm\cdot}(0)
+\]
+für ein geeignetes $n_0$.
+Sei nun ein $0 \ne x ∈ M_{n_0}$. Dann ist, da $M_{n_0}$ ein Unterraum ist, auch $\alpha x ∈ M_{n_0}$ für alle $\alpha ∈ \K$.
+Das heißt,
+\[
+ |\alpha | \cdot \norm x = \norm{\alpha x} < \tilde r \text{ für alle } \alpha ∈ \K,
+\]
+was bereits $\alpha = 0$ impliziert. Das ist ein Widerspruch.
+
+
+\begin{beispiel-nn}[Räume beschränkter Funktionen]
+ Sei $S$ eine beliebige Menge und $B(S) \coloneq \{ f: S → \K, f(s)$ ist beschränkt $\}$.
+ Dann wird $B(S)$ mit
+ \[
+ \norm f _{B(S)} \coloneq \sup_{x ∈ S} |f(x)| < \infty ,
+ \]
+ der $\sup$-Norm, zu einem Banachraum.
+ Dabei ist offensichtlich, dass $\norm\cdot_{B(S)}$ tatsächlich eine Norm ist, und wir werden in einer Übung zeigen, dass die induzierte Metrik tatsächlich vollständig ist.
+\end{beispiel-nn}
+
+\begin{lemma-nn}
+ \label{lemma-vollst-ubertragt-abgeschl-teilmengen}
+ Sei $(X,d)$ ein metrischer Raum, $Y ⊂ X$. Es gilt
+ \begin{enumerate}
+ \item
+ Wenn $(X,d)$ vollständig ist und $Y$ abgeschlossen, dann ist auch $(Y,d|_{Y×Y}$ vollständig.
+ \item
+ Wenn $(Y,d|_{Y×Y}$ vollständig ist, so ist $Y$ abgeschlossen in $(X,d)$.
+ \end{enumerate}
+\end{lemma-nn}
+\begin{proof}
+ Übungsaufgabe.
+\end{proof}
+
+
+\begin{beispiel-nn}[Räume stetiger Funktionen]
+ Sei $K ⊂ ℝ^n$ kompakt, also nach Heine-Borel abgeschlossen und beschränkt.
+ Dann ist
+ \[
+ C(k) = \{ f: K → \K, f \text{ stetig} \}
+ \]
+ ein normierter Raum mit
+ \[
+ \norm{f}_{C(K)} = \norm{f}_{\infty } = \max_{t ∈ K} |f(t)|,
+ \]
+ der Maximumsnorm.
+ Dieses Maximum wird tatsächlich immer angenommen, da $K$ kompakt ist (Satz von Minimum und Maximum).
+ Insbesondere sind alle stetigen Funktionen auf $K$ beschränkt. Damit gilt offensichtlich $C(K) ⊂ B(K)$ und $\norm{f}_{C(K)} = \norm{f}_{B(K)}$ für alle $f ∈ C(K)$.
+ Da jede stetige Funktion auf kompakten Teilmengen von metrischen Räumen auch gleichmäßig stetig ist, das heißt
+ \[
+ ∀ \epsilon > 0 ∃ \delta > 0: \left( |t_1-t_2| < \delta \implies |f(t_1)-f(t_2)| < \epsilon \right) ∀ t_1,t_2 ∈ K
+ \]
+\end{beispiel-nn}
+
+\begin{lemma}
+ $C(K)$ ist ein abgeschlossener Unterraum von $(B(K), \norm\cdot_{B(K)})$ und somit insbesondere auch (mit \cref{lemma-vollst-ubertragt-abgeschl-teilmengen}) vollständig.
+\end{lemma}
+\begin{proof}
+ Sei $(f_i)_{i ∈ ℕ}$ eine konvergente (in $(B(K),\norm\cdot_{B(K)})$) Folge in $C(K)$.
+ Dann existiert ein $f ∈ B(K)$ mit $f_i \xrightarrow[i → \infty ]{\norm{\cdot}_{B(K)}} f$.
+ Wir müssen zeigen, dass $f$ bereits stetig ist.
+ Für beliebige $t₁, t_2 ∈ K$ gilt
+ \[
+ |f(t_1)-f(t_2) | \le \underbrace{|f_i(t_1)-f_i(t_2)|}_{< \epsilon /3 \text{ für } |t_1-t_2| < \delta ^{(i)}(\epsilon )} + 2 \underbrace{\norm{f_i - f}_{B(K)}}_{< \epsilon /3 \text{ für } i > i_0} < \epsilon .
+ \]
+ Damit ist $f$ auch gleichmäßig stetig, also insbesondere auch stetig und in $C(K)$.
+\end{proof}
+Das heißt, die Stetigkeit der Folgenglieder $(f_i)_{i ∈ ℕ} ⊂ C(K)$ überträgt sich auf die Grenzfunktion und Konvergenz in $(C(K),\norm\cdot_{\infty })$ ist „gleichmäßig auf $K$“.
+Wegen dieser Eigenschaft ist die Maximumsnorm $\norm\cdot_\infty $ auch die natürliche Norm auf $C(K)$.
+Andere mögliche Normen (und damit andere Topologien) auf $C(K)$ wären z.B.
+\[
+ \norm{f}_p = \left( \int_K |f(t)|^p dt \right)^{1/p}, \quad 1 \le p < \infty .
+\]
+Allerdings ist die Konvergenz in dieser Topologie impliziert keine Stetigkeit für die Grenzfunktion.
+
+
+\begin{beispiel-nn}
+ Sei $\Omega ⊂ ℝ^n$ offen und analog
+ \[
+ C(\Omega) \coloneq \{ f: \Omega → \K, f \text { stetig }\}.
+ \]
+ Hier können Funktionen aber auch unbeschränkt sein. Also braucht $\sup |f|$ nicht mehr zu existieren.
+\end{beispiel-nn}
+
+\begin{definition}
+ Es sei $(K_m)_{m ∈ ℕ}$ eine \emph{Ausschöpfung} von $\Omega$ mit kompakten Mengen $K_= ⊂ \Omega$, das heißt, es gelte
+ \[
+ \begin{cases}
+ \Omega = \bigcup_{m ∈ ℕ} K_m, \quad K_m ⊂ K_{m+1}, \\
+ K ⊂ \Omega \text { kompakt } \implies K ⊂ K_m \text { f ür ein } m ∈ ℕ
+ \end{cases}
+ \]
+\end{definition}
+Man nehme z.B.
+\[
+ K_m = \{ x ∈ \Omega ⊂ ℝ^n: \norm{x} \le m, \operatorname{dist}(x,∂\Omega) \ge 1/m\},
+\]
+wobei $\operatorname{dist}(x,∂\Omega) \coloneq \inf\{ \norm{x-y}: y ∈ ∂\Omega\}$ und $∂M = \cl \Omega \setminus \Omega$.
+
+Dann ist $C(\Omega)$ mit der Metrik
+\[
+ d(f,0) = \sum_{m ∈ ℕ} 2^{-m} \frac{\norm{f}_{C(K_m)}}{1+\norm{f}_{C(K_m)}}
+\]
+ein Fréchetraum, also ein metrisierbarer linearer Raum nach \cref{satz-abzaehlbares-prod-seminormierter-raeume}, da
+\[
+ \norm{f}_{C(K_m)} = 0 ∀ m ∈ ℕ \implies f = 0 ∈ C(\Omega).
+\]
+
+Es gilt in diesem Raum
+\[
+ d(f_i,f) \xrightarrow[i → \infty ]{} 0 \gdw
+ \norm{f_i-f}_{C(K_m)} \xrightarrow[i → \infty ]{} ∀m ∈ ℕ,
+\]
+was ja gerade gleichmäßige Konvergenz auf jeder Kompakten Menge $K ⊂ \Omega$ bedeutet.
+Damit ist Stetigkeit der Folgenglieder $(f_i)_{i ∈ ℕ} ⊂ C(\Omega)$ impliziert Stetigkeit der Grenzfunktion $f ∈ C(\Omega)$, da Stetigkeit nur eine lokale Eigenschaft ist.
+
+Wir werden in der Übung sehen, dass $C(\Omega)$ mit dieser Metrik $d_{C(\Omega)}$ nicht normierbar ist.
+
+\begin{beispiel-nn}[Räume differenzierbarer Funktionen]
+ \begin{enumerate}
+ \item
+ Betrachte die Menge $C^\ell(K) = \{ f: K → ℝ, D^\alpha f$ existiert und ist stetig für$|\alpha | < \ell \}$ der $\ell$-mal stetig differenzierbaren Funtktionen auf einer kompakten Menge $K ⊂ ℝ^n$ mit $\ell ∈ ℕ_0$
+ Dabei ist $\alpha = (\alpha _1,…,\alpha _n) ∈ ℕ_0^n$ ein Multiindex, $|\alpha | = \sum_{i=1}^n \alpha _i$ und
+ \[
+ D^\alpha f = \frac{∂^{|\alpha |} f}{∂x_1^{\alpha _1}\cdots∂x_n^{\alpha _n}}.
+ \]
+ Dann wird $C^\ell(K)$ mit der Norm
+ \[
+ \norm{f}_{C^\ell(K)} = \max_{|\alpha | \le l} \max_{x ∈ K} | D^\alpha f(x)|
+ \]
+ zu einem Banachraum. Die meisten Eigenschaften sind klar, die Vollständigkeit folgt unmittelbar aus der Vollständigkeit von $C(K)$
+ Konvergenz in $C^\ell(K)$ bedeutet gerade gleichmäßige Konvergenz aller partiellen Ableitungen bis zur Ordnung $\ell$ auf ganz $K$.
+ \item
+ Sei $\Omega ⊂ ℝ^n$ offen und
+ $\C^\ell(\Omega) = \{ f: \Omega → ℝ, D^\alpha f$ existiert und ist stetig für$|\alpha | < \ell \}$
+ der Raum der $\ell$-mal stetig differenzierbaren Funtktionen auf $\Omega$ mit $\ell ∈ ℕ_0$.
+ $C^\ell(\Omega)$ wird mit der Metrik
+ \[
+ d(f,g) \coloneq \sum_{m ∈ ℕ} 2^{-m} \frac{p_{m,l}(f-g)}{1+p_{m,l}(f-g)}, \quad p_{m,l}(f) = \max_{|\alpha | \le \ell} \norm{D^\alpha f}_{C(K_m)},
+ \]
+ wobei die $K_m$ Ausschöpfungen von $\Omega$ mit kompakten Mengen sind, zu einem Fréchetraum.
+ Konvergenz in $C^\ell(\Omega)$ bedeutet gerade gleichmäßige Konvergenz aller partiellen Ableitungen bis zur Ordnung $\ell$ auf jedem Kompaktum, das in $\Omega$ enthalten ist.
+ Auch dieser Raum ist nicht normierbar mit einem analogem Argument wie bei den stetigen Funktionen auf $\Omega$.
+ \item
+ Wir betrachten nun einige Unterräume von $\C^\ell(\Omega)$:
+ \begin{enumerate}[label=(\roman*)]
+ \item
+ $\C^\ell_B(\Omega) = \{ f: \Omega → ℝ, D^\alpha f$ existiert, ist beschränkt und ist stetig für$|\alpha | < \ell \}$
+ wird zum normierten Raum mit
+ \[
+ \norm{f}_{C^\ell_B(\Omega)} = \max_{|\alpha | \le l} \sup_{x ∈ \Omega} | D^\alpha f(x)|
+ \]
+ Zwar gilt $C^\ell_B(\Omega) ⊂ C^\ell(\Omega)$ (als Mengen), jedoch besitzt $C^\ell_B(\Omega)$ nicht die Relativtopologie von $\C^\ell(\Omega)$, wie wir in einer Übung sehen werden.
+ \begin{definition}
+ \begin{enumerate}
+ \item
+ Für $\Omega ⊂ ℝ^n$ offen und $f: \Omega → ℝ$ heißt
+ \[
+ \supp f \coloneq \cl{ \{ x ∈ \Omega, f(x) \ne 0 \}}
+ \]
+ der \emph{Träger} oder \emph{Support} von $f$.
+ \item
+ Wir sagen für eine Menge $M ⊂ \Omega$ \emph{$M$ liegt kompakt in $\Omega$}, wenn $\cl M $ kompakt ist und $\cl M ⊂ \Omega$. Wir schreiben dafür $M ⊂⊂ \Omega$.
+ \end{enumerate}
+ \end{definition}
+ \item
+ $C_0^\ell(\Omega) = \{ f ∈ C^\ell(\Omega) : \supp f ⊂⊂ M \}$
+ Funktionen mit $\supp f ⊂⊂ M $ haben Luft zum Rand von $\Omega$:
+ \[
+ \operatorname{dist}(\supp(f), ∂\Omega) > 0,
+ \]
+ denn sowohl $\supp f$ als auch $∂\Omega$ sind abgeschlossen.
+ Es gibt verschiedene Möglichkeiten, Topologien für $C_0^\ell(\Omega)$ zu wählen:
+ \begin{enumerate}
+ \item
+ $C_0^\ell(\Omega) ⊂ C^\ell(M)$ mit Spurtopologie von der Metrik.
+ \item
+ $C_0^\ell(\Omega) ⊂ C_B^\ell(M)$ mit Spurtopologie von der Norm.
+ \end{enumerate}
+ Diese Topologien sind jedoch nicht identisch.
+ \end{enumerate}
+ \item
+ Sei $\Omega ⊂ ℝ^n$ offen und
+ $C^\infty (\Omega) = \{ f: \Omega → ℝ, D^\alpha f $ existiert und ist stetig für alle $\alpha ∈ ℕ_0^n \} = \bigcap_{\ell ∈ ℕ}C^\ell(\Omega)$.
+ Wir definieren die Topologie wieder über eine Metrik durch Seminormen
+ \[
+ d(f,g) \coloneq \sum_{m ∈ ℕ} 2^{-m} \frac{p_{m}(f-g)}{1+p_{m}(f-g)}, \quad p_{m}(f) = \max_{|\alpha | \le m} \norm{D^\alpha f}_{C(K_m)}.
+ \]
+ Mit dieser Metrik wird $C^\ell(\Omega)$ zum Fréchetraum.
+ Konvergenz in $C^\infty (\Omega)$ bedeutet gerade gleichmäßige Konvergenz aller partiellen Ableitungen auf jedem Kompaktum, das in $\Omega$ enthalten ist.
+ Auch dieser Raum ist nicht normierbar mit einem analogem Argument wie bei den stetigen Funktionen auf $\Omega$.
+ \item
+ Sei $\Omega ⊂ ℝ^n$ offen und $C_0^\infty (\Omega) = \{ f ∈ C^\infty (\Omega) : \supp f ⊂⊂ M \}$ der \emph{Raum der Testfunktionen}.
+ Ein Beispiel für so eine Funktion ist
+ \[
+ f(x) =
+ \begin{cases}
+ c \exp \left( - \frac{1}{{1-|x|^2}} \right), & |x| < 1 \\
+ 0, & |x| \ge 1
+ \end{cases},
+ \]
+ wobei $\Omega = B^{\norm\cdot_\infty}_2(0)$, $|\cdot| = \norm\cdot_2$ und $c ∈ ℝ$ konstant.
+ Offensichtlich ist $C_0^\infty (\Omega) ⊂ C^\infty (\Omega)$.
+ Wenn man auf $C_0^\infty (\Omega)$ jedoch die Spurtopologie wählt, bekommt man später Probleme (bestimmte Funktionale auf $C_0^\infty (\Omega)$ sind nicht mehr stetig, wie wir in einer Übungsaufgabe sehen werden.
+ Man nennt Funktionale auf $C_0^\infty (\Omega)$ auch Distributionen).
+ Außerdem wäre der $C_0^\infty (\Omega)$ mit dieser Metrik nicht vollständig -- der Träger der Grenzfunktion muss nicht mehr beschränkt sein.
+ \begin{definition-nn}
+ Sei $M ⊂ X$ und $X$ ein linearer Raum. Dann heißt
+ \[
+ \conv (M) \coloneq \{ x: ∃\alpha _i > 0, x_i ∈ M, i ∈ \{1,…,k\}: \sum_{i=1}^k \alpha _i = 1, \sum_{i=1}^k \alpha _i x_i = x \}
+ \]
+ die \emph{konvexe Hülle} von $M$.
+ \end{definition-nn}
+ Aus Gründen, die erst später zu verstehen sind, wählt man auf $C^\infty _0(\Omega)$ folgende lokalkonvxe Topologie:
+ Setze
+ \[
+ p(\xi) \coloneq \sum_{k ∈ ℕ} 2^{-k} \frac{\norm \xi _{C^k(\Omega)}}{1 + \norm \xi _{C^k(\Omega)}}, \quad \xi ∈ C_0^\infty (\Omega)
+ \]
+ Sei $(D_j)_{j ∈ ℕ}$ eine Ausschöpfung von $\Omega$ mit offenen Mengen, also $D_j ⊂ D_{j+1}, D_j ⊂⊂ \Omega, \bigcup_{j ∈ ℕ} D_j = \Omega$.
+ Eine mögliche Wahl wäre beispielsweise $D_j = K_j^\circ$, wobei die $K_j$ wie oben sind.
+ Für $\epsilon = (\epsilon _j)_{j ∈ ℕ} ∈ ℝ^\infty , \epsilon _j > 0$ für alle $ℕ$ definieren wir eine Umgebungsbasis der $0 ∈ C_0^\infty (\Omega)$ durch alle Mengen
+ \[
+ U_\epsilon \coloneq \conv \left[ \bigcup_{j ∈ ℕ} \{ \xi ∈ C^\infty _0 : p(\xi) < \epsilon _j \} \right] ⊂ C_0^\infty (\Omega).
+ \]
+ mit $\epsilon = (\epsilon _j)_{j ∈ ℕ} ∈ ℝ^\infty , \epsilon _j > 0$ und endliche Schnitte davon. Andere Umgebungen umgeben sich durch Translation.
+ Die so definierte Topologie nennen wir $\T_\D$ und den Raum $C_1^\infty (\Omega)$ auch $\D(\Omega)$.
+ Es stellt sich heraus, dass diese Topologie tatsächlich unabhängig von der gewählten Ausschöpfung ist.
+ Außerdem ist $(\D(\Omega),\T_\D)$ ein topologischer linearer Raum, das heißt, die Vektorraumoperationen sind stetig.
+ \begin{lemma}[Charakterisierung offener Mengen in $\D(\Omega)$]
+ Es gilt
+ \[
+ O ∈ \T_\D \iff ∀ ξ ∈ O ∃ \epsilon =(e_j)_{j ∈ ℕ} ∈ ℝ^\infty , e_j > 0: e+U_\epsilon ⊂ O.
+ \]
+ Das heißt, die Topologie $\T_\D$ und die Topologie
+ \[
+ \tilde T_\D = \{ O ⊂ C_0^\infty (\Omega): ∀ ξ ∈ O ∃ \epsilon = (\epsilon _j)_{j ∈ ℕ} ∈ ℝ^\infty , \epsilon _j > 0: \epsilon + U_\epsilon ⊂ O \}
+ \]
+ sind gleich.
+ \end{lemma}
+ \begin{proof}
+ Übung.
+ \end{proof}
+ \begin{korollar}
+ Die Mengen $U_\epsilon$ sind bereits eine Umgebungsbasis der Null.
+ Nach Definition sind sie aber auch Konvex, das heißt $(\D(\Omega),\T_\D)$ ist ein lokalkonvexer Hausdorff-Raum.
+ \end{korollar}
+ \begin{satz}
+ $ξ_m \xrightarrow[m → \infty ]{} 0 \gdw$
+ \[
+ \begin{cases}
+ (i), & \text{Es existiert $D$ offen mit $D ⊂⊂ \Omega$ und
+ $ξ_m ∈ C_0^\infty (D)$ für alle $m ∈ ℕ$} \\
+ (ii), & \text{Für jedes $k ∈ ℕ$ gilt:
+ $\norm{ξ_m}_{C^k(\cl{\Omega})} \xrightarrow[m → \infty ]{} 0$}
+ \end{cases}
+ \]
+ \end{satz}
+ \begin{proof}
+ Zeige nur „$\Leftarrow$“. Sei dazu $(ξ_m)_{m ∈ ℕ}$ eine Folge mit (i) und (ii).
+ Wähle nun $D_j$ von oben mit $D ⊂ D_j$ ($j$ ist fest).
+ Sei nun $\epsilon =(\epsilon _i)_{i ∈ ℕ}, \epsilon _i > 0$ gegeben. Dann müssen wir zeigen, dass für alle $m > m_0$ schon $ξ_m ∈ U_\epsilon $ gilt.
+ Zunächst sind nach (i) $ξ_m ∈ C^\infty _0(D_j)$ .
+ Außerdem gilt
+ \[
+ p(\xi_m) \le \underbrace{\sum_{k=1}^N 2^{-k} \frac{ \norm{\xi_m}_{C^k(\cl \Omega)} } {1+ \norm{\xi_m}_{C^k(\cl \Omega)} }}_{\text{wegen (i)} < \epsilon _j/2 \text{ für $m \ge m_0(\epsilon _j,N)$}} + \underbrace{\sum_{k=N+1} 2^{-k}}_{<\epsilon _j/2 \text{ für $n$ groß genug}} < \epsilon _j.
+ \]
+ \end{proof}
+ \end{enumerate}
+ \end{beispiel-nn}
+ \begin{beispiel-nn}[Lebesgue-integrierbare Funktionen]
+ Betrachten wir nun Lebesgue-integrierbare Funktionen.
+ Bereits eingeführt wurden die Räume $\L^p(\Omega)$ und $L^p(\Omega)$, $0 < p < \infty $, wobei $\Omega ⊂ ℝ^n$ offen.
+ Diese sind für $1 \le p < \infty $ normiert, und für $0 < p < 1$ quasi-normiert.
+ Für $p = \infty $ setzen wir
+ \[
+ \L^\infty (\Omega) \coloneq \{ f: \Omega → ℝ ∪ \{ -\infty , \infty \}, f \text{ messbar und fast überall beschränkt} \}.
+ \]
+ Damit haben wir offenbar
+ \[
+ C(\Omega) ∩ B(\Omega) ⊂ \L^\infty (\omega).
+ \]
+ Sei
+ \[
+ \norm f _{\L^\infty (\Omega)} \coloneq \supess_{t ∈ \Omega} |f(t)| \coloneq \inf_{M ⊂ \Omega \text{ NM}} \sup_{t ∈ \Omega \setminus M} |f(t)|.
+ \]
+ Dann gilt für $f ∈ \L^\infty (\Omega)$
+ \[
+ \norm f = 0 \gdw f = 0 \text{ fast überall}
+ \]
+ Mit $N \coloneq \{ f ∈ \L^\infty (\Omega) : \norm f = 0 \}$ wird
+ \[
+ L^\infty (\Omega) \coloneq \left( \L^\infty (\Omega)/N, \norm\cdot_{L^\infty (\Omega)} \right)
+ \]
+ zu einem normiertem Raum.
+\end{beispiel-nn}
+
+\begin{bemerkung-nn}
+ \begin{enumerate}
+ \item
+ Es gilt die \emph{Hölder'sche Ungleichung}. Für $f ∈ L^p(\Omega)$, $g
+ ∈ L^q(\Omega)$,$\frac 1 p + \frac 1 q = 1$ ist
+ \[
+ \norm{f g }_{L^1(\Omega)} \le \norm{f}_{L^p(\Omega)} \norm g _{L^q(\Omega)}.
+ \]
+ \item
+ Für $\Omega$ messbar und beschränkt gilt
+ \[
+ L^∞(\Omega) ⊂ L^p(\Omega) ⊂ L^q(\Omega), \quad 0 < q < p \le ∞.
+ \]
+ \item
+ $C_0^∞(\Omega)$ ist nicht abgeschlossen in $(L^p(\Omega),\norm-_p)$.
+ Für $1 ≤ p < ∞$ gilt sogar
+ \[
+ \cl{C_0^∞(\Omega)}^{\norm-_{L^p(\Omega)}} = L^p(\Omega),
+ \]
+ das heißt, $C_0^∞(\Omega)$ liegt dicht in $L^p(\Omega)$.
+ \item
+ Satz von Riesz-Fischer: $(L^p(\Omega),\norm-_{L^p(\Omega)})$ ist für $1
+ ≤ p ≤ ∞$ ein Banach"=Raum.
+ \end{enumerate}
+\end{bemerkung-nn}
+
+\begin{lemma}
+ $L^p(0,1)$ ist für $0 < p < 1$ nicht lokalkonvex. Tatsächlich sind
+ $\emptyset$ und $L^p(0,1)$ die einzigen offenen konvexen Mengen.
+\end{lemma}
+
+\begin{beispiel}[Sobolev-Räume]
+ Wir kennen aus der Analysis bereits die partielle Integration: Für alle $f, h ∈ C^1(\cl \Omega)$, wobei $\Omega$ beschränkt und $∂M$ hinreichend glatt, gilt
+ \[
+ ∫_\Omega f(t) \cdot \left[ \frac ∂ {∂t_i} h(t) \right] \dd t = ∫_{∂\Omega} f(t) h(t) \nu_i \dd S(t) - ∫_\Omega \left[ \frac ∂ {∂t_i} f(t) \right] h(t) \dd t,
+ \]
+ wobei $\nu = (\nu_1, …, \nu_n)^T$ die äußere Einheitsnormale ist.
+
+ \begin{bemerkung-nn}
+ Ist $f$ oder $h ∈ \C_0^∞(\Omega)$, so verschwinden die Randterme.
+ \end{bemerkung-nn}
+
+ \begin{definition}\label{03-definiton-schwache-ableitung}
+ Sei $f ∈ L^p(\Omega)$. Dann heißt $g ∈ L^p(\Omega)$ \emph{verallgemeinerte Ableitung} oder \emph{schwache Ableitung} von $f$ nach $t_i$ für ein $i ∈ \{1,…,n\}$, falls für alle $\phi ∈ C_0^∞(\Omega)$ gilt:
+ \[
+ ∫_\Omega f(t) \frac {∂\phi}{∂t_i}(t) \dd t = - ∫_\Omega g(t) \phi(t) \dd t.
+ \]
+ \end{definition}
+
+ \begin{lemma}
+ Verallgemeinerte Ableitungen sind eindeutig bestimmt.
+ \end{lemma}
+
+ \begin{bemerkung-nn}
+ \begin{enumerate}
+ \item Wir schreiben dafür $d_{t_i} f \coloneq g$.
+ \item
+ Für beschränktes $\Omega$ und $f ∈ C^1(\cl \Omega)$ ist \cref{03-definiton-schwache-ableitung} mit der klassischen Ableitung $g := D_{t_i} f ∈ C^0(\cl \Omega) ⊂ L^p(\cl \Omega)$ erfüllt.
+ Also haben wir in dieser Situation
+ \[
+ d_{t_i} f = D_{t_i} f.
+ \]
+ Also stimmen die klassische und die schwache Ableitung überein.
+ \item
+ Induktiv kann man höhere (schwache) Ableitungen definieren.
+ \end{enumerate}
+ \end{bemerkung-nn}
+
+ \begin{definition}
+ Sei $k ∈ N_0, 1 ≤ p < ∞$. Dann ist der \emph{Sobolev"=Raum} $W^{k,p}(Ω) := \{ f ∈ L^p(Ω): f$ besitzt verallgemeinerte Ableitungen $d^αf ∈ L^p(Ω)$ für alle $α ∈ ℕ_0^n$ mit $0 ≤ |α| ≤ k \}$.
+ \end{definition}
+
+ \begin{lemma}[Leibniz'sche Regel]
+ Sei $1 ≤ p < ∞$ und $Ω ⊂ ℝ^n$ offen. Dann gilt für alle $f ∈ W^{k,p}(Ω)$ und für alle $α ∈ ℕ_0^n$ mit $0 ≤ |α| ≤ k$:
+ \[
+ ∀\phi ∈ C_0^∞(Ω): ∫_Ω d^α f(t) \phi(t) \dd t = (-1)^{|α|} ∫_Ω f(t) D^α \phi(t) \dd t
+ \]
+ \end{lemma}
+
+ \begin{bemerkung-nn}
+ Ist umgekehrt $(f^α)_{0 ≤ |α| <= k} ⊂ L^p(Ω)$ eine Familie von Funktionen, für die
+ \[
+ ∀\phi ∈ C_0^∞(Ω): ∫_Ω d^α f(t) \phi(t) \dd t = (-1)^{|α|} ∫_Ω f(t) D^α \phi(t) \dd t
+ \]
+ gilt, so ist $f^0 ∈ W^{k,p}(Ω)$ und $d^αf^0 = f^α$.
+ \end{bemerkung-nn}
+
+ \begin{satz}
+ $W^{k,p}(Ω)$ ist mit der Norm
+ \[
+ \norm{f}_{W^{k,p}(Ω)} := \left( \sum_{0 ≤ |α| ≤ k} \norm{d^α f}^p_{L^p(\Omega)}\right)^{1/p}
+ \]
+ ein Banachraum.
+ \end{satz}
+\end{beispiel}
+
+Seien $f_n → f ∈ L^p(\Omega), h ∈ C_0^\infty (\Omega)$.
+Dann
+\[
+ \lim_{n → \infty } ∫_\Omega f_n(t) h(t) \dd t = ∫_\Omega f(t) h(t) \dd t,
+\]
+denn
+\begin{align*}
+ ∫_\Omega (f_n(t) - f(t)) h(t) \dd t &\le ∫_{\supp h} M |f_n(t) - f(t)| \dd t
+\\ & \stackrel{\mathclap{\text{Hölder}}}{\le} \; M [ \supp(h)]^{1/q}
+\norm{f_n-f}_{L^p(\Omega)} → 0.
+\end{align*}
+
+\section{Beschränkte und kompakte Mengen in metrischen linearen Räumen}
+
+Wir wissen bereits nach dem Satz von Heine-Borel, dass eine Teilmenge $K ⊂ ℝ^n$ genau dann kompakt ist, wenn sie abgeschlossen und beschränkt ist.
+Beschränktheit bedeutet hier Beschränktheit in einer (beliebigen, da alle äquivalent) Norm.
+
+Nun wollen wir so ein Konzept für Beschränktheit auch in allgemeinen metrischen (topologischen) linearen Räumen finden.
+
+\begin{problem-nn}
+ Die natürliche Übertragung $d(x,0) \le M$, $x ∈ B$ definiert \emph{keine} Beschränktheit.
+ Gründe dafür sind:
+ \begin{enumerate}
+ \item
+ In einigen metrischen Räumen gilt ohnehin $d(x,0) \le 1$ für alle $x ∈ X$.
+ \item
+ Ist $d$ eine Metrik auf $X$. Dann ist $\tilde d \coloneq \frac d {1+d} \le 1$ eine zu $d$ äquivalente Metrik auf $X$, wie wir in Topologie gesehen haben.
+ \end{enumerate}
+\end{problem-nn}
+
+\begin{definition}
+ Sei $(X,\T)$ ein topologischer linearer Raum, $B ⊂ X$ heißt \emph{beschränkt}, falls zu jeder offenen Umgebung $U$ von $0 ∈ X$ ein $\alpha > 0$ existiert, so dass $B ⊂ \alpha U = \{\alpha u: u ∈ U\}$, das heißt jede Nullumgebung lässt sich so weit „aufblasen“, dass sie $B$ überdeckt.
+\end{definition}
+
+\begin{bemerkung-nn}
+ Der Begriff „Beschränktheit“ hängt also von der Topologie ab.
+\end{bemerkung-nn}
+
+
+\begin{satz}
+ Sei $(X,d)$ ein metrischer linearer Raum, dessen Metrik gemäß \cref{satz-abzaehlbares-prod-seminormierter-raeume} von abzählbar vielen Seminormen $(p_n)_{n ∈ ℕ}$ induziert ist.
+ Dann ist eine Teilmenge $B ⊂ X$ genau dann beschränkt, wenn für jedes $k ∈ ℕ$ ein $M_k > 0$ existiert mit $p_k(x) \le M_k$ für alle $x ∈ B$.
+\end{satz}
+\begin{proof}
+ „⇒“: Sei $k ∈ ℕ$ gegeben.
+ Setze $r_k \coloneq \frac 1 {2^{k+1}}$ und $U \coloneq B_{r_k}(0)$.
+ Da $B$ beschränkt ist, gibt es $\alpha = \alpha _k > 0$, dass
+ \begin{align*}
+ & B ⊂ \alpha U = \alpha B_{r_k}(0) \\
+ \gdw & \alpha ^{-1} B ⊂ B_{r_k} (0) \\
+ \gdw &d(\alpha ^{-1} x, 0) < r_k ∀ x ∈ B
+ \end{align*}
+ Dann gilt schon $p_k(x) \le M_k \coloneq \alpha _k$ für alle $x ∈ B$, denn
+ \[
+ \frac 1 {2^{k+1}} = r_k > d(\alpha _k^{-1} x, 0
+ \ge 2^k \frac {p_k(\alpha _k^{-1}x)}{1+p_k(\alpha _k^{-1} x)}
+ = 2^{-k} \frac{\alpha _k^{-1} p_k(x)}{1+\alpha _k^{-1} p_k(x)}.
+ \]
+ Also mit $\eta \coloneq \alpha _k^{-1} p_k(x)$ gilt $\frac 1 2 > \frac \eta {1+\eta}$, also $\eta < 1$ oder $p_k(x) \le M_k$ für alle $x ∈ B$.
+
+ „⇐“:
+ Sei also $p_k(x) \le M_k$ für alle $x ∈ B$ und $k ∈ ℕ$.
+ Wir müssen nun zeigen, dass es für jedes $r > 0$ ein $\alpha > 0$ gibt mit $B ⊂ \alpha B_r(0)$, also $\alpha ^{-1} B ⊂ B_r(0)$.
+ Sei also $r > 0$ gegeben.
+ Wähle nun $m_0 ∈ ℕ$ mit $\sum_{n=m_0+1}^\infty 2^{-n} < r/2$.
+ Wähle $\alpha > 0$ mit $\sum_{n=1}^{m_0} 2^{-n} \frac{\alpha ^{-1} M_k}{1+\alpha ^{-1} M_k} < r/2$.
+ Dann gilt für alle $x ∈ B$
+ \begin{align*}
+ d(\alpha ^{-1} x, 0) &=
+ \sum_{n ∈ ℕ} 2^{-n} \frac{\alpha ^{-1} p_n(x)}{1+\alpha ^{-1} p_n(x)} \\
+ &\le \sum_{n=1}^{m_0} 2^{-n} \frac{\alpha ^{-1} p_n(x)}{1+\alpha ^{-1} p_n(x)} + \sum_{n=m_0+1}^\infty 2^{-n} < r/2 + r/2 = r.
+ \end{align*}
+\end{proof}
+
+\begin{korollar}
+ Sei $(X,\norm\cdot)$ ein normierter linearer Raum,
+ Dann ist eine Teilmenge $B ⊂ X$ genau dann beschränkt, wenn $M > 0$ existiert mit $\norm{x} \le M$ für alle $x ∈ B$.
+\end{korollar}
+\begin{proof}
+ Wähle $p_1(x) = \norm x$ und $p_k \equiv 0$ für $k \ge 2$ und verwende den vorherigen Satz.
+\end{proof}
+
+
+\begin{bemerkung}
+ Kugeln $B_r(0)$ in $(X,d)$, wobei $d$ wie in \cref{satz-abzaehlbares-prod-seminormierter-raeume}, sinid also immer unbeschränkt,
+ weil nichttriviale Unterräume $M_{n_0} ⊂ B_r(x)$ existieren.
+ Insbesondere ist dies gültig in den Räumen $\K^\infty , C(\Omega), C^\ell(\Omega)$ und $C^\infty (\Omega)$.
+\end{bemerkung}
+
+\begin{definition}
+ Ein topologischer linearer Raum $(X,\T)$ heißt \emph{lokalbeschränkt}, falls $0 ∈ X$ eine beschränkte Umgebung besitzt.
+\end{definition}
+
+\begin{bemerkung}
+ Normierte Räume sind lokalbeschränkt und lokalkonvex. Es gilt aber auch die Umkehrung:
+\end{bemerkung}
+
+\begin{satz}[Kolmogorov]
+ Ein topologischer linearer Raum $(X, \T)$ ist genau dann normierbar, das heißt, die Topologie wird von einer Norm induziert,
+ wenn $(X,\T)$ lokalkonvex und lokalbeschränkt ist.
+\end{satz}
+
+\begin{beispiel-nn}
+ Die Räume $\K^\infty , C(\Omega), C^\ell(\Omega)$ und $C^\infty (\Omega)$ sind nicht lokalbeschränkt, aber lokalkonvex. Somit sind sie auch nicht normierbar.
+ Auch $L^p(0,1)$ mit $0 < p < 1$ ist nicht lokalkonvex, aber lokalbeschränkt, also nicht normierbar.
+\end{beispiel-nn}
+
+\begin{definition}
+ Sei $(X,\T)$ ein topologischer linearer Raum.
+ Eine Umgebung $U$ der Null heißt \emph{kreisförmig} oder \emph{balanced}, falls
+ \[
+ t U ⊂ U, \quad |t| < 1
+ \]
+\end{definition}
+
+\begin{lemma}
+ Sei $(X,\T)$ ein topologischer linearer Raum.
+ Dann besitzt die Null eine Umgebungsbasis aus kreisförmigen Mengen.
+\end{lemma}
+\begin{proof}
+ Übung.
+\end{proof}
+
+\begin{warnung-nn}
+ Metrikkugeln müssen im Allgemeinen nicht kreisförmig sein (obwohl die uns bekannten Kugeln dies sind).
+ Gegenbeispiel: $X = ℝ$, $d(x,y) \coloneq \left| ∫_x^y 1+\ind{ℝ_-}(s)\; ds \right|$.
+\end{warnung-nn}
+
+\begin{lemma}
+ Sei $(X,\T)$ ein topologischer linearer Raum und $V ∈ \T$ eine Umgebung der 0.
+ Dann gilt
+ \[
+ X = \bigcup_{n ∈ ℕ} n V.
+ \]
+\end{lemma}
+\begin{proof}
+ „$\supset$“: klar.
+
+ „⊂“: Sei $x ∈ X$. Setze $β_n = 1/n, n ∈ ℕ$. Dann gilt
+ \[
+ β_n x \xrightarrow[n → \infty ]{} 0,
+ \]
+ also $β_n ∈ V$ für $n \ge n_0$. Damit haben wir aber $x ∈ n_0 V$.
+\end{proof}
+
+\begin{satz}
+ Sei $(X,\T)$ ein topologischer linearer $T_2$-Raum und $K ⊂ X$ kompakt.
+ Dann ist $K$ abgeschlossen und beschränkt.
+\end{satz}
+\begin{definition-nn}
+ Falls auch die Umkehrung gilt, dann besitzt $X$ die \emph{Heine"=Borel"=Eigenschaft}.
+\end{definition-nn}
+\begin{warnung-nn}
+ Im Allgemeinen ist die Umkehrung falsch.
+\end{warnung-nn}
+\begin{proof}
+ Nach einer Übungsaufgabe ist $K$ bereits abgeschlossen.
+ Also müssen wir nur zeigen, dass $K$ auch beschränkt ist.
+ Sei $V ∈ \T$ eine Nullumgebung.
+ Sei $W ⊂ \T$ eine kreisförmige Umgebung der $0$, die ganz in $V$ enthalten ist.
+ Da
+ \[
+ K ⊂ X = \bigcup_{n ∈ ℕ} n W
+ \]
+ eine offene Überdeckung von $K$ ist, besitzt diese wegen $K$ kompakt eine endliche Teilüberdeckung
+ \[
+ K ⊂ \bigcup_{i=1}^s n_i W \stackrel{(*)}= n_s W, \quad n_1 < n_2 < … < n_s,
+ \]
+ also folgt die Behauptung mit $\alpha = n_s$. $(*)$ gilt wegen der Kreisförmigkeit und $\left|\frac {n_i}{n_s}\right| \le 1$.
+\end{proof}
+\begin{bemerkung-nn}
+ Ohne die Hausdorff-Eigenschaft gilt dies nicht. Gegenbeispiel: $X = ℝ$ mit der Klumpentopologie.
+ Dann ist jede Teilmenge von $ℝ$ kompakt, aber nur $\emptyset$ und $ℝ$ sind abgeschlossen.
+\end{bemerkung-nn}
+
+\begin{definition}
+ \begin{enumerate}
+ \item
+ In einem topologischen Raum $(X,\T)$ heißt eine Menge $A ⊂ X$ \emph{relativ kompakt}, falls $\cl A$ kompakt ist.
+ \item
+ In einem metrischen Raum $(X,d)$ heißt eine Menge $A ⊂ X$ \emph{präkompakt}, falls für jedes $\epsilon > 0$ die Menge $A$ von endlich vielen Bällen mit Radius $\epsilon $ überdeckt werden kann.
+ \end{enumerate}
+\end{definition}
+
+\begin{satz}
+ Sei $(X,d)$ ein metrischer Raum, $\emptyset \ne A ⊂ X$. Dann sind äquivalent:
+ \begin{enumerate}
+ \item
+ $A$ ist kompakt.
+ \item
+ $A$ ist folgenkompakt.
+ \item
+ $(A,d|_{A×A})$ ist vollständig und $A$ präkompakt.
+ \end{enumerate}
+\end{satz}
+\begin{proof}
+ $(a) \iff (b)$ wurde bereits gezeigt. Wir zeigen nur $(b) ⇒ (c)$:
+ Sei $(x_n)_{n ∈ ℕ} ⊂ A$ eine Cauchy-Folge. Nach (b) besitzt $(x_n)_{n ∈ ℕ}$ einen Häufungspunkt $x^*$.
+ Da $(x_n)_{n ∈ ℕ}$ Cauchy-Folge ist, konvergiert $(x_n)_{n ∈ ℕ}$ schon gegen $x^*$. Damit ist $A$ vollständig.
+
+ Angenommen, $A$ wäre nicht präkompakt. Dann gibt es $\epsilon > 0$, so dass $A$ keine endliche Überdeckung mit $\epsilon $-Kugeln besitzt.
+ Dadurch kann man eine Folge $(x_k)_{k ∈ K}$ definieren, mit $d(x_k,x_j) > \epsilon $ für $k \ne j$.
+ Dann besitzt $(x_k)_{k ∈ K}$ offensichtlich keine Cauchy-Teilfolge, also auch keinen Häufungspunkt.
+ Also $A$ präkompakt.
+\end{proof}
+
+\begin{korollar}
+ Ist $(X,d)$ ein vollständiger metrischer Raum und $A ⊂ X$, dann ist $A$ genau dann präkompakt, wenn $A$ relativ kompakt ist.
+\end{korollar}
+
+
+\begin{satz}[Ascoli-Arzela]
+ Sei $S ⊂ ℝ^n$ kompakt und $C(S,ℝ^m)$ mit der Norm
+ \[
+ \norm{f}_∞ \coloneq \max_{x ∈ S} |f(x)|_{ℝ^m}
+ \]
+ ausgestattet. Sei $A ⊂ C(S;ℝ^m)$. Dann sind äquivalent:
+ \begin{enumerate}
+ \item $A$ ist präkompakt.
+ \item
+ $A$ ist beschränkt und gleichgradig stetig, das heißt,
+ \[
+ \sup_{f ∈ A} |f(x)-f(y)|_{ℝ^m} \xrightarrow[|x-y|→ 0]{} 0.
+ \]
+ \end{enumerate}
+\end{satz}
+
+\begin{satz}[Fréchet, Kolmogorov]
+ Sei $1 ≤ p < ∞$. Dann ist $A ⊂ L^p(ℝ^n)$ genau dann präkompakt, wenn
+ \begin{enumerate}[label=(\roman*)]
+ \item
+ $A$ ist beschränkt.
+ \item
+ $A$ ist im Mittel gleichgradig stetig, das heißt
+ \[
+ \sup_{f ∈ A} \norm{f(\cdot + h) - f}_{L^p(ℝ^n)} \xrightarrow[|h| → 0]{} 0.
+ \]
+ \item
+ \[
+ \sup_{f ∈ A} \norm{f}_{L^p(ℝ^n \setminus B_R(0))} \xrightarrow[R → ∞]{} 0.
+ \]
+ \end{enumerate}
+\end{satz}
+
+\begin{bemerkung-nn}
+ Der Satz gilt auch für Teilmengen $Ω$ von $ℝ^n$ mit den offensichtlichen Anpassungen. Ist $Ω$ beschränkt, so wird (iii) überflüssig.
+\end{bemerkung-nn}
+
+
+\section{Stetige lineare Operatoren}
+
+\begin{satz}
+ 3.6.4.
+\end{satz}
+Hier gilt $M = \inf \{ c \ge 0:$ mit $C$ gilt (5) $\}$.
+\begin{proof}
+ $(1) \iff (2)$ schon gezeigt.
+
+ $(3) \iff (4)$ klar durch die Charakterisierung von beschränkten Mengen in
+ normierten Räumen und Ausnutzung der Linearität.
+
+ $(2) \Rightarrow (4)$. Sei $T$ stetig in $x^*$. Wähle $\epsilon > 0$, so dass $T(\cl B_\epsilon (x^*)) ⊂ B_1(T(x^*))$.
+ Dann gilt für alle $x ∈ \cl B _1 (0)$
+ \[
+ x^* + \epsilon x ∈ \cl B_\epsilon (x^*)
+ \]
+ und $T(x^*) + \epsilon T(x) = T(x^* + \epsilon x) ∈ B_1(T(x^*))$, das heißt $\epsilon T(x) ∈ B_1(0)$ oder $\norm{T(x)}_Y \le \frac 1 {\epsilon } =: M$
+
+ $(4) \Rightarrow (5)$. Für $x \ne 0$ gilt
+ \[
+ \norm{T(x)} \le \norm x \norm{T\left( \frac x {\norm x} \right)} \le M \norm x,
+ \]
+ also gilt die Aussage mit $C \coloneq M$.
+
+ $(5) \Rightarrow (1)$. Für $x, x_1 ∈ X$ gilt
+ \[
+ \norm{T(x) - T(x_1)} = \norm {T(x-x_1)} \le C \norm x-x_1 \xrightarrow[x → x_1]{} 0.
+ \]
+ Damit ist $T$ stetig in $x_1$.
+\end{proof}
+
+\begin{korollar}
+ Sei die Situation wie in 6.4 Ist $T$ zusätzlich bijektiv, so ist $T$ genau dann ein Homöomorphismus, wenn es Konstanten $m, M > 0$ gibt mit
+ \[
+ m \norm x \le \norm {T(x)} \le M \norm {x}
+ \]
+ für alle $x ∈ X$
+\end{korollar}
+\begin{beweis}
+ klar.
+\end{beweis}
+
+\begin{warnung-nn}
+ $T$ linear, bijektiv und stetig impliziert selbst in normierten Räumen noch nicht, dass auch die Inverse Abbildung $T^{-1}$ auch stetig ist, wie wir in der Übung sehen werden.
+ Sind $X$ und $Y$ aber Banachräume, so gilt dies aber (Satz von der offenen Abbildung).
+\end{warnung-nn}
+
+Nun zur Charakterisierung von Stetigkeit in metrischen linearen Räumen.
+
+\begin{satz}
+ Sei $T: X → Y$ linear, $X$, $Y$ lineare metrische Räume.
+ Dann ist $T$ genau dann stetig, wenn $T$ beschränkt ist.
+\end{satz}
+
+In topologischen linearen Räumen gilt dies jedoch nciht.
+
+\begin{satz}
+ 3.6.7
+\end{satz}
+\begin{proof}
+ Nur „$\Leftarrow$“: Nach 6.6 reicht es, Beschränktheit von $T$ zu zeigen, also dass, wenn $B ⊂ X$ beschränkt ist, auch $T(B) ⊂ Y$ beschränkt ist.
+ $B ⊂ X$ ist genau dann beschränkt, wenn für alle $k ∈ ℕ$ $C_k > 0$ existieren mit $p_k(x) \le C_k$ für alle $x ∈ B$.
+ Nach Voraussetzung ist dann aber auch für alle $x ∈ B$
+ \[
+ q_m(Tx) \le M_m(C_{n_1} + … + C_{n_k}) =: K_m,
+ \]
+ was nach 5.2 heißt, dass $T(B)$ beschränkt in $Y$ ist.
+\end{proof}
+
+\begin{definition}
+ Seien $X, Y$ topologische lineare Räume. Dann bezeichnet $\L(X, Y) \coloneq \{ T: X → Y: T$ linear und stetig $\}$ den \emph{Raum der stetigen (beschränkten) Operatoren}.
+ Im Spezialfall $Y = \K$ sei $X' \coloneq \L(X, \K)$ der \emph{Raum der stetigen Funktionale} oder auch der \emph{Dualraum von $X$}.
+\end{definition}
+\begin{bemerkung-nn}
+ \begin{enumerate}
+ \item
+ $\L(X,Y)$ ist wieder ein linearer Raum.
+ \item
+ Metrische lineare Räume haben Dualräume, die im Allgemeinen nicht mehr metrisierbar sind.
+ \item
+ $X' = \{ 0\}$ ist möglich, wie wir in der Übung sehen werden
+ \item
+ Ist $X$ jedoch normierbar, so folgt aus den Hahn-Banach-Sätzen, dass $X'$ nichttrivial ist.
+ \item
+ Falls $X$ und $Y$ normierte Räume sind, dann wird $\L(X, Y)$ ebenfalls zu einem normierten Raum mit der Operatornorm
+ \begin{align*}
+ \norm T &\coloneq \norm T _{\L(X,Y)} \coloneq \sup \{\norm x _X \le 1\} \norm {Tx}_Y \\ &= \inf \{ C \ge 0: ∀x ∈ X: \norm {Tx} \le C \norm x \}.
+ \end{align*}
+ Das heißt, wir haben
+ \begin{equation}
+ \label{eq:61}
+ ∀x ∈ X: \norm {Tx}_Y \le \norm T \norm x _X
+ \end{equation}
+ Also haben wir
+ \[
+ \norm{(T_1 + T_2)x} = \norm{T_1x + T_2x} \le \norm{T_1x} + \norm{T_2x} \le \left( \norm{T_1} + \norm{T_2} \right) \norm{x},
+ \]
+ und somit $T_1 + T_2 ∈ \L(X,Y)$ und $\norm{T_1 + T_2} \le \norm{T_1} + \norm{T_2}$ nach \eqref{eq:61}.
+ \item
+ Auf $\L(ℝ^n,ℝ^m)$ ergeben sich die bekannten Matrixnormen.
+ \end{enumerate}
+\end{bemerkung-nn}
+
+\begin{satz}
+ Seien $X, Y$ normierte Räume, $Y$ vollständig. Dann ist $\L(X,Y)$ ein Banachraum.
+ Insbesondere ist $X'$ immer ein Banachraum.
+
+ Sei $Z$ ebenfalls ein normierter Raum.
+ Ist $T ∈ \L(X,Y)$, $S ∈ \L(Y,Z)$, so ist $ST ∈ \L(X,Z)$ und $\norm{ST}_{\L(X,Z)} \le \norm S \norm T$.
+\end{satz}
+
+\begin{proof}
+ Es ist nur noch die Vollständigkeit zu zeigen.
+ Sei dazu $(T_n)_{n ∈ ℕ}$ eine Cauchy-Folge in $\L(X,Y)$.
+ Das heißt, für jedes $\epsilon > 0$ existiert ein $N_0$ mit $\norm {T_n - T_m} < \epsilon $ für $n, m > N_0$.
+ Also mit \eqref{eq:61} $\norm {T_n x - T_mx} \le \norm {T_n - T_m} \norm x < \epsilon \norm x$ für alle $x ∈ X$ und $n,m > N_0$.
+ Insbesondere ist $(T_nx)_{n ∈ ℕ}$ eine Cauchy-Folge in $Y$. Da $Y$ vollständig ist, besitzt diese Folge einen Grenzwert $y_x ∈ Y$.
+ Wir definieren eine Abbildung
+ \[
+ T: X → Y, x ↦ y_x.
+ \]
+ Dann ist $T$ linear, weil alle $T_n$ linear sind. Also ist nur die Stetigkeit von $T$ und die Konvergenz von $(T_n)_{n ∈ ℕ}$ gegen $T$ zu zeigen.
+ Für die Stetigkeit bekommt man unter Verwendung der Dreicksunglechung direkt
+ \[
+ \left| \norm {T_n} - \norm{T_m} \right| \le \norm {T_n - T_m} < \epsilon \quad ∀ n, m \ge N_0,
+ \]
+ also eine Cauchyfolge $\left( \norm{T_n} \right)_{n ∈ ℕ}$ in $ℝ$, die wegen der Vollständigkeit von $ℝ$ konvergent, also insbesondere auch beschränkt ist.
+ Damit gibt es $M > 0$ mit $\norm {T_n} \le M$ für alle $n ∈ ℕ$, also mit~\eqref{eq:61}
+ \[
+ \norm{Tx} \xleftarrow[n → \infty ]{} \norm{T_nx } \le M \norm x, ∀ x ∈ X,
+ \]
+ also die stetigkeit von $T$.
+ Jetzt zur Konvergenz:
+ Für $\norm x \le$ 1 gilt
+ \[
+ \norm {T_n x - T_m x } < \epsilon , \quad ∀n, m \ge N_0,
+ \]
+ also durch Grenzwertbildung $n → \infty $
+ \[
+ \norm {T_n x - T x } < \epsilon , \quad ∀n \ge N_0,
+ \]
+ und mit~\eqref{eq:61}
+ \[
+ \norm {T_n -T} = \sup_{\norm x \le 1} \norm {T_n x - T_x} < \epsilon , \quad ∀ n \ge N_0,
+ \]
+ das heißt $T_n → T$ wie gewünscht.
+
+ Für den Zusatz haben wir
+ \[
+ \norm {S(Tx)} \le \norm S \norm {Tx} \le \norm S \norm T \norm x.
+ \]
+ Da das für alle $x ∈ X$ gilt, haben wir $\norm {ST} \le \norm S \norm T$.
+\end{proof}
+
+
+\begin{korollar}
+ Ist $X$ ein Banachraum, dann ist $\L(X) \coloneq \L(X,X)$ eine \emph{Banachalgebra}, das heißt ein vollständiger normierter Vektorraum mit einer Multiplikation, so dass für $T, S ∈ \L(X)$ gilt:
+ \[
+ \norm {TS} \le \norm T \norm S.
+ \]
+\end{korollar}
+
+\begin{bemerkung}
+ Ist $T ∈ \L(X,Y)$, so ist $\ker T$ als Urbild der abgeschlossenen Menge $\{ 0\}$ stets abgeschlossen in $X$.
+ Das Bild hingegen $R(T) \coloneq \im T$ ist im Allgemeinen jedoch nicht abgeschlossen.
+ Wann sind Elemente in $\L(X)$ invertierbar?
+\end{bemerkung}
+
+\begin{satz}
+ Sei $X$ ein Banachraum und $T ∈ \L(X)$ mit $\limsup\limits_{m → \infty } \norm{T}^{1/m} < 1$. Dann ist $(\id - T)^{-1} ∈ \L(X)$ und es gilt
+ \[
+ (\id-T)^{-1} = \lim_{m → \infty } \sum_{n = 0}^m T^n =: \sum_{n = 0}^\infty T^n ∈ \L(X).
+ \]
+ mit Konvergenz in $\L(X)$.
+\end{satz}
+\begin{proof}
+ Wähle $m_0$ und $\Theta < 1$ mit $\norm {T^n} < \Theta ^n$ für $n \ge m_0$.
+ Für $S_k = \sum_{n=0}^k T^n$ gilt dann für $m_0 \le k < l$
+ \[
+ \norm{ S_l - S_k} = \norm { \sum_{n=k+1}^l T^n} \le \sum_{n=k+1}^l \norm{ T^k} \le \sum_{n=k+1}^l \Theta ^n < \epsilon , \quad k, l \ge N_0.
+ \]
+ Damit ist $(S_k)_{k ∈ ℕ}$ eine Cauchy-Folge in $\L(X)$ und somit konvergent.
+ Sei $S$ der Grenzwert. Dann gilt für jedes $x ∈ X$ auch $S_k x \xrightarrow[k → \infty ]{\norm\cdot_{X}} Sx$, also damit ist für alle $x∈ X$
+ \[
+ (\id - T) Sx = \lim_{k → \infty } (\id -T) S_k x = \lim_{k → \infty } \sum_{n=0}^k (T^n -T^{n-1})x = \lim_{k→\infty } x - T^{k+1}x = x.
+ \]
+ Damit ist $(\id -T)S = \id$. Da sich analog $S(\id-T) = \id$ auch zeigen lässt, folgt die Behauptung.
+\end{proof}
+
+
+\begin{lemma}
+ 3.7.6
+\end{lemma}
+\begin{bemerkung-nn}
+ Mit $\Theta = 1$ geht es nicht immer. Gegenbeispiel: Sei $X = C[0,1] ∩ \{ x(0) =
+ 0 \}$ und $M = \{ x ∈ X : g∫_0^1 x(t) dt = 0 \}$.
+ Dann ist $M$ ein abgeschlossener linearer Unterraum, weil $T: X → ℝ, ∫_0^1 \cdot$ stetig ist und somit $M = T^{-1}(\{0\})$ als Urbild einer abgeschlossenen Menge in $ℝ$ abgeschlossen ist.
+ Angenommen, ($\Theta =1$), es existierte ein $x_\Theta = x ∈ X$ mit $\norm x_1 = $ und $\norm {x-x_1} \ge 1 $ für alle $x ∈ M$.
+ Dann setze
+ \[
+ c(y) \coloneq \frac{∫_0^1 x_1(t) dt}{∫_0^1 y(t) dt} ∈ ℝ
+ \]
+ für alle $y \not\in M$. Man beachte, dass dies wohldefiniert ist.
+ Dann ist $x_1 - c(y)y ∈ M$, also $1 \le \norm{ x_1 - c(y)y - x_1} = |c(y)|\norm y$.
+ Dann $∫_0^1 x_1 c(y)y\; dt = 0 $ oder $\frac {1}{|c(y)} \le \norm y $ oder $\left| ∫_0^1 y(t)\;dt \right| \le \left| ∫_0^1x_1(t)\;dt \right| \norm y$ für alle $y ∈ X \setminus M)$.
+ Wähle $y_n(t) = t^{1/n} ∈ X$, also $\norm {y_n} = 1$.
+ Es gilt $\left| ∫_0^1 y_n(t) dt \right| \le \left| ∫_0^1 x_1(t) dt \right| \le 1$ für alle $n ∈ ℕ$, also
+ $∫_0^1 x_1(t) dt = 1$ und $x_1(t) \le 1$, was aber bereits impliziert, dass $x_1$ identisch 1 ist. Damit ist $x_1 \not\in X$.
+\end{bemerkung-nn}
+
+\begin{satz}
+ 7.7
+\end{satz}
+\begin{proof}
+ „⇐“ war Korollar 7.4.
+
+ „⇒“. Angenommen, $\dim X = \infty.$ Sei $S^1 \coloneq \{ x ∈ X: \norm x = 1\}$.
+ Da $S^1$ abgeschlossen und beschränkt ist, ist $S^1$ nach Annahme kompakt.
+ Wähle $x_1 ∈ S^1$ und $M_1 \coloneq \lspan \{ x_1 \} \subsetneq X$.
+ $M_1$ ist ein abgeschlossener Unterraum nach Korollar 7.5.
+ Nach Ries existiert ein $x_2 ∈ S_1$ mit $\norm {x_2-x_1} \ge \Theta \coloneq \frac 1 2 $.
+ Setze nun $M_2 \coloneq \lspan \{x_1,x_2\}$.
+ Da $M_2$ ein abgeschlossener Unterraum ist, existiert ein $x_3 ∈ S_1$ mit $\norm {x_3 - x} \ge \Theta $ für alle $x ∈ M_2$, also insbesondere $\norm {x_3-x_1} \ge \Theta = \frac 1 2$ und $\norm {x_3-x_2} \ge \Theta = \frac 1 2$.
+ Iterativ (da $\dim X = \infty $) existiert $x_n ∈ S_1$ mit $\norm {x_m - x_n} \ge \frac 1 2$ für $m \ge n$.
+ Somit haben wir eine Folge $(x_n)_{n ∈ ℕ}$ ohne Häufungspunkt in $S^1$ gefunden im Widerspruch zu $S^1$ kompakt.
+\end{proof}
+
+Damit sind in unendlich-dimensionalen normierten Räumen weder die Sphären noch die abgeschlossenen Kugeln kompakt.
+
+
+\begin{definition}
+ Ein topologischer linearer Raum $X$ heißt \emph{lokalkompakt}, wenn $0 ∈ X$ eine Umgebung $U$ besitzt, deren Abschluss kompakt ist.
+\end{definition}
+
+\begin{korollar}
+ Sei $X$ normiert, $\dim X = \infty $. Dann ist $X$ nicht lokalkompakt.
+\end{korollar}
+\begin{proof}
+ Angenommen, dass doch. Dann gibt es $r > 0$, so dass $S_r = \{ x ∈ X : \norm x = r\} ⊂ \cl U$.
+ Da $\cl U$ nach Annahme kompakt ist und $S_r$ abgeschlossen, ist $S_r$ ebenfalls kompakt. Das ist ein Widerspruch.
+\end{proof}
+
+
+%%% Local Variables:
+%%% mode: latex
+%%% TeX-master: "funkana-ebook"
+%%% End:
diff --git a/ch04-unitaere-raeume.tex b/ch04-unitaere-raeume.tex
new file mode 100644
index 0000000..1cd310f
--- /dev/null
+++ b/ch04-unitaere-raeume.tex
@@ -0,0 +1,447 @@
+\chapter{Unitäre Räume und Hilberträume}
+\section{Grundbegriffe}
+Sei wieder $\K = \R$ oder $\K = ℂ$.
+
+\begin{definition}
+ Sei $X$ ein linearer Raum über $\K$.
+ Eine Abbildung $\langle \cdot, \cdot \rangle: X × X → \K$ heißt \emph{Skalarprodukt} auf $X$, falls gilt
+ \begin{enumerate}[label=(U\arabic*)]
+ \item
+ $\langle x, x \rangle > 0$ für alle $0 \ne x ∈ X$.
+ \item
+ $\langle x, y \rangle = \cl {\langle y, x \rangle}$ für alle $x, y ∈ X$.
+ \item
+ $\langle x, \alpha y + β z \rangle = \alpha \langle x, y \rangle + β \langle x,z \rangle$ für alle $\alpha , β ∈ \K$, $x,y,z ∈ X$.
+ \end{enumerate}
+ $(X,\langle -,- \rangle)$ heißt \emph{Skalarproduktraum}, \emph{unitärer Raum} oder \emph{Prähilbertraum}.
+\end{definition}
+
+\begin{bemerkung-nn}
+ Offenbar ist $\langle -,- \rangle$ in der ersten Komponente konjugiert linear.
+\end{bemerkung-nn}
+
+\begin{satz}
+ Sei $(X, \langle -,- \rangle)$ ein unitärer Raum. Dann gelten die folgenden Aussagen:
+ \begin{enumerate}
+ \item
+ Durch $\norm x \coloneq \sqrt{\langle x, x \rangle}$ wird eine Norm definiert.
+ Dadurch wird jeder unitäre Raum auf natürliche Art und Weise normiert und trägt dadurch die induzierte natürliche Topologie.
+ \item
+ $|\langle x,y \rangle| \le \norm x \norm y$ mit Gleichheit genau dann, wenn $x$ und $y$ linear abhängig (Cauchy-Schwarz-Ungleichung).
+ \item
+ $\norm {x+y}^2 + \norm{x-y}^2 = 2(\norm x^2 + \norm y^2)$ (Parallelogrammgleichung),
+ \item
+ Für $\K = ℝ$ gilt
+ \[
+ \langle x,y \rangle = \frac 1 4 \left( \norm { x+y}^2 - \norm{x-y}^2 \right),
+ \]
+ für $\K = ℂ$
+ \[
+ \langle x, y \rangle = \frac 1 4 \left( \norm {x+y}^2 - \norm{x-y}^2 - i \norm{x+iy}^2 + i\norm{x-iy}^2 \right).
+ \]
+ \end{enumerate}
+\end{satz}
+\begin{proof}
+ \begin{enumerate}
+ \item
+ Einfaches Nachrechnen unter Verwendung von (b)
+ \item
+ Für $y = 0$ ist die Behauptung klar. Sei also $y \ne 0, \alpha ∈ ℂ$.
+ Dann
+ \[
+ \langle x + \alpha y, x+\alpha y \rangle = \langle x, x \rangle + \cl \alpha \langle y, x \rangle + \alpha \langle x,y \rangle + |\alpha |^2 \langle y,y \rangle.
+ \]
+ Speziell für $\cl \alpha \coloneq - \frac{\langle x,y \rangle}{\langle y,y \rangle}$ ergibt sich
+ \[
+ 0 \le \langle x + \alpha y, x+\alpha + \rangle = \langle x,x \rangle - \frac{|\langle x,y \rangle^2|}{\langle y,y \rangle} - \frac{|\langle x,y \rangle^2|}{\langle y,y \rangle} + \frac{|\langle x,y \rangle^2|}{\langle y,y \rangle} = \langle x,x \rangle - \frac{|\langle x,y \rangle^2|}{\langle y,y \rangle}.
+ \]
+ Durch Umstellen ergibt sich
+ \[
+ \langle x, x \rangle \ge \frac{|\langle x,y \rangle|^2}{\langle y,y \rangle} \gdw |\langle x,y \rangle|^2 \le \norm x ^2 \norm y^2.
+ \]
+ Die CSU erhält man durch Wurzel ziehen.
+ Gleichheit gilt genau dann, wenn
+ \[
+ \langle x+ \alpha y, x+\alpha y \rangle = 0 \gdw x + \alpha y = 0,
+ \]
+ also wenn $x$ und $y$ linear abhängig sind.
+ \item
+ Es gilt
+ \[
+ \norm {x \pm y}^2 = \norm x^2 \pm 2\Re(\langle x,y \rangle) + \norm y ^2.
+ \]
+ Addieren dieser Gleichungen für $+$ und $-$ ergibt die Behauptung.
+ \item
+ Es gilt
+ \begin{align*}
+ \norm {x+y}^2 - \norm{x-y}^2 &= (\norm x^2 + 2 \Re \langle x,y\rangle + \norm y^2) - (\norm x^2 - 2 \Re \langle x,y \rangle + \norm y^2) \\
+ & = 4 \Re \langle x,y \rangle.
+ \end{align*}
+ Analog haben wir
+ \[
+ -i \norm{x+iy}^2 + i \norm{x-iy}^2 = … = 4i \Im \langle x,y \rangle,
+ \]
+ was die Behauptung impliziert.
+ \end{enumerate}
+\end{proof}
+
+\begin{satz}
+ Sei $(X,\norm\cdot)$ ein normierter Raum, der die Parallelogrammgleichung erfüllt.
+ Dann definieren
+ \[
+ \langle x,y \rangle = \frac 1 4 \left( \norm { x+y}^2 - \norm{x-y}^2 \right),
+ \]
+ und
+ \[
+ \langle x, y \rangle = \frac 1 4 \left( \norm {x+y}^2 - \norm{x-y}^2 - i \norm{x+iy}^2 + i\norm{x-iy}^2 \right).
+ \]
+ Skalarprodukte auf $X$ (für $\K = ℝ$ bzw $ℂ$).
+\end{satz}
+\begin{proof}
+ Stupides nachrechnen (oder so ähnlich).
+\end{proof}
+
+\begin{bemerkung}
+ \begin{enumerate}
+ \item
+ Die Paralellogrammgleichung ist somit charakteristisch für unitäre Räume.
+ \item
+ $(C(S),\norm\cdot_\infty )$ mit $S ⊂ ℝ^n$ kompakt erfüllt dies nicht.
+ \item
+ Die Abbildung $\langle \cdot,\cdot \rangle$ in unitären Räumen ist stetig in beiden Komponenten als unmittelbare Konsequenz aus der Stetigkeit der Norm.
+ \end{enumerate}
+\end{bemerkung}
+
+\begin{definition}
+ Ein bezüglich der Norm $\norm \cdot \coloneq \sqrt{ \langle \cdot,\cdot \rangle}$ vollständiger unitärer Raum $(X,\langle \cdot,\cdot \rangle)$ heißt \emph{Hilbertraum}.
+\end{definition}
+
+
+Hier fehlt eine VL.
+
+\begin{korollar}
+ $\hat y$ erfüllt die Gleichung aus dem vorherigen Satz genau dann, wenn $(x- \hat y) \perp Y$ gilt.
+\end{korollar}
+\begin{proof}
+ „⇐“:
+ Sei also $\hat y ∈ Y$ mit $x-\hat y \perp Y$, also $x-\hat y \perp (\hat y - y)$ für $y ∈ Y$ beliebig.
+ Dann gilt mit Pythagoras
+ \[
+ \norm{x-y}^2 = \norm{x-\hat y + \hat y - y}^2 = \norm{x-\hat y}^2 + \norm{\hat y - y}^2 \ge \norm{x-\hat y}^2,
+ \]
+ was die Behauptung impliziert.
+\end{proof}
+\begin{bemerkung-nn}
+ Damit gilt im Hilbertraum das Riesz'sche Lemma (3.7.6) mit $\Theta = 1$.
+ Setze dazu
+ $ x_{\Theta =1} \coloneq \frac{x-\hat y }{\norm{x-\hat y}} $
+ für ein $x \notin Y$. Dann ist $\norm{x_\Theta } = 1$ und für alle $z ∈ Y$
+gilt $\norm {z-x_\Theta }^2 + 2 \Re \langle z,x_\Theta \rangle + \norm{x_\Theta
+}^2 \ge 1 = \Theta $.
+\end{bemerkung-nn}
+\begin{satz}
+ Es sei $Y$ ein vollständiger Unterraum eines unitären Raums $X$.
+ Dann existiert zu jedem $x ∈ X$ eine eindeutige Zerlegung der Form
+ \[
+ x= y + v
+ \]
+ mit $y ∈ Y$ und $v ∈ Y^\perp$, das heißt $X = Y \oplus Y^\perp$.
+\end{satz}
+\begin{proof}
+ Jedes $x ∈ X$ lässt sich als $x = \hat y + (x- \hat y)$ schreiben, wobei $\hat y$ wie im Vorherigen Satz ist.
+ Dann ist $\hat y ∈ Y$ und $(x-\hat y) ∈ Y^\perp$.
+ Für die Eindeutigkeit seien $x = y_1 + v_1 = y_2 + v_2$ zwei Darstellungen von $x$ mit $y_i ∈ Y, v_i ∈ Y^\perp, i=1,2$.
+ Dann $y_1 - y_2 = v_2 - v_1$, wobei die linke Seite in $Y$ ist und die rechte in $Y^\perp$, aber $Y ∩ Y^\perp = \{ 0\}$ nach einem vorherigen Resultat, also $y_1 = y_2$ und $v_1 = v_2$.
+\end{proof}
+\begin{bemerkung-nn}
+Weil für jedes $x ∈ X$ das Element $y = \hat y(x) ∈ Y$ in dieser Darstellung eindeutig ist, lässt sich dadurch eine Abbildung $P: X → X, x ↦ y$ definieren.
+Diese Abbildung ist eine Projektion, das heißt $P \circ P = P$.
+Wir schreiben für $P$ auch $\Proj_Y : X → X$ mit Wertebereich $\im P = Y$ und $P|_Y = \id|_Y$.
+\end{bemerkung-nn}
+\begin{korollar}
+ Falls $M ⊂ X$ ein Unterraum des Hilbertraums $X$ ist, dann gilt
+ \[
+ \cl M = (M^\perp)^\perp.
+ \]
+\end{korollar}
+\begin{proof}
+ „⊂“ wurde bereits in Definition 2.1 gezeigt.
+
+ „$\supset$“: Falls $(M^\perp)\perp \ne \cl M$, dann existiert $x_0 ∈ (M^\perp)^\perp \setminus \cl M$.
+ Da $X$ ein Hilbertraum ist, ist $\cl M$ vollständig.
+ Nach dem Satz vom orthogonalen Komplement gibt es eine eindeutige orthogonale Zerlegung von $x_0 = \hat x_0 + h_0^\perp$ mit $\hat x_0 = \Proj_M(x_0) ∈ \cl M$ und $x_0^\perp ∈ (\cl M)^\perp$.
+ Da $x_0^\perp ∈ (\cl M)^\perp$, ist auch $x_0^\perp ∈ (M)^\perp$ und $x_0 ∈ (M^\perp)^\perp$, also insbesondere $\langle x_0, x_0^\perp \rangle = 0$.
+ Das bedeute mit Hilfe der Zerlegung
+ \[
+ 0 = \langle x_0, x_0^\perp \rangle
+ = \langle \hat x_0 + x_0^\perp, x_0^\perp \rangle
+ = \langle \hat x_0, x_0 ^\perp \rangle + \langle x_0^\perp, x_0^\perp \rangle
+ = \langle x_0^\perp, x_0^\perp \rangle
+ = \norm{x_0^\perp}^2.
+ \]
+ somit ist bereits $x_0^\perp = 0$, also $x_0 = \hat x_0 ∈ \cl M$.
+ Damit ist $\cl M = (M^\perp)^\perp$.
+\end{proof}
+\begin{bemerkung-nn}
+Die Abbildung $P$ ist beschränkt mit Operatornorm $\norm P = \sup\limits_{x \ne 0} \frac{\norm{P(x)}}{\norm x} \le 1$,
+denn für jedes $x = y + v$ mit $y ∈ Y, v ∈ Y^\perp$ gilt
+\[
+ \norm{P(x)}^2 = \norm{y^2} \le \norm y^2 + 2 \Re \langle y, v \rangle + \norm{v}^2 = \norm{y +v}^2 = \norm{x}^2.
+\]
+Desweiteren ist $P$ symmetrisch, das heißt für alle $x_1, x_2 ∈ X $ ist
+\[
+ \langle P(x_1), x_2 \rangle = \langle x_1, P(x_2) \rangle.
+\]
+Ist $x_1 = y_1 + v_2$, $x_2 = y_2 + v_2$ mit $y_i ∈ Y, v_i ∈ Y^\perp, i=1,2$, dann ist
+\[
+ \langle P(x_1), x_2 \rangle = \langle y_1,x_2 \rangle = \langle y_1,y_2 + v_2 \rangle
+ = \langle y_1,y_2 \rangle = \langle y_1+v_1, y_2 \rangle = \langle x_1, P(x_2) \rangle.
+\]
+\end{bemerkung-nn}
+\begin{korollar}
+ Es Sei $Y \ne \{0\}$ ein vollständiger Unterraum des unitären Raums $X$ mit der Projektion $P = \Proj_Y: X → Y ⊂ X$. Dann gilt
+ \begin{enumerate}
+ \item $x-P(x) \perp Y $ für alle $x ∈ X$.
+ \item
+ $P$ ist symmetrisch.
+ \item
+ $P$ ist beschränkt mit Operatornorm $\norm P = 1$.
+ \end{enumerate}
+\end{korollar}
+\begin{proof}
+ (1) und (2) wurden bereits gezeigt. Bei (3) fehlt nur noch „$\ge$“.
+ Da $P_Y = \id|_Y$ und $Y \ne \{0\}$ ist das aber ebenfalls klar.
+\end{proof}
+Zentral in der Hilbertraumtheorie ist der Begriff der Hilbertraumbasis.
+\begin{definition}
+ Ein Orthonormalsystem $(\hat e_k)_{k ∈ ℕ}$ eines unitären Raums $X$ heißt eine Orthonormalbasis oder eine \emph{Hilbertraumbasis}, falls eine der folgenden äquivalenten Bedingungen erfüllt ist:
+ \begin{enumerate}
+ \item Für alle $x ∈ X$ gilt die Vollständigkeitsrelation
+ \[
+ \lim_{n → \infty } \norm{x - \sum_{k=1}^n \langle \hat e_k, x \rangle \hat e_k} = 0
+ \]
+ \item
+ Für alle $x, y ∈ X$ ist
+ \[
+ \langle x,y \rangle = \sum_{k=1}^\infty \cl{\langle \hat e_k. x \rangle} \langle \hat e_k, y \rangle.
+ \]
+ \item
+ Für alle $x ∈ X$ gilt die Parseval-Gleichung
+ \[
+ \norm{x}^2 = \sum_{k=1}^\infty \left| \langle \hat e_k, x \rangle \right|^2.
+ \]
+ \end{enumerate}
+\end{definition}
+\begin{proof}
+ Übung.
+\end{proof}
+\begin{bemerkung-nn}
+ \begin{enumerate}
+ \item Statt (a) kann man auch
+ \[
+ x = \lim_{n → \infty } \sum_{k=1}^n \langle \hat e_k, x \rangle \hat e_k
+ = \sum_{k=1}^\infty \langle \hat e_k,x \rangle \hat e_k
+ \]
+ schreiben. Dies nennt man die Fourier-Reihe von $x$.
+ \item
+ Die approximierenden Elemente
+ \[
+ \sum_{k=1}^n \langle \hat e_k, x \rangle \hat e_k
+ \]
+ liegen offenbar in $\lspan S$ wenn $S = \{ \hat e_k : k ∈ ℕ \}$ ist,
+ was sich nicht notwendigerweise auf den Grenzwert überträgt.
+ Falls $X$ aber vollständig ist (also ein Hilbertraum), so sind diese Aussagen äquivalent zu $\cl{\lspan S}^{\norm\cdot} = X$.
+ \end{enumerate}
+\end{bemerkung-nn}
+\begin{satz}
+ \begin{enumerate}
+ \item
+ Für einen unitärer Raum $X$ gilt: Jede Hilbertraumbasis ist auch ein vollständiges Orthonormalensystem.
+
+ \item
+ Ist zusätzlich $X$ ein Hilbertraum und $(\hat e_k)_{k ∈ ℕ}$ ein vollständiges Orthonormalensystem, dann ist $(\hat e_k)_{k ∈ ℕ}$ auch eine Hilbertraumbasis.
+ \end{enumerate}
+\end{satz}
+\begin{proof}
+ \begin{enumerate}
+ \item
+ Sei $S$ wie oben. Sei $x ∈ X$ mit $x \perp S$. Nach (c) gilt dann
+ \[
+ \sum_{k=1}^\infty \big| \underbrace{\langle \hat e_k^\infty , x \rangle}_{=0} \big|^2 = \norm x ^2,
+ \]
+ also $\norm x = 0$ und $x = 0$.
+ \item
+ Sei nun $S$ ein abzählbares vollständiges Orthonormalensystem und $X$ ein Hilbertraum.
+ Führe den Beweis indirekt.
+ Angenommen, $S$ wäre keine Hilbertraumbasis.
+ Dann gelten die Eigenschaften (a)-(c) aus der Definition nicht und wegen der obigen Bemerkung ist dann $Y \coloneq \cl{\lspan S} \subsetneq X$.
+ $Y$ ist also ein abgeschlossener Unterraum von $X$, und da $X$ Hilbertraum ist, damit vollständig.
+ Nach Satz 2.9 ist $X = Y \oplus Y^\perp$.
+ Insbesondere ist also $Y^\perp \ne \{ 0\}$.
+ Damit gibt es ein $x ∈ X \setminus \{ 0\}$ mit $x ∈ Y^\perp$, also
+ \[
+ \langle \hat e_k, x \rangle = 0
+ \]
+ für alle $k ∈  ℕ$ im Widerspruch zur Vollständigkeit von $S$.
+ \end{enumerate}
+\end{proof}
+\begin{frage-nn}
+ Hat jeder Hilbertraum $H$ mit $\dim H = \infty $ ein abzählbares vollständiges ONS (also eine Hilbertbasis)?
+\end{frage-nn}
+Die Antwort darauf ist nein, aber falls $H$ zusätzlich separabel ist, dann ist sie ja.
+Dagegen ist die Existenz eines vollständigen Orthonormalensystems (also eventuell überabzählbar, also keine ONB) kein Problem:
+\begin{satz}
+ In jedem Hilbertraum $X \ne \{ 0\}$ gibt es ein vollständiges Orthonormalensystem.
+ Es lässt sich sogar jedes ONS $S_0$ zu einem vollständigen Orthonormalensystem $\tilde S_0$ mit $S_0 ⊂ \tilde S_0$ ergänzen.
+\end{satz}
+\begin{proof}
+ Simple Anwendung von Zorns Lemma.
+\end{proof}
+\begin{beispiel}
+ \begin{enumerate}
+ \item
+ Sei $X = L^2(0,2\pi), \K = ℝ$.
+ Dann ist ein VONS in $X$ gegeben durch
+ \[
+ S = \left\{ \frac 1 {\sqrt{2\pi }}\right\}
+ ∪ \left\{ \frac 1 {\sqrt{\pi }} \cos(nx) : n ∈ ℕ\right\}
+ ∪ \left\{ \frac 1 {\sqrt{\pi }} \sin(nx) : n ∈ ℕ\right\}.
+ \]
+ In der klassischen Fourieranalysis werden Entwicklungen nach diesem VONS $S$ untersucht.
+ Man zeigt dort, dass $\lspan S$ bezüglich $\norm\cdot_\infty $ dicht liegt in $C_{\text{per}}([0,2\pi]) = \{ f: \R → \R: f$ ist stetig und $2\pi $-periodisch $\}$.
+ Die Aussage von 2.13(2) und (2.10) liefert nur die Begründung
+für die Dichtheit von $\lspan S$ in $\norm-_{L^2}$.
+ \item
+ Durch $(f,g)_\mu \coloneq ∫_a^b \mu (t) f(t) g(t)\; dt $, wobei $\mu > 0$ und stetig auf $(a,b)$, ist auf $L^2(a,b)$ ein reelles Skalarprodukt definiert.
+ Für verschiedene Gewichtsfunktionen $\mu $ und verschiedene Wahlen von $(a,b)$ erhält man $\mu $-orthogonale Polynomsysteme durch Anwendung des Gram-Schmidt-Verfahrens auf die Monome $\{t^i: i ∈ ℕ_0\}$.
+ \begin{enumerate}[label=(\roman*)]
+ \item
+ $a=-1, b=1$, $\mu (t) = 1$ liefert die Legendre-Polynome.
+ \item
+ $a=-1, b=1$, $\mu (t) = \frac 1 {\sqrt{1-t^2}}$ liefert die Tschebyscheff-Polynome.
+ \item
+ $a=0, b=\infty $, $\mu (t) = \exp(-t)$ liefert die Laguerre-Polynome.
+ \item
+ $a=-\infty , b=\infty $, $\mu (t) = \exp(-t^2)$ liefert die Hermite-Polynome.
+ \end{enumerate}
+ \item
+ Ist $X$ ein unitärer Raum mit ONB, kann er formal vervollständigt werden:
+ Sei also $(\hat e_k)_{k ∈ ℕ} ⊂ X$ diese ONB, dann ist
+ \[
+ H \coloneq \left\{ \sum_{k=1}^\infty c_k \hat e_k: (c_k)_{k ∈ ℕ} ∈ \ell^2 \right\}
+ \]
+ ist ein Hilbertraum, den man die Vervollständigung von $X$ nennt.
+ Das Skalarprodukt zwischen $x = \sum_{k ∈ ℕ} c_k \hat e_k$ und $y = \sum_{k ∈ ℕ} d_k \hat e_k$
+ wird definiert als
+ \[
+ \langle x,y \rangle \coloneq \sum_{k=1}^\infty \cl{c_k} d_k.
+ \]
+ Tatsächlich kann $H$ mit dem Koordinatenraum $\ell^2 = \ell^2(ℕ)$ identifiert werden.
+ Die Abbildung
+ \[
+ \Phi: \ell^2(ℕ) → H, (c_k)_{k ∈ℕ} ↦ \sum_{k=1}^\infty c_k \hat e_k
+ \]
+ ist linear, bijektiv und normerhaltend wegen der Parsevalgleichung
+ \[
+ \norm{x}^2 = \sum_{k=1}^\infty \left| \langle \hat e_k, x \rangle \right|^2.
+ \]
+ Also $\ell^2(ℕ)$ und $H$ isometrisch und insbesondere $H$ vollständig.
+ \end{enumerate}
+\end{beispiel}
+
+
+% VL NÄCHSTE WOCHE
+
+Der Satz 4.1 liefert also, dass die Abbildung $J_x: X → X', y ↦ y'$ definiert
+durch $y': X → \K, x ↦ \langle y,x \rangle$ bijektiv ist.
+Wir schreiben nun
+\[
+\lAngle J_x(y),x \rAngle = \lAngle J_x(y),x \rAngle_{X'×X} \coloneq J(x)(y)[x]
+= \langle y,x \rangle.
+\]
+Diese Abbildung ist sesquiliniear, das heißt
+\[
+ J_x (y_1 + y_2) = J_x (y_1) + J_x(y_2), \quad y_1, y_2 ∈ X,
+\]
+\[
+ J_x(\alpha y) = \cl{\alpha} J_x(y), \quad \alpha ∈ \K,
+\]
+denn
+\[
+ \lAngle J_x(\alpha y),x \rAngle = \langle \alpha y, x \rangle = \cl \alpha \langle y, x \rangle = \cl \alpha J_x(y) [x] = \cl \alpha \lAngle J_x(y), x \rAngle
+ \lAngle \cl \alpha J_x(y), x \rAngle,
+\]
+also $X \cong X'$ sesquilinear isomorph.
+
+Gilt da sauch topologisch?
+Die Topologie von $X'$ sei hierbei die von $\L(X, \K)$, also die von der Norm $\norm{y'}_{X',N} = \sup_{\norm{x} \le 1}|y'[x]|$ erzeugte.
+\begin{satz}
+ $X$ und $X'$ sind Hilberträume und $J_x: X → X'$ ist kanonischer sesquilinearer Isomorphismus, der die Norm erhält, also eine Isometrie.
+
+ Genauer gilt:
+ \begin{enumerate}
+ \item
+ $\langle y_1', y_2' \rangle_{X'} \coloneq \cl{ \langle y_1, y_2 \rangle_X}$, wobei $J_x(y_1) = y_1', J_x(y_2) = y_2'$, macht $X'$ zum Skalarproduktraum.
+ \item
+ Die durch $\langle -,- \rangle_{X'}$ induzierte Norm
+ \[
+ \norm{y'}_{X',S} = \sqrt{\langle y', y' \rangle_{X'}}
+ \]
+ ist gerade die von $X' = \L(X, \K)$ bekannte, das heißt, $\norm{y'}_{X',S} = \norm{y'}_{X',N}$.
+ \item
+ Da $(X',\norm-_{X',N})$ schon bekanntlich vollständndig ist, ist $(X', \langle -,- \rangle)$ damit ein Hilbertraum.
+ \item
+ $J_x: X → X'$ ist eine Isometrie.
+ \end{enumerate}
+\end{satz}
+\begin{proof}
+ \begin{enumerate}
+ \item
+ Beispielsweise ist
+ \[
+ \langle \alpha y_1' , y_2' \rangle_{X'} \stackrel{\text{def}}{=} \cl{\langle \cl \alpha y_1, y_2 \rangle_X} = \cl{ \alpha \langle y_1, y_2 \rangle_X} = \cl{\alpha} \langle y_1',y_2' \rangle_{X'},
+ \]
+ die anderen Eigenschaften folgen analog.
+ \item
+ Wegen $y'[x] = \langle y,x \rangle$ und $\norm{y'}_{X',S} = \sqrt{\langle y', y' \rangle_{X'}} = \sqrt{\langle y, y \rangle_X} = \norm{y}$, das heißt, es genügt, zu zeigen, dass
+ \[
+ \norm{y'}_{X',N} = \sup_{\norm x \le 1} |y'[x]| = \norm{y}_{X} \quad \text{für alle $y ∈ X$}.
+ \]
+ hierbei ist aber „$\le$“ gerade die Cauchy"=Schwarzsche Ungleichung, für „$\ge$“ wähle $x = \frac y {\norm y _{X}}$ für $y \ne 0$ ($y=0$ ist sowieso klar).
+ \item
+ nichts zu zeigen.
+ \item
+ $J_x: X → X'$ ist eine Isometrie, denn $y ↦ J_x(y) = y'$ und $\norm{J_X(y)}_X = \norm{y'}_{X'} = \norm{y}_X$ für alle $y ∈ X$.
+ \end{enumerate}
+\end{proof}
+
+\section{Separable Hilberträume}
+\begin{definition}
+ Ein metrischer Raum $(X,d)$ heißt \emph{separabel}, wenn es $U ⊂ X$ dicht
+ und abzählbar gibt.
+\end{definition}
+
+\begin{beispiele}
+ $ℝ^n, ℂ^n, \ell^2, L^2(\Omega)$ für $\Omega ⊂ ℝ^n$ offen sind separable Hilberträume.
+\end{beispiele}
+
+\begin{satz}
+ In einem separablen unendlich"=dimensionalen Hilbertraum $(X,\langle -,- \rangle)$ gilt
+ \begin{enumerate}
+ \item Jedes ONS in $X$ ist höchstens abzählbar.
+ \item
+ Sei $S = (\hat e_i)_{i ∈ ℕ}$ ein VONS in $X$. Dann existiert zu jeder
+ Folge $\alpha = (\alpha _k)_{k ∈ ℕ} ∈ \ell^2$ genau ein $x ∈ X$ mit $\langle \hat
+ e_k, x \rangle = \alpha _k, k ∈ ℕ$ (Satz von \emph{Riesz-Fischer}).
+ \item
+ $X$ ist isometrisch isomorph zum $\ell^2$. Insbesondere sind
+ $L^2(\Omega)$ und $\ell^2$ isometrisch isomorph.
+ \end{enumerate}
+\end{satz}
+
+
+\section{Riesz'scher Darstellungssatz und Lax-Milgram}
+Für einen topologischen linearen Raum $X$ ist der Dualraum $X' = \{x': X → \K, x' $ linear und stetig $\}$ definiert.
+Im Allgemeinen kann auch $X' = \{0\}$ gelten.
+Ist $X$ jedoch ein Hilbertraum, so ist stets $X' \ne \{0\}$, denn zu $y ∈ X$ ist durch $y'[x] \coloneq \langle y,x \rangle, x ∈ X$ jeweils ein $y' ∈ X'$ erklärt.
+Tatsächlich bekommt man dadurch sogar schon alle Elemente des Dualraums: \ No newline at end of file
diff --git a/ch05-hahn-banach.tex b/ch05-hahn-banach.tex
new file mode 100644
index 0000000..ce92a84
--- /dev/null
+++ b/ch05-hahn-banach.tex
@@ -0,0 +1,370 @@
+\chapter{Der Satz von Hahn-Banach und seine Konsequenzen}
+\section{Fortsetzbarkeit linearer Funktionale}
+
+Wir fragen uns, ob sich Abbildungen so erweitern, dass gewisse Eigenschaften (wie z.B. Linearität oder Stetigkeit) erhalten bleiben.
+
+\begin{definition}
+ Eine Abbildung $A: M → Y$ heißt eine Fortsetzung einer Abbildung $A_0: M_0 → X$, falls
+ \begin{enumerate}
+ \item $ M_0 ⊂ M$,
+ \item $∀x ∈ M_0: A_0 x = Ax $.
+ \end{enumerate}
+ Wir schreiben dann $A = A|_{M_0}$.
+\end{definition}
+
+\begin{satz}
+ Seien $(X,\norm-)$ und $(X_0,\norm-)$ normietre Räume, $X_0 ⊂ X$ dicht in $X$.
+ Weiter sei $(Y, \norm-_{Y})$ ein Banachraum und $A_0 : X_0 → Y$ stetig und linear.
+ Dann gibt es genau eine stetige lineare Fortsetzung $A : X → Y$ von $A_0$ auf $X$.
+ Für diese gilt:
+ \[
+ \norm{A_0}_{\L(X_,Y)} = \norm{A}_{\L(X,Y)}.
+ \]
+\end{satz}
+\begin{proof}
+ Zeigen wir zunächst die Existenz der Fortsetzung.
+ Da $X_0$ dicht in $X$ ist, existiert zu jedem $x ∈ X$ eine Folge $(x_n)_{n \ge1}$, die ganz in $X_0$ liegt und gegen $x$ konvergiert.
+ Wir behaupten, dass $(A_0x_n)_{n ∈ ℕ}$ eine Cauchy-Folge in $Y$ ist.
+ Dazu beachte, dass
+ \[
+ \norm{A_0 x_n - A_0 x_m}_{Y} \le \norm{A_0}_{\L(X_0,Y)} \norm{x_n-x_m} \xrightarrow[n,m → \infty ]{} 0.
+ \]
+ Da $Y$ ein Banachraum ist, ist $(A_0x_n)_{n\ge1}$ konvergiert, etwa gegen $y$.
+ Wir setzen $Ax \coloneq y$.
+ Zunächst ist $A$ wohldefiniert, denn wenn $(z_n)_{n \ge 1}$ eine weitere Folge mit $\lim_{n → \infty } z_n = x$ ist, dann gilt
+ $z_n - x_n \xrightarrow[n→\infty ]{} 0$ und
+ \begin{align*}
+ \norm{A_0 z_n - y} &\le \norm{A_0 z_n - A_0 x_n} + \norm{A_0 x_n - y} \\
+ & \le
+ \norm{A_0} \norm{z_n - x_n} + \norm{A_0 x_n - y} \xrightarrow[n→\infty ]{} 0.
+ \end{align*}
+ Offensichtlich ist $A$ eine Fortsetzung von $A_0$.
+ Dass $A$ linear ist, ist ebenfalls klar.
+ Zur Stetigkeit ist
+ \begin{align*}
+ \norm{Ax}_Y &= \norm{\lim_{n → \infty } A_0 x_n}_Y = \lim_{n → \infty } \norm{A_0 x_n}_{Y} \\
+ &\le
+ \lim_{n → \infty } \norm{A_0}_{\L(X_0,Y)} \norm{x_n}_X = \norm{A_0} \norm{x}.
+ \end{align*}
+ Damit ist $A$ beschränkt, also auch stetig.
+
+ Es gilt $\norm{A_0}_{\L(X_0,Y)} = \norm{A}_{\L(X,Y)}$:
+ „$\ge$“ ist aus dem Vorherigen klar. Für die andere Ungleichung ist
+ \[
+ \norm{A}_{L(X,Y)} =
+ \sup_{\norm{x \le 1}, x ∈ X} \norm{Ax}_{Y}
+ \ge
+ \sup_{\norm{x \le 1}, x ∈ X_0} \norm{Ax}_{Y} = \norm{A_0}_{\L(X_0,Y)}.
+ \]
+
+ Für die Eindeutigkeit sei $B: X → Y$ eine weitere stetige, lineare Fortsetzung von $A_0$.
+ Wie oben existiert zu jedem $x ∈ X$ eine Folge $(x_n)_{n ∈ ℕ} ⊂ X$ mit $\lim_{n → \infty } x_n = x$.
+ Dann ist
+ \[
+ Ax_n = A_0 x_n = Bx_n \quad ∀ n ∈ ℕ
+ \]
+ und für $x ∈ X$
+ \[
+ \norm{B_x - A_x} \le \norm{B_x - Bx_n} + \norm{Bx_n - Ax_n} + \norm{Ax_n - Ax} \xrightarrow[n→\infty ]{} 0,
+ \]
+ da $A$ und $B$ stetig sind. Also $Bx = Ax$ für alle $x ∈ X$ und damit $B = A$.
+\end{proof}
+
+\begin{korollar}
+ Ist $A ∈ \L(X,Y)$, $X, Y$ normiert sowie $Y$ vollständig und $M ⊂ X$ dicht, dann gilt:
+ Falls $Ax = 0$ für alle $x ∈ M$, dann ist $A$ schon die Nullabbildung auf $X$.
+\end{korollar}
+\begin{proof}
+ ~
+\end{proof}
+
+Ist $X_0$ nicht dicht in $X$, wird die Fortsetzung schwieriger.
+
+
+\begin{satz}
+ Auf dem linearen Raum $X$ über $ℝ$ gebe es eine Abbildung $p: X → ℝ$ mit:
+ \begin{enumerate}[label=(\roman*)]
+ \item
+ $p(\alpha x) = \alpha p(x)$ für alle $\alpha \ge 0, x ∈ X$ (positiv homogen)
+ \item
+ $p(x+y) \le p(x) + p(y)$ für alle $x, y ∈ X$ (subadditiv)
+ \end{enumerate}
+
+ Weiter seine $X_0$ ein linearer Teilraum von $X$ und $f_0 : X_0 → ℝ$ eine lineare Abbildung mit
+ \[
+ ∀x ∈ X_0 : f_0(x) \le p(x).
+ \]
+ Dann gibt es eine lineare Fortsetzung $f: X → ℝ$ von $f_0$, welche die Ungleichung respektiert, das heißt
+ \[
+ f|_{X_0} = f_0 \quad \text{und} \quad ∀x ∈ X: f(x) \le p(x).
+ \]
+\end{satz}
+\begin{bemerkung-nn}
+ Halbnormen oder Normen $p$ Erfüllen die Voraussetzungen dieses Satzes.
+\end{bemerkung-nn}
+\begin{proof}
+ Schritt 1.
+ Wir setzen $f_0$ auf $X_1 \coloneq X_0 \oplus \lspan{x_1}$ für ein $x_1 \not\in X$ (existiert immer solange $X_0 \subsetneqq X$).
+ Offenbar hat jedes $x ∈X_1$ eine eindeutig Darstellung als
+ $ y = y + \alpha x_1 $, mit $y ∈ X_0$, $\alpha ∈ ℝ$.
+ Dann ist mit $c ∈ ℝ$ beliebig
+ \[
+ f(x) = f(y + \alpha (x_1)) \coloneq f_0(y) + \alpha c
+ \]
+ eine lineare Abbildung $X_1 → ℝ$, die $f_0$ fortsetzt.
+ Wir müssen $c$ so wählen, dass $f(x) \le p(x)$ für alle $x ∈ X_1$, also $f_0(y) + \alpha c \le p(y+\alpha x_1)$ für alle $y ∈ X_0, \alpha ∈ ℝ$.
+ Mit (i) ist diese Bedingung äquivalent zu zwei anderen Bedingungen:
+ \begin{enumerate}
+ \item
+ Für $a > 0$: $f_0(y/\alpha ) + c \le p(y/\alpha + x_1)$.
+ \item
+ Für $\alpha < 0$: $f_0(-y/\alpha ) - c \le p(-y/\alpha - x_1)$
+ \end{enumerate}
+ für alle $y ∈ X_0$. Der Fall $\alpha = 0$ ist nach Annahme ohnehin klar.
+ Um diese Bedingungen erfüllen zu können, muss $c ∈ ℝ$ so gewählt werden, dass
+ \[
+ ∀y_1, y_2 ∈ X_0: f_0(y_1) - p(y_1 - x_1) \le c \le p(y_2 + x_2) - f_0(y_2).
+ \]
+ Das ist möglich, da
+ \[
+ f_0(y_1) + f_0(y_2) = f_0(y_1+y_2) \le p(y_1 + y_2) = p(y_1 - x_1 + y_2 + x_1) \le p(y_1 - x_1)+p(y_2+x_1).
+ \]
+ Folglich gilt
+ \[
+ \sup_{y_1 ∈ X_0} f_0(y_1-p(y_1-x_1)) \le \inf{y_2 ∈ X_0} p(y_2+x_1)-f_0(y_2).
+ \]
+
+
+ Schritt 2.
+ Finde eine maximale Fortsetzung mit dem Lemma von Zorn.
+ Betrachte dazu
+ \[
+ \{: X \supset D_g \supset X_0 → ℝ\}: g|_{X_0} = f_0 ∧ ∀x ∈ D_g: g(x) \le p(x) \}.
+ \]
+ Diese Menge ordnen wir mit $\succeq$ definiert durch
+ \[
+ h \succeq g \gdw h \text{ ist Fortsetzung von $g$}.
+ \]
+ Nach dem Lemma von Zorn existiert eine maximale Fortsetzung $g^*$ von $f_0$ mit $g^*(x) \le p(x)$ für alle $x ∈ X$.
+ Wäre $D_{g^*}$ nicht $X$, so verfahre wie in Schritt 1 im Widerspruch zur Maximalität.
+ Damit hat $g^*$ die gewünschten Eigenschaften.
+\end{proof}
+
+\begin{bemerkung-nn}
+ \begin{enumerate}
+ \item
+ Ohne die Zusatzforderung $f(x) \le p(x)$ für alle $x ∈X$ ist die lineare Fortsetzbarkeit trivial.
+ \item
+ Eine Fortsetzung für lineare Funktionale $f_0: X_0 → \K = ℂ$ ist analog möglich. % yos IV 4
+ \end{enumerate}
+\end{bemerkung-nn}
+
+%% HIER FEHLT EINE VORLESUNG
+
+\begin{satz}[5.3.1]
+ Sei $(X,\norm\cdot)$ ein normierter Raum über $ℝ$, $M ⊂ X$ abgeschlossen und konvex und $0 ∈ M$.
+
+ Dann existiert zu jedem $x_0 \not\in M$ ein $f ∈ X'$ mit
+ \[
+ f(x_0) > 1 ∧ ∀ x ∈ M: f(x) \le 1.
+ \]
+\end{satz}
+Die Hyperebene $H = \{ x ∈ X: f(x) = 1 + \epsilon \}$ für $0 < \epsilon < f(x_0) < 1$ trennt also $x_0$ und $M$.
+
+\begin{proof}
+ Setze $2r \coloneq \inf_{y ∈ M} \norm{y - x_0}$ (positiv, da $M$ abgeschlossen).
+ Sei $N \coloneq \cl{M + \cl{B_r(0)}} = \cl{\{ z = y + u: y ∈ M, u ∈ \cl{B_r(0)}\}} ⊂ X$.
+ Dann ist (i) $N$ abgeschlossen und (ii) $\cl{B_r(0)} ⊂ N$, da $0 ∈ M$, insbesondere ist $0 ∈ N^\circ$.
+ (iii) ist $N$ konvex: Es genügt, zu zeigen, dass $A = M + B_r(0)$ konvex ist, denn dann ist auch $\cl A$ konvex.
+ Sei $x _i = y_i + v_i, y_i ∈ M, v_i ∈ \cl{B_r(0)}, i=1,2$ und $\alpha ∈ (0,1)$. Dann ist
+ \[
+ \alpha x_1 + (1-\alpha )x_2 = \underbrace{[\alpha y_1 + (1-\alpha )y_2]}_{∈ M} + \underbrace{[\alpha u_1+(1-\alpha )v_2]}_{∈ \cl{B_r(0)}}.
+ \]
+ (iv) ist $x_0 \not\in N$.
+ Angenommen, $x_0 ∈ N$. Dann existiert eine Folge $z_n = y_n + u_n$ in $A$ mit $z_n → x_0 (n→\infty )$.
+ Dann ist für $n_0$ hinreichend groß
+ \[
+ \frac r 2 > \norm{z_{n_0} - x_0} = \norm{y_{n_0 - x_0} + u_{n_0}} \ge |\underbrace{\norm{y_{n_0-x_0}}}_{\ge 2r} - \underbrace{\norm{u_{n_0}}}_{\le r}| \ge r.
+ \]
+
+ Verwende nun das Minkowski-Funktional
+ \[
+ p_N(x) \coloneq \inf \{ρ > 0: ρ^{-1} x ∈ N\}, \quad x ∈ X.
+ \]
+ Dieses hat die Eigenschaften
+ \begin{enumerate}
+ \item
+ $p_N(\alpha x) = \alpha p_n(x),\quad \alpha \ge 0, x ∈ X$ (positiv homogen)
+ \item
+ $p_N(x+y) \le p_N(x) + p_N(y), \quad x, y ∈ X$ (subadditiv)
+ \item
+ $p_N(x) \le 1 \iff x ∈ N$
+ \item
+ Ist zusätzlich $\cl{B_r(0)} ⊂ N$, so gilt $p_nNx) \le r^{-1}\norm x$ für alle $x ∈ X$.
+ \end{enumerate}
+ Sei nun $X_0 \coloneq \lspan\{x_0\}$ und $f_0 : X_0 → ℝ$ linear definiert durch $f_0(x_0) \coloneq p_N(x_0)$.
+ Wir behauptung, dass $f_0 (x) \le p_N(x)$ für alle $x = \lambda x_0 ∈ X_0$.
+ Falls $\lambda \ge 0$, so ist $f_0(x) = f_0(\lambda x_0) = \lambda p_N(x_0) = p_N(\lambda x_0) = p_N(x)$.
+ Falls $\lambda < 0$, so ist wegen $p_n \ge 0$ ohnehin $f_0(\lambda x_0) = \lambda p_N(x_0) \le 0 \le p_N(\lambda x_0)$.
+ Da $p_N$ die Bedingungen (i) und (ii) aus Hahn-Banach erfüllt,
+ gibt es eine lineare Fortsetzung $f$ von $f_0$ mit $f(x) \le p_N(x)$ für alle $x ∈ X$.
+
+ Nun ist $f$ stetig, also $f ∈ X'$, denn für alle $x ∈ X$ gilt
+ \begin{multline*}
+ |f(x) = \max\{f(x), -f(x)\} = \max\{f(x),f(-x)\} \le \max\{p_N(x),p_N(-x)\} \\
+ \le \max\left\{\frac{\norm{x}}{r},\frac{\norm{-x}}{r}\right\} = \frac{\norm x}{r}.
+ \end{multline*}
+
+ Außerdem erfüllt $f$ die Gleichung 3.1 (?), denn
+ \[
+ f(x_0) = f_0(x_0) = p_n(x_0) > 1
+ \]
+ und für $x ∈ M ⊂ N$ gilt
+ \[
+ f(x) \le p_N(x) \le 1.
+ \]
+\end{proof}
+
+\section{Einbettung von $X$ in seinen Bidualraum}
+Zunächst zur Motivation: Sei $X$ ein normierter linearer Raum.
+Dann existiert $X'$ und ist ein Banachraum.
+Aber dann existiert auch $X'' \coloneq (X')'$ und ist ebenfalls ein Banachraum.
+Unser Ziel wird es nun sein, $X$ in $X''$ einzubetten.
+
+\begin{definition}
+ Die kanonische Abbildung $J_0: X → X''$ ist definiert durch
+ \[
+ J_0(x) [x'] = \lAngle J_0(x), x' \rAngle_{X''×X'} \coloneq \lAngle x', x \rAngle_{X'×X} = x'[x] ∈ \K
+ \]
+ für $x ∈ X, x' ∈ X'$.
+
+ Offensichtlich gilt für $x ∈ X$ fest $J_0(x): X' → \K$ linear, aber $J_0(x)$ ist auch stetig bzw beschränkt:
+ Dazu ist
+ \[
+ |J_0(x)[x']| = | \langle \langle x',x \rAngle \le \norm{x'}_{X'} \underbrace{\norm{x}_X}_{=: M}.
+ \]
+ Also ist $J_0(x) ∈ X''$, also insbesondere $J_0$ wohldefiniert.
+ Wegen der linearität von $J_0$ in $x$ schreiben wir statt $J_0(x)$ auch $J_0 x$.
+\end{definition}
+
+\begin{satz}
+ Die kanonische Abbildung $J_0: X → X''$ ist eine normerhaltende lineare Einbettung von $X$ in seinen Bidualraum $X''$.
+\end{satz}
+
+\begin{warnung-nn}
+ $J_0$ ist in der Regel nicht surjektiv.
+\end{warnung-nn}
+
+\begin{proof}
+ Zur Injektivität: Seien $x_1, x_2 ∈ X$ mit $J_0x_1 = J_0x_2$.
+ Dann ist für jedes $x' ∈ X'$
+ \[
+ \lAngle x',x_1 \rAngle = J_0 x_1[x'] = J_0x_2[x'] = \lAngle x', x_2 \rAngle,
+ \]
+ also wegen Linearität von $x'$
+ \[
+ \lAngle x', x_1-x_2 \rAngle = 0.
+ \]
+ Mit Folgerung 2.3(1) folgt $x_1-x_2 = 0$.
+
+ Zur Isometrieeigenschaft bleibt zu zeigen: $\norm{J_0x} = \norm{x}$ für alle $x ∈ X''$.
+ „$\le$“: Aus (4.1) folgt bereits
+ \[
+ \norm{J_0(x)}_{X''} = \sup_{\norm{x'} \le 1} |J_0(x)[x'] \le \norm{x}_X.
+ \]
+ „$\ge$“: Zu $x_0 ∈ X$ existiert nach Korollar 2.1 ein $x_0' ∈ X'$ mit
+ $\norm{x_0'}_{X'} = 1$ und $x_0'[x_0]= \norm{x_0}$.
+ Also folgt
+ \[
+ \underbrace{|J_0x_0[x_0']|}_{\le \norm{J_0x_0}_{X''}} = \lAngle x_0', x_0 \rAngle = \norm{x_0}.
+ \]
+ Da $x_0$ beliebig war, gilt $\norm{J_0x}_{X''} \ge \norm{x}$.
+\end{proof}
+
+\begin{definition}
+ Ein Banachraum $X$ heißt \emph{reflexiv}, wenn $J_0$ surjektiv ist, also $X$ und $X''$ isomorph sind vermöge $J_0$.
+\end{definition}
+
+\begin{bemerkung-nn}
+ Ein unvollständiger normierter Raum hätte offensichtlich keine Chance, reflexiv zu sein.
+\end{bemerkung-nn}
+
+\begin{warnung-nn}
+ „vermöge $J_0$“ in der Definition ist wesentlich, denn es gibt Beispiele mit $X \cong X''$, aber $J_0$ ist nicht surjektiv. %% werner, I 4.7
+\end{warnung-nn}
+
+\begin{satz}
+ Jeder Hilbertraum $H$ ist reflexiv
+\end{satz}
+\begin{proof}
+ Übung.
+\end{proof}
+
+\begin{bemerkung-nn}
+ Offensichtlich sind $H$ und $H''$ isometrisch isomorph:
+ Denn $H$ und $H'$ sind bereits konjugiert linear isomorph via $J_H, X → X'$ (Kapitel IV, \S 5, aus Ries'schem Darstellungssatz).
+ Mit dem gleichen Argument sind $H'$ und $H''$ konjugiert linear isomorph via $J_{H'}$, also $H$ und $H''$ linear isometrisch durch $J_{H'} \circ J_H$.
+ Dies genügt aber nicht für den Nachweis der Reflexivität.
+ Dafür müssen wir zu $x'' ∈ H''$ ein $x ∈ H$ finden mit $J_0x = x''$.
+\end{bemerkung-nn}
+
+\begin{bemerkung-nn}
+ Wozu Reflexivität gut ist, werden wir später im Kapitel über schwache Topologien genauer sehen.
+ Beispielsweise ist $\cl{B_1(0)}$ im reflexiven Banachraum $X$ schwach folgenkompakt, das heißt jede Folge in $\cl{B_1(0)}$ hat eine schwach konvergente Teilfolge mit Grenzwert in $\cl{B_1(0)}$.
+ Dies ist zum Beispiel in der Variationsrechnung sehr wichtig.
+\end{bemerkung-nn}
+
+\begin{definition}
+ Eine Folge $(x_n)_{n ∈ ℕ}$ in einem normierten Raum $X$ heißt \emph{schwach konvergent} gegen $x ∈ X$ (in Zeichen: $x_n \xrightharpoonup[n → \infty ]{} x$), wenn
+ \[
+ \lim_{n → \infty } x'[x_n] = x'[x]
+ \]
+ für alle $x' ∈ X'$ gilt.
+\end{definition}
+
+\begin{bemerkung-nn}
+ Der Grenzwert (so er denn existiert) ist eindeutig. Denn ist $x'[x] = x'[\tilde x]$ für alle $x' ∈ X'$, so folgt $x = \tilde x$ mit Folgerung 2.3 (2).
+\end{bemerkung-nn}
+
+\begin{beispiel-nn}
+ Für $(\hat e_i)_{i ∈ ℕ}$ Hilbertraumbasis in einem separablem Hilbertraum $X$ gilt
+ \[
+ \hat e_i \rightharpoonup 0 ∈ X (i → \infty )
+ \]
+\end{beispiel-nn}
+
+\begin{bemerkung-nn}
+ $(\hat e_i)_{i ∈ ℕ}$ ist nicht konvergent in der Normtopologie, die Folge ist noch nicht mal Cauchy, insbesondere ist $\norm{\hat e_i - 0} \not\rightarrow 0 (i → \infty )$.
+\end{bemerkung-nn}
+
+\begin{proof}
+ Der kanonische Isomorphismus $J_X: X → X', y ↦ y'$ mit $y'[x] = \langle y,x \rangle$ für alle $x ∈ X$ liefert
+ \[
+ X' = \{ x' : x' ∈ X'\} = \{ J_X(y) : y ∈ X\}.
+ \]
+ Zu zeigen ist $\lim\limits_{i → \infty }x'[\hat e_i] = x'[0]$ für alle $x' ∈ X'$, also äquivalent
+ $\lim\limits_{i → \infty } J_x(y)[\hat e_i] = J_x(y)[0]$ für alle $y ∈ X$ bzw. $\lim\limits_{i → \infty } \langle y, \hat e_i \rangle = \langle y, 0 \rangle$ für alle $y ∈ X$.
+
+ Sei also $y ∈ X$ fest gewählt. Dann ist $y = \sum_{i=1}^\infty \alpha _i \hat e_i$ mit $\alpha _i = \langle \hat e_i, y \rangle$.
+ Es gilt $\sum_{i=1}^\infty |\alpha _i|^2 < \infty $ (vgl Def 4.2.12).
+ Damit folgt $\alpha _i = \langle \hat e_i, y \rangle → 0 (i → \infty )$, weil $\alpha ∈ \ell^2$.
+ Damit folgt die Schwache Konvergenz von $(\hat e_i)_{i ∈ ℕ}$.
+\end{proof}
+
+
+\begin{satz}
+ Sei $M$ ein abgeschlossener Unterraum eines Banachraums $(X, \norm -)$.
+ \begin{enumerate}
+ \item
+ Ist $X$ reflexiv, so ist auch $(M, \norm -)$ reflexiv.
+ \item
+ Ist $X$ ist reflexiv, so auch $X'$.
+ \end{enumerate}
+
+\end{satz}
+
+
+%%% Local Variables:
+%%% mode: latex
+%%% TeX-master: "funkana-ebook"
+%%% End: \ No newline at end of file
diff --git a/funkana-ebook.tex b/funkana-ebook.tex
new file mode 100644
index 0000000..4bf0282
--- /dev/null
+++ b/funkana-ebook.tex
@@ -0,0 +1,68 @@
+\documentclass[12pt,footinclude=yes,headinclude=no,a5paper,DIV=50,twoside=false,chapterprefix=true,headings=big]{skript}
+\title{Funktionalanalysis}
+\subtitle{Mitschrift zur Vorlesung}
+\author{\gitAuthorIsoDate \\ {\small\gitReferences} \\ {\small \gitAbbrevHash}}
+% \author{Prof. Dr. Maier-Paape}
+\date{WS 17/18}
+
+\AtBeginDocument{
+\newcommand\norm[1]{\left\|#1\right\|}
+\def\R{\mathbb{R}}
+\def\C{\mathbb{C}}
+\def\K{\mathbb{K}}
+\def\N{\mathbb{N}}
+\def\L{\mathcal{L}}
+\def\T{\mathcal{T}}
+\def\U{\mathcal{U}}
+\def\D{\mathcal{D}}
+\def\dd{\;\mathrm{d}}
+\def\eps{\varepsilon}
+\def\iff{\Leftrightarrow} \def\gdw{\;\Longleftrightarrow\;}
+\newcommand\cl[1]{\overline{#1}}
+\newcommand\ind[1]{\mathbb{1}_{#1}}
+\newcommand\Pot[1]{\mathcal{P}(#1)}
+\DeclareMathOperator{\End}{End}
+\DeclareMathOperator{\grad}{grad}
+\DeclareMathOperator{\lspan}{span}
+\DeclareMathOperator{\supp}{supp}
+\DeclareMathOperator*{\supess}{sup\,ess}
+\DeclareMathOperator{\conv}{conv}
+\DeclareMathOperator{\Proj}{proj}
+\DeclareMathOperator{\im}{im}
+\DeclareMathOperator{\id}{id}
+\renewcommand{\Re}{\operatorname{Re}}
+\renewcommand{\Im}{\operatorname{Im}}
+\def\phi{\varphi}
+\def\epsilon{\varepsilon}
+\def\Tnat{\ensuremath{\T_{\mathrm{nat}}}}
+\def\Tcof{\ensuremath{\T_{\mathrm{cof}}}}
+}
+
+\addbibresource{ref.bib}
+
+\begin{document}
+\sloppy % ohne Ränder sind overfull hboxes eher schlecht, erlaube Unmengen an Kleber.
+\maketitle
+Dies ist eine Vorlesungsmitschrift, die nichts mit den Dozenten oder dem Lehrstuhl, der die Veranstaltung hält, zu tun hat.
+
+Alle Fehler sind vermutlich einzig und allein meine Schuld.
+Über Verbesserungen und Vervollständigungen freue ich mich sehr (bevorzugt Patches an \verb|ulli.kehrle@rwth-aachen.de|).
+
+Der Quelltext dieser Mitschrift ist unter \url{https://git.server-speed.net/users/hrnz/funkana.git} online verfügbar.
+
+Es werden regelmäßig PDFs unter \url{https://hrnz.li/files/funkana/funkana.pdf} (DIN A4, für große Bildschirme und zum Ausdrucken) und \url{https://hrnz.li/files/funkana/funkana-ebook.pdf} (DIN A5, keine Ränder, bietet sich wohl für Ebook-Reader, Smartphones und Tablets an) erscheinen.
+
+\tableofcontents
+\cleardoublepage
+
+\include{motivation}
+\include{ch01-lineare-struktur}
+\include{ch02-topologie}
+\include{ch03-topologisch-lineare-raeume}
+\include{ch04-unitaere-raeume}
+\include{ch05-hahn-banach}
+
+\nocite{*}
+\printbibliography
+
+\end{document}
diff --git a/funkana.tex b/funkana.tex
index 92764ab..7d586d5 100644
--- a/funkana.tex
+++ b/funkana.tex
@@ -1,9 +1,11 @@
-\documentclass[12pt,twoside=false,chapterprefix=true,headings=big]{skript}
+\documentclass[12pt,DIV=12,twoside=false,chapterprefix=true,headings=big]{skript}
\title{Funktionalanalysis}
\subtitle{Mitschrift zur Vorlesung}
+\author{\gitAuthorIsoDate \\ {\small\gitReferences} \\ {\small \gitAbbrevHash}}
% \author{Prof. Dr. Maier-Paape}
\date{WS 17/18}
+\AtBeginDocument{
\newcommand\norm[1]{\left\|#1\right\|}
\def\R{\mathbb{R}}
\def\C{\mathbb{C}}
@@ -13,6 +15,7 @@
\def\T{\mathcal{T}}
\def\U{\mathcal{U}}
\def\D{\mathcal{D}}
+\def\dd{\;\mathrm{d}}
\def\eps{\varepsilon}
\def\iff{\Leftrightarrow}
\def\gdw{\;\Longleftrightarrow\;}
@@ -28,26 +31,38 @@
\DeclareMathOperator{\Proj}{proj}
\DeclareMathOperator{\im}{im}
\DeclareMathOperator{\id}{id}
-\let\Re\relax
-\DeclareMathOperator{\Re}{Re}
-\let\Im\relax
-\DeclareMathOperator{\Im}{Im}
+\renewcommand{\Re}{\operatorname{Re}}
+\renewcommand{\Im}{\operatorname{Im}}
\def\phi{\varphi}
+\def\epsilon{\varepsilon}
\def\Tnat{\ensuremath{\T_{\mathrm{nat}}}}
\def\Tcof{\ensuremath{\T_{\mathrm{cof}}}}
+}
+
+\addbibresource{ref.bib}
\begin{document}
\maketitle
-
Dies ist eine Vorlesungsmitschrift, die nichts mit den Dozenten oder dem Lehrstuhl, der die Veranstaltung hält, zu tun hat.
-Alle Fehler sind vermutlich meine Schuld.
-Über Verbesserungen und Vervollständigungen freue ich mich sehr.
-Diese Mitschrift ist unter \url{https://git.server-speed.net/users/hrnz/funkana.git} verfügbar.
+Alle Fehler sind vermutlich einzig und allein meine Schuld.
+Über Verbesserungen und Vervollständigungen freue ich mich sehr (bevorzugt Patches an \verb|ulli.kehrle@rwth-aachen.de|).
+
+Der Quelltext dieser Mitschrift ist unter \url{https://git.server-speed.net/users/hrnz/funkana.git} online verfügbar.
+
+Es werden regelmäßig PDFs unter \url{https://hrnz.li/files/funkana/funkana.pdf} (DIN A4, für große Bildschirme und zum Ausdrucken) und \url{https://hrnz.li/files/funkana/funkana-ebook.pdf} (DIN A5, keine Ränder, bietet sich wohl für Ebook-Reader, Smartphones und Tablets an) erscheinen.
\tableofcontents
\cleardoublepage
-\input{inhalt}
+\include{motivation}
+\include{ch01-lineare-struktur}
+\include{ch02-topologie}
+\include{ch03-topologisch-lineare-raeume}
+\include{ch04-unitaere-raeume}
+\include{ch05-hahn-banach}
+
+\nocite{*}
+\printbibliography
\end{document}
diff --git a/inhalt.tex b/inhalt.tex
index a0fb88a..2957d64 100644
--- a/inhalt.tex
+++ b/inhalt.tex
@@ -1,9 +1,7 @@
-\section*{Motivation}
-\markboth{}{Motivation}
-
+\section*{Motivation} \markboth{}{Motivation}
In der klassischen Analyis haben wir Funktionen im $\K^n$, wobei $\K$ entweder $ℝ$ oder $ℂ$ ist, untersucht.
Dabei war das Betrachten von Eigenschaften wie Konvergenz, Stetigkeit und Differenzierbarkeit sehr nützlich.
-Die Funktionalanalysis beschäftigt sich nun mit vergleichbaren Problemen in üblicherweise unendlich-dimensionalen Funktionenräumen.
+Die Funktionalanalysis beschäftigt sich nun mit vergleichbaren Problemen in üblicherweise unendlich"=dimensionalen Funktionenräumen.
Hierfür werden wir versuchen, die aus der klassischen Analysis bekannten Untersuchungsmethoden zu verallgemeinern.
Doch zunächst ein paar Probleme, für deren Lösung man die Funktionalanalysis benötigt.
@@ -11,41 +9,41 @@ Doch zunächst ein paar Probleme, für deren Lösung man die Funktionalanalysis
Ein klassisches Beispiel aus der Variationsrechnung:
Wir wollen die Funktion
\[
- f(u) = \int_0^π |u'(x)|^2 dx
+ f(u) = \int_0^\pi |u'(x)|^2 dx
\]
- unter den Nebenbedingungungen $u(0) = u(π) = 0$ und $\int_0^π |u(x)|^2 dx = 1$ minimieren.
+ unter den Nebenbedingungungen $u(0) = u(\pi ) = 0$ und $\int_0^\pi |u(x)|^2 dx = 1$ minimieren.
In der klassischen Analysis haben wir für Minimierungsprobleme mit Nebenbedingungungen Lagrange-Multiplikatoren genutzt.
- Im unendlich-dimensionalen Fall ist das jedoch nicht so einfach.
- Wir betrachten $f : Y → ℝ$ wie oben, wobei $Y$ eine Teilmenge des unendlich-dimensionalen Funktionenraums
+ Im unendlich"=dimensionalen Fall ist das jedoch nicht so einfach.
+ Wir betrachten $f : Y → ℝ$ wie oben, wobei $Y$ eine Teilmenge des unendlich"=dimensionalen Funktionenraums
\[
- X = \left\{ u ∈ C^1[0,π]: u(0) = u(π) = 0 \right\}
+ X = \left\{ u ∈ C^1[0,\pi ]: u(0) = u(\pi ) = 0 \right\}
\]
ist, die durch
\[
- Y = \left\{ u ∈ X: \int_0^π |u(x)|^2 dx = 1 \right\}
+ Y = \left\{ u ∈ X: \int_0^\pi |u(x)|^2 dx = 1 \right\}
\]
gegeben ist.
- Zwar ist $Y$ (in der $\L^2([0,π])$-Metrik) beschränkt und abgeschlossen, jedoch nicht kompakt.
+ Zwar ist $Y$ (in der $\L^2([0,\pi ])$-Metrik) beschränkt und abgeschlossen, jedoch nicht kompakt.
\end{problem-nn}
-\begin{problem-nn}[Fourierreihenentwicklung]
+\begin{problem-nn}
Sei $\mathcal T = \{ 1, \cos t, \sin t, \cos (2t), \sin (2t), … \} =
\{\phi_i\}_{i ∈ ℕ}$. Dann ist bekanntlich
\[
- \langle \phi_i, \phi_j \rangle = ∫_0^{2π} φ_i(t) φ_j(t) dt = 2π δ_{i,j},
+ \langle \phi_i, \phi_j \rangle = ∫_0^{2\pi } φ_i(t) φ_j(t) dt = 2\pi \delta _{i,j},
\]
- wobei $δ_{i,j}$ das Kronecker-Delta bezeichne.
+ wobei $\delta _{i,j}$ das Kronecker-Delta bezeichne.
Also lässt sich durch Normierung ein Orthonormalsystem aus $\mathcal T$ gewinnen.
- Jetzt fragen wir uns, ob sich jede $2π$-periodische Funktion $u$ bezüglich eines geeigneten Konvergenzbegriffs in eine Reihe $u = \sum_{i ∈ ℕ} α_i φ_i$ mit $α_i ∈ ℝ$ entwickeln können.
+ Jetzt fragen wir uns, ob sich jede $2\pi $-periodische Funktion $u$ bezüglich eines geeigneten Konvergenzbegriffs in eine Reihe $u = \sum_{i ∈ ℕ} \alpha _i φ_i$ mit $\alpha _i ∈ ℝ$ entwickeln können.
Bereits bekannt ist, dass das für das entsprechende endlich-dimensionale Problem geht: Sei $T = \{ e_1,…,e_n\}$ die kanonische Standardbasis des $ℝ^n$
Dann gilt bekanntlich
\[
- \langle e_i, e_j \rangle_{ℝ^n} = δ_{i,j}
+ \langle e_i, e_j \rangle_{ℝ^n} = \delta _{i,j}
\]
und für jedes $x ∈ ℝ^n$ ist
\[
- x = \sum_{i=1}^n α_i e_i, \quad α_i = \langle x, e_i \rangle_{ℝ^n}.
+ x = \sum_{i=1}^n \alpha _i e_i, \quad \alpha _i = \langle x, e_i \rangle_{ℝ^n}.
\]
- Wir fragen uns nach den Zusammenhängen zwischen den Problemen im endlich- und unendlich-dimensionalen.
+ Wir fragen uns nach den Zusammenhängen zwischen den Problemen im endlich- und unendlich"=dimensionalen.
\end{problem-nn}
\begin{problem-nn}
Das Biegemoment eines Trägers kann man als Randwertaufgabe (gesucht ist $u: [0,1] → ℝ$, gegeben sind $p,r: [0,1] → ℝ$)
@@ -54,7 +52,7 @@ Doch zunächst ein paar Probleme, für deren Lösung man die Funktionalanalysis
\]
bestimmen. Mit Hilfte der sogenannten Green'schen Funktion lässt sich diese Randwertaufgabe in eine Integralgleichung
\[
- (T_u)(t) := ∫_0^1 G(t,s) \big(r(s)-p(s)u(s)\big) ds = u
+ (T_u)(t) \coloneq ∫_0^1 G(t,s) \big(r(s)-p(s)u(s)\big) ds = u
\]
umwandeln. Das heißt, man sucht einen Fixpunkt eines Integraloperators $T$ in einer geeigneten Menge von Funktionen.
\end{problem-nn}
@@ -77,11 +75,11 @@ Sei im folgenden stets $\K = ℝ$ oder $\K = ℂ$. Zunächst die
\[
\cdot : \K × X → X
\]
- heißt $\K$-Vektorraum, falls für alle $α, β ∈ \K$ und $x, y ∈ X$ gilt:
+ heißt $\K$-Vektorraum, falls für alle $\alpha , β ∈ \K$ und $x, y ∈ X$ gilt:
\begin{enumerate}[label=(V\arabic*)]
- \item $α x+y) = αx + βy$
- \item $(α+β)x = αx + βx$
- \item $(αβ)x = α(βx)$
+ \item $\alpha x+y) = \alpha x + βy$
+ \item $(\alpha +β)x = \alpha x + βx$
+ \item $(\alpha β)x = \alpha (βx)$
\item $1 \cdot x = x$
\end{enumerate}
\end{definition}
@@ -91,7 +89,7 @@ Sei im folgenden stets $\K = ℝ$ oder $\K = ℂ$. Zunächst die
\end{bemerkung-nn}
\begin{bemerkung-nn}
- Eine nichtleere Teilmenge $Y ⊂ X$ ist bereits dann ein linearer Raum, falls aus $α, β ∈ \K$, $x, y ∈ Y$ bereits $αx + βy ∈ Y$ folgt, also $Y$ abgeschlossen unter den Vektorraumoperationen ist.
+ Eine nichtleere Teilmenge $Y ⊂ X$ ist bereits dann ein linearer Raum, falls aus $\alpha , β ∈ \K$, $x, y ∈ Y$ bereits $\alpha x + βy ∈ Y$ folgt, also $Y$ abgeschlossen unter den Vektorraumoperationen ist.
$Y$ heißt dann \emph{linearer Teilraum} oder auch \emph{linearer Unterraum}.
\end{bemerkung-nn}
@@ -99,24 +97,24 @@ Sei im folgenden stets $\K = ℝ$ oder $\K = ℂ$. Zunächst die
Zu jeder Teilmenge $M ⊂ X$ bildet die Menge aller Linearkombinationen von je endlich vieler Elemente einen linearen Teilraum von $X$.
Dieser heißt die \emph{lineare Hülle} von $M$ oder der \emph{Aufspann} von $M$
\[
- \lspan M = \left\{ x ∈ X: ∃ l ∈ ℕ, α_1,…,α_l ∈ \K, m_1,…,m_l ∈ M \text{ mit } \sum_{i=1}^l α_i m_i = x \right\}.
+ \lspan M = \left\{ x ∈ X: ∃ l ∈ ℕ, \alpha _1,…,\alpha _l ∈ \K, m_1,…,m_l ∈ M \text{ mit } \sum_{i=1}^l \alpha _i m_i = x \right\}.
\]
\end{bemerkung-nn}
\begin{bemerkung-nn}
- $M = \{x_λ\}_{λ ∈ Λ} ⊂ X$ heißt \emph{Basis} oder \emph{Hamel-Basis} von $X$, falls $M$ \emph{linear unabhängig}, das heißt,
- $0 ∈ X$ lässt sich nur auf triviale Art und Weise als Linearkombination endlich vieler der $x_λ$ schreiben, und $\lspan M = X$ ist.
+ $M = \{x_\lambda \}_{\lambda ∈ \Lambda } ⊂ X$ heißt \emph{Basis} oder \emph{Hamel-Basis} von $X$, falls $M$ \emph{linear unabhängig}, das heißt,
+ $0 ∈ X$ lässt sich nur auf triviale Art und Weise als Linearkombination endlich vieler der $x_\lambda $ schreiben, und $\lspan M = X$ ist.
\end{bemerkung-nn}
\begin{bemerkung-nn}
- Besitzt $X$ eine Basis von $n < ∞$ Elementen, dann heißt $n$ die \emph{Dimension} von $X$ und wir schreiben $\dim X = n$.
- Andernfalls heißt $X$ \emph{unendlich-dimensional} ($\dim X = ∞$).
+ Besitzt $X$ eine Basis von $n < \infty $ Elementen, dann heißt $n$ die \emph{Dimension} von $X$ und wir schreiben $\dim X = n$.
+ Andernfalls heißt $X$ \emph{unendlich-dimensional} ($\dim X = \infty $).
\end{bemerkung-nn}
\begin{bemerkung-nn}
Seien $X_1, X_2 ⊂ X$ lineare Teilräume. Dann ist
\[
- X_1 + X_2 := \left\{ αx_1 + βx_2: α, β ∈ \K, x_1 ∈ X_1, x_2 ∈ X_2 \right\}
+ X_1 + X_2 \coloneq \left\{ \alpha x_1 + βx_2: \alpha , β ∈ \K, x_1 ∈ X_1, x_2 ∈ X_2 \right\}
\]
ebenfalls ein linearer Teilraum.
Falls $X_1 ∩ X_2 = \{ 0\}$, schreiben wir $X_1 \oplus X_2$ und nennen die Summe \emph{direkt}.
@@ -139,16 +137,16 @@ Sei im folgenden stets $\K = ℝ$ oder $\K = ℂ$. Zunächst die
\[
C[a,b] = \{x: [a,b] → \K, x \text { ist stetig}\}
\]
- ein $\K$-Vektorraum mit $\dim C[a,b] = ∞$.
+ ein $\K$-Vektorraum mit $\dim C[a,b] = \infty $.
Zum Beispiel sind die Monome $(t^k)_{k ∈ ℕ}$ ein unendliches linear unabhängiges System, jedoch keine Basis.
Tatsächlich ist jede Basis dieses Raumes überabzählbar.
\end{beispiel}
\section{Lineare Abbildungen}
\begin{definition}
- Seien $X, Y$ lineare Räume über $\K$. $A: X → Y$ heißt \emph{linear}, falls für alle $x_1, x_2 ∈ X$ und $α, β ∈ \K$ gilt:
+ Seien $X, Y$ lineare Räume über $\K$. $A: X → Y$ heißt \emph{linear}, falls für alle $x_1, x_2 ∈ X$ und $\alpha , β ∈ \K$ gilt:
\[
- A(αx_1 + βx_2) = αA(x_1) + βA(x_2).
+ A(\alpha x_1 + βx_2) = \alpha A(x_1) + βA(x_2).
\]
$A: X → \K$ heißt \emph{lineares Funktional}.
Für $A$ linear heißt $R(A) = \im A = \{A(x): x ∈ X\}$ der \emph{Bildraum} von $A$ und $N(A) = \ker A = \{ x ∈ X: A(x) = 0\}$ der \emph{Kern} von $A$.
@@ -157,7 +155,7 @@ Sei im folgenden stets $\K = ℝ$ oder $\K = ℂ$. Zunächst die
\begin{bemerkung}
Sei $A: X → Y$ linear.
\begin{enumerate}
- \item Sei $M ⊂ X $ ein linearer Unterraum. Dann ist $A(M) ⊂ Y$ wieder ein linearer Unterraum und es gilt $\dim A(M) \le \dim M$ mit Gleichheit bei injektivität.
+ \item Sei $M ⊂ X $ ein linearer Unterraum. Dann ist $A(M) ⊂ Y$ wieder ein linearer Unterraum und es gilt $\dim A(M) \le \dim M$ mit Gleichheit bei Injektivität.
\item Es gilt
\[
A \text{ injektiv} \Longleftrightarrow N(A) = \{ 0\}.
@@ -167,14 +165,14 @@ Sei im folgenden stets $\K = ℝ$ oder $\K = ℂ$. Zunächst die
X/(N(A)) \cong \im A.
\]
\item
- Falls $\dim X = \dim Y = n < ∞$, dann ist $A$ genau dann injektiv, wenn $A$ surjektiv ist.
+ Falls $\dim X = \dim Y = n < \infty $, dann ist $A$ genau dann injektiv, wenn $A$ surjektiv ist.
\item
- $A: X → Y$ ist linear und bijektiv genau dann, wenn es eine lineare Umkehrabbildung $A^{-1}: Y → X$.
+ $A: X → Y$ ist linear und bijektiv genau dann, wenn es eine lineare Umkehrabbildung $A^{-1}: Y → X$ gibt.
\item
Falls so ein $A: X → Y$ linear und bijektiv existiert, nennen wir $X$ und $Y$ \emph{linear isomorph.}
$A$ heißt dann ein \emph{linearer Isomorphismus}.
- Nur falls $\dim X = \dim Y < ∞$ sind $X$ und $Y$ auch „topologisch“ isomorph.
+ Nur falls $\dim X = \dim Y < \infty $ sind $X$ und $Y$ auch „topologisch“ isomorph.
In diesem Fall erhält man die Prototypen $ℝ^n$ und $ℂ^n$ für endlich-dimensionale Vektorräume und andere gitbt es nicht (die sie auch als Topologische Räume isomorph sind).
\end{enumerate}
\end{bemerkung}
@@ -183,7 +181,7 @@ Sei im folgenden stets $\K = ℝ$ oder $\K = ℂ$. Zunächst die
$X = \{ x: [a,b] → ℝ, x, \dot x, \ddot x \text{ stetig},\; x(a) = \dot x(a) = 0\}$ ist ein linearer Raum.
Sei $Y = C[a,b]$ und $A: X → Y$ gegeben durch
\[
- (Ax)(t) := \ddot x(t) + c_1 (t) \dot x (t) + c_2 (t) x(t), \quad t ∈ [a,b], c_1,c_2 ∈ C[a,b].
+ (Ax)(t) \coloneq \ddot x(t) + c_1 (t) \dot x (t) + c_2 (t) x(t), \quad t ∈ [a,b], c_1,c_2 ∈ C[a,b].
\]
Dann ist $A$ linear, weil differenzieren linear ist und $A$ ist injektiv:
Zunächst ist $x = 0$ eine Lösung der linearen Differentialgleichung zweiter Ordnung $Ax = 0$.
@@ -201,16 +199,16 @@ Sei im folgenden stets $\K = ℝ$ oder $\K = ℂ$. Zunächst die
\begin{beispiel-nn}
Sei $X = Y = C[a,b]$, $A: X → X$ gegeben durch
\[
- (Ax)(t) := ∫_a^b k(s,t) x(s) ds, \quad t ∈ [a,b],
+ (Ax)(t) \coloneq ∫_a^b k(s,t) x(s) ds, \quad t ∈ [a,b],
\]
wobei $k : [a,b] × [a,b] → ℝ$ stetig und gegeben ist.
Dann ist $A$ linear, da das Integral linear ist.
- Auch ist, wenn $λ ∈ ℝ$ ein Parameter ist, die Abbildung
+ Auch ist, wenn $\lambda ∈ ℝ$ ein Parameter ist, die Abbildung
\[
- (A_λx)(t) := λx(t) - (Ax)t), \quad t ∈ [a,b]
+ (A_\lambda x)(t) \coloneq \lambda x(t) - (Ax)t), \quad t ∈ [a,b]
\]
linear.
- Die Probleme $Ax = y$ (bei gegebenem $y ∈ Y$ und gesuchtem $x ∈ X$) oder $A_λ x = 0$ (gesucht ist $λ ∈ ℝ$ und eine nichttriviale Lösung $x ∈ X \setminus \{ 0\}$)
+ Die Probleme $Ax = y$ (bei gegebenem $y ∈ Y$ und gesuchtem $x ∈ X$) oder $A_\lambda x = 0$ (gesucht ist $\lambda ∈ ℝ$ und eine nichttriviale Lösung $x ∈ X \setminus \{ 0\}$)
heißen Integralgleichungen erster und zweiter Ordnung.
\end{beispiel-nn}
@@ -233,7 +231,7 @@ Sei im folgenden stets $\K = ℝ$ oder $\K = ℂ$. Zunächst die
Ax = (0,ξ_1, ξ_2, \dots) ∈ \ell^2.
\]
$A$ heißt (Rechts-)Shiftoperator und ist linear und injektiv, jedoch nicht surjektiv.
- Solche Abbildungen gibt es für $\dim X = \dim Y < ∞$ nicht.
+ Solche Abbildungen gibt es für $\dim X = \dim Y < \infty $ nicht.
\end{beispiel-nn}
\section{Duale Räume}
@@ -247,14 +245,14 @@ Wir schreiben nun
\]
Wir setzen
\[
- X^f := \left\{ x': x' \text{ ist lineares Funktional auf } X \right\}.
+ X^f \coloneq \left\{ x': x' \text{ ist lineares Funktional auf } X \right\}.
\]
Hierbei sollte man nicht $x'$ nicht mit der Ableitung von $x$ verwechseln.
Auch ist $\langle -, - \rangle_{X × X^f}$ kein Skalarprodukt.
Der Raum $X^f$ wird auf natürlicher Weise zum linearen Raum mit
\[
- (αx_1' + βx_2')(x) := αx_1'(x) + βx_2'(x), \quad x ∈ X, x_1', x_2' ∈ X^f, α, β ∈ \K.
+ (\alpha x_1' + βx_2')(x) \coloneq \alpha x_1'(x) + βx_2'(x), \quad x ∈ X, x_1', x_2' ∈ X^f, \alpha , β ∈ \K.
\]
So ist
\[
@@ -263,7 +261,7 @@ So ist
bilinear.
\begin{definition}
$X^f$ heißt der \emph{algebraische Dualraum} zu $X$.
- $X^{ff} := (X^f)^f$ heißt der \emph{biduale Raum} zu $X$.
+ $X^{ff} \coloneq (X^f)^f$ heißt der \emph{biduale Raum} zu $X$.
\end{definition}
\begin{beispiel-nn}
@@ -273,7 +271,7 @@ bilinear.
\]
mit
\[
- \langle x', x'' \rangle := \langle x. x' \rangle \quad ∀x' ∈ X^f.
+ \langle x', x'' \rangle \coloneq \langle x. x' \rangle \quad ∀x' ∈ X^f.
\]
Damit ist $x'': X^f → \K$ linear wohldefiniert.
\end{beispiel-nn}
@@ -283,23 +281,23 @@ bilinear.
\end{definition}
\begin{bemerkung}
- $X$ ist genau dann algebraisch reflexiv, wenn $\dim X < ∞$ ist.
+ $X$ ist genau dann algebraisch reflexiv, wenn $\dim X < \infty $ ist.
- Im Fall $\dim X < ∞$ lässt sich leicht eine duale Basis angeben:
- Sei dazu $M := \{x_1,…,x_n\}$ eine Basis von $X$. Dann wird durch
+ Im Fall $\dim X < \infty $ lässt sich leicht eine duale Basis angeben:
+ Sei dazu $M \coloneq \{x_1,…,x_n\}$ eine Basis von $X$. Dann wird durch
\[
- \langle x_i, x_k' \rangle := δ_{i,k}
+ \langle x_i, x_k' \rangle \coloneq \delta _{i,k}
\]
- und linearer Fortsetzung die Menge $ M := \{x_1',…,x_n'\} ⊂ X^f$ erklärt.
+ und linearer Fortsetzung die Menge $ M \coloneq \{x_1',…,x_n'\} ⊂ X^f$ erklärt.
Dann ist $M'$ eine Basis von $X'$, die die \emph{duale Basis} von $M$ genannt wird.
- Tatsächlich ist $X^f$ im Falle $\dim X = ∞$ wesentlich größer.
+ Tatsächlich ist $X^f$ im Falle $\dim X = \infty $ wesentlich größer.
Man wählt deshalb eine (neue) Defintion des Dualraums:
\end{bemerkung}
\begin{definition}[Dualraum]
Zu einem linearen Raum $X$ ist
\[
- X' := \left\{ x' : X → \K, x' \text{ linear und stetig} \right\} ⊂ X^f
+ X' \coloneq \left\{ x' : X → \K, x' \text{ linear und stetig} \right\} ⊂ X^f
\]
der Dualraum von $X$.
\end{definition}
@@ -320,7 +318,7 @@ Um Allerdings von Stetigkeit reden zu können, müssen wir zunächst \emph{Topol
\item
Für alle Mengen $X$ ist $\T = \Pot X$ eine Topologie, die sogenannte \emph{diskrete Topologie} oder \emph{feinste Topologie} auf $X$.
\item
- In Analysis I wird eine Menge $U ⊂ ℝ$ für offen erklärt, wenn es zu jedem $x ∈ U$ ein $ε > 0$ gibt, so dass für alle $ y ∈ ℝ$ mit $|x - y| < ε$ auch $y ∈ U$ gilt.
+ In Analysis I wird eine Menge $U ⊂ ℝ$ für offen erklärt, wenn es zu jedem $x ∈ U$ ein $\epsilon > 0$ gibt, so dass für alle $ y ∈ ℝ$ mit $|x - y| < \epsilon $ auch $y ∈ U$ gilt.
Aus der Analysis ist bekannt, dass die so definierten offenen Mengen den Axiomen genügen.
Diese Topologie $\Tnat$ wird \emph{natürliche Topologie} genannt.
\item
@@ -342,22 +340,22 @@ Um Allerdings von Stetigkeit reden zu können, müssen wir zunächst \emph{Topol
\item
$U ⊂ X$ heißt \emph{Umgebung von $A$}, wenn es eine offene Menge $V$ gibt mit $A ⊂ V ⊂ U$. Wir setzen
\[
- \U_A := \U_A (\T) := \{ U ⊂ X : U\; \text{Umgebung von $A$}\}.
+ \U_A \coloneq \U_A (\T) \coloneq \{ U ⊂ X : U\; \text{Umgebung von $A$}\}.
\]
$\U_A$ heißt \emph{Umgebungssystem} oder \emph{Umgebungsfilter} von $A ⊂ X$.
- Für $x ∈ X$ setzen wir $\U_x := \U_{\{x\}}$. $x$ heißt dann \emph{innerer Punkt} von $U$ für alle $U ∈ \U_x$.
+ Für $x ∈ X$ setzen wir $\U_x \coloneq \U_{\{x\}}$. $x$ heißt dann \emph{innerer Punkt} von $U$ für alle $U ∈ \U_x$.
\item
$x ∈ X$ heißt \emph{Häufungspunkt} von $M$, falls jede Umgebung von $x_0$ ein $y ∈ M$ enthält mit $y \ne x$.k
\item
Das \emph{Innere von M} ist
\[
- M^\circ := \bigcup \left\{ U ∈ \T: U ⊂ M \right\}
+ M^\circ \coloneq \bigcup \left\{ U ∈ \T: U ⊂ M \right\}
\]
die größte offene Menge, die in $M$ enthalten ist.
\item
Der \emph{Abschluss von} M ist
\[
- \cl M := \bigcap \left\{ U ⊂ M: U \text{ abgeschlossen} \right\}
+ \cl M \coloneq \bigcap \left\{ U ⊂ M: U \text{ abgeschlossen} \right\}
\]
die kleinste abgeschlossene Menge, die $M$ enthält.
\item
@@ -455,7 +453,7 @@ Um Allerdings von Stetigkeit reden zu können, müssen wir zunächst \emph{Topol
\begin{definition}[Relativtopologie oder Spurtopologie]
$M \subset \T$ eines topologischen Raumes $(X,\T)$ lässt sich in natürlicher Weise
- zu einem topologischen Raum machen, nämlich mit $\T' := {M \cap V : V \in \T}$.
+ zu einem topologischen Raum machen, nämlich mit $\T' \coloneq {M \cap V : V \in \T}$.
\end{definition}
\begin{bemerkung}
$M = M \cap X \in \T'$ da $X \in \T$, d.h. $M$ ist offen in der Spurtopologie.
@@ -507,21 +505,23 @@ Um Allerdings von Stetigkeit reden zu können, müssen wir zunächst \emph{Topol
\item
Es gilt
\[
- \lim_{n \to ∞} x_n = x \; \Longleftrightarrow \; \lim_{n→∞} d(x,x_n) = 0.
+ \lim_{n \to \infty } x_n = x \; \Longleftrightarrow \; \lim_{n→\infty } d(x,x_n) = 0.
\]
\item
- Es ist $x_0 ∈ M$ genau dann ein innerer Punkt von $M ⊂ X$, wenn ein $ε > 0$ existiert mit $B_ε(x_0) ⊂ M$.
+ Es ist $x_0 ∈ M$ genau dann ein innerer Punkt von $M ⊂ X$, wenn ein $\epsilon > 0$ existiert mit $B_\epsilon (x_0) ⊂ M$.
\item
- $M$ ist nirgends dicht in $X$ genau dann, wenn es zu jeder Kugel $B_ε(x_0)$ mit $x_0 ∈ X, ε > 0$ eine Kugel $B_δ(x_1) ⊂ B_ε(x_0)$ mit $B_θ(x_1) ∩ M = \emptyset$ gibt.
+ $M$ ist nirgends dicht in $X$ genau dann, wenn es zu jeder Kugel
+$B_\epsilon (x_0)$ mit $x_0 ∈ X, \epsilon > 0$ eine Kugel $B_\delta (x_1) ⊂
+B_\epsilon (x_0)$ mit $B_\delta(x_1) ∩ M = \emptyset$ gibt.
\item
Seien $(X,d_X)$ und $(Y,d_Y)$ metrische Räume.
Dann ist auch $(X×Y,d_{X×Y})$ ein metrischer Raum vermöge der Metrik
\[
- d_{X×Y}((x_1,y_1),(x_2,y_2)) := \max\{d_x(x_1,x_2),d_y(y_1,y_2)\}
+ d_{X×Y}((x_1,y_1),(x_2,y_2)) \coloneq \max\{d_x(x_1,x_2),d_y(y_1,y_2)\}
\]
oder auch mit
\[
- d_{X×Y}((x_1,y_1),(x_2,y_2)) := \sqrt{d_x^2(x_1,x_2)+d_y^2(y_1,y_2)}.
+ d_{X×Y}((x_1,y_1),(x_2,y_2)) \coloneq \sqrt{d_x^2(x_1,x_2)+d_y^2(y_1,y_2)}.
\]
Tatsächlich induzieren diese beiden Metriken die gleiche Topologie (nämlich die Produkttopologie)
\item
@@ -532,7 +532,7 @@ Um Allerdings von Stetigkeit reden zu können, müssen wir zunächst \emph{Topol
heißen \emph{Isometrien}.
\item
Ein metrischer Raum muss im allgemeinen keine lineare Struktur haben.
- Man betrachte hierzu die Menge $X := \{1,2,3,4,5,6\}$ mit der diskreten Metrik.
+ Man betrachte hierzu die Menge $X \coloneq \{1,2,3,4,5,6\}$ mit der diskreten Metrik.
Diese kann keine Vektorraumstruktur haben, da $|X| = 6$ keine Primzahlpotenz ist.
\end{enumerate}
\end{lemma}
@@ -561,7 +561,7 @@ Nun ein paar Charakterisierungen von kompakten Mengen in metrischen Räumen.
\section{Vollständigkeit in metrischen Räumen und der Satz von Baire}
\begin{definition}
- Eine Folge $(x_n)_{n ∈ ℕ} ⊂ X$ in $(X,d)$ heißt \emph{Cauchy-Folge}, falls zu jedem $ε > 0$ ein $N = N(ε)$ existiert mit $d(x_m,x_n) < ε$ für alle $n,m \ge N$.
+ Eine Folge $(x_n)_{n ∈ ℕ} ⊂ X$ in $(X,d)$ heißt \emph{Cauchy-Folge}, falls zu jedem $\epsilon > 0$ ein $N = N(\epsilon )$ existiert mit $d(x_m,x_n) < \epsilon $ für alle $n,m \ge N$.
\end{definition}
\begin{lemma}
@@ -580,14 +580,14 @@ jedoch lässt sich jeder metrische Raum zu einem vollständigen Erweitern.
Dieser Raum $(\tilde X, \tilde d)$ heißt die Vervollständigung von $(X,d)$.
\end{satz}
\begin{proof}
- Zwei Cauchyfolgen $(x_n)_{n ∈ ℕ}$ und $(y_n)_{n ∈ ℕ}$ seien äquivalent, falls $d(x_n,y_n) \xrightarrow[n → ∞]{} 0$.
+ Zwei Cauchyfolgen $(x_n)_{n ∈ ℕ}$ und $(y_n)_{n ∈ ℕ}$ seien äquivalent, falls $d(x_n,y_n) \xrightarrow[n → \infty ]{} 0$.
Hierdurch ist eine Äquivalenzrelation definiiert. Sei $[(x_n)_{n ∈ ℕ}]$ die vom Repräsententaten $(x_n)_{n ∈ ℕ}$ erzeugte Klasse. Man setzt
\[
- \tilde X := \{ [ (x_n)_{n ∈ ℕ}] : (x_n)_{n ∈ ℕ} \text{ ist Cauchy-Folge in }(X,d)\}
+ \tilde X \coloneq \{ [ (x_n)_{n ∈ ℕ}] : (x_n)_{n ∈ ℕ} \text{ ist Cauchy-Folge in }(X,d)\}
\]
und
\[
- \tilde d([(x_n)_{n ∈ ℕ}],[(y_n)_{n ∈ ℕ}]) := \lim_{n → ∞} d(x_n,y_n).
+ \tilde d([(x_n)_{n ∈ ℕ}],[(y_n)_{n ∈ ℕ}]) \coloneq \lim_{n → \infty } d(x_n,y_n).
\]
Dann ist $(d(x_n,y_n))_{n ∈ ℕ}$ eine Cauchy-Folge in $ℝ$, da
\[
@@ -605,17 +605,18 @@ jedoch lässt sich jeder metrische Raum zu einem vollständigen Erweitern.
\begin{bemerkung-nn}
Wendet man diese Technik auf $ℚ$ mit der natürlichen Metrik an, dann erhält man $(ℝ,d)$ als vollständige Hülle.
+ Man beachte jedoch, dass dies nicht für die Konstruktion von $ℝ$ ausreicht, da hier schon die Existenz von $ℝ$ verwenden wird -- Aber das funktioniert größtenteils analog.
\end{bemerkung-nn}
\begin{satz}[Schachtelsatz]\label{schachtelsatz}
Sei $(X,d)$ ein vollständiger metrischer Raum und seien
- $(x_n)_{n * ℕ} ⊂ X$ und $(r_n)_{n ∈ ℕ} ⊂ (0,∞) $ Folgen mit der Eigenschaft
+ $(x_n)_{n * ℕ} ⊂ X$ und $(r_n)_{n ∈ ℕ} ⊂ (0,\infty ) $ Folgen mit der Eigenschaft
\begin{enumerate}
\item $\cl B_{r_{n+1}}(x_{n+1}) ⊂ B_{r_n} (x_n)$
- \item $\lim_{n \to ∞} r_n = 0$.
+ \item $\lim_{n \to \infty } r_n = 0$.
\end{enumerate}
- Dann gibt es genau ein $x_0 ∈ X$ mit $x_0 ∈ \bigcap_{n ∈ ℕ \cl B_{r_n} (x_n)}$.
+ Dann gibt es genau ein $x_0 ∈ X$ mit $x_0 ∈ \bigcap_{n ∈ ℕ} \cl B_{r_n} (x_n)$.
\end{satz}
\begin{proof}
@@ -625,14 +626,14 @@ jedoch lässt sich jeder metrische Raum zu einem vollständigen Erweitern.
\]
Also
\[
- d(x_{n+p},x_n) \le r_n \xrightarrow[n → ∞]{} 0.
+ d(x_{n+p},x_n) \le r_n \xrightarrow[n → \infty ]{} 0.
\]
Damit ist $(x_n){n ∈ ℕ}$ eine Cauchyfolge und damit konvergiert gegen ein $x_0 ∈ X$.
Außerdem gilt
\[
- d(xp,x_n) \le \underbrace{d(x_0, x_{n+p})}_{→ 0 (p → ∞)} + \underbrace{d(x_{n+p},x_n)}_{ \le r_n}.
+ d(x_p,x_n) \le \underbrace{d(x_0, x_{n+p})}_{→ 0 (p → \infty )} + \underbrace{d(x_{n+p},x_n)}_{ \le r_n}.
\]
- Damit folgt für $p → ∞$
+ Damit folgt für $p → \infty $
\[
d(x_0, x_n) \le r_n \quad ∀ n ∈ ℕ
\]
@@ -640,7 +641,7 @@ jedoch lässt sich jeder metrische Raum zu einem vollständigen Erweitern.
Für die Eindeutigkeit sei $\tilde x_0$ ebenfalls in $\bigcap_{n ∈ ℕ} \cl B_{r_n}(x_n)$.
Dann folgt
\[
- d(x_0,\tilde x_0) \le \underbrace{d(x_0,x_n)}_{\le r_n} + \underbrace{d(x_n, \tilde x_0)}_{\le r_n} \le 2r_n \xrightarrow[n → ∞]{} 0.
+ d(x_0,\tilde x_0) \le \underbrace{d(x_0,x_n)}_{\le r_n} + \underbrace{d(x_n, \tilde x_0)}_{\le r_n} \le 2r_n \xrightarrow[n → \infty ]{} 0.
\]
Doch damit war bereits $x_0 = \tilde x_0$.
\end{proof}
@@ -672,7 +673,7 @@ Der folgende Satz wird beim Beweis mehrerer fundamentaler Sätze benötigt, z.B
B_{r_2}(x_2) ⊂ B_{r_1/2} (x_1)
\]
und $B_{r_2}(x_2) ∩ M_2 = \emptyset$.
- Durch Fortsetzen dieses Schemas finden wir eine Folge $(x_n)_{n ∈ ℕ} ⊂ X$ und Radien $(r_n)_{n ∈ ℕ} ⊂ (0,∞)$ mit $r_n \le r/2^n \xrightarrow[n → ∞]{} 0$.
+ Durch Fortsetzen dieses Schemas finden wir eine Folge $(x_n)_{n ∈ ℕ} ⊂ X$ und Radien $(r_n)_{n ∈ ℕ} ⊂ (0,\infty )$ mit $r_n \le r/2^n \xrightarrow[n → \infty ]{} 0$.
Damit sind alle Voraussetzungen von \cref{schachtelsatz} erfüllt. Folglich existiert genau ein
\[
\tilde x ∈ \bigcap_{n ∈ ℕ} B_{r_n} (x_n) ⊂ B_r(x_0) ⊂ M.
@@ -711,14 +712,14 @@ Der folgende Satz wird beim Beweis mehrerer fundamentaler Sätze benötigt, z.B
% $\cl{B_{r_n}(x_n)} ⊂ U_n ∩ B_{r_{n-1}} (x_{n-1})$
% \end{enumerate}
% Dazu beachte man, dass $U_n ∩ B_{r_{n-1}}(x_{n-1})$ nichtleer und offen
-% ist, also existiert $x_n$ und $\frac 1 n > ε > 0$ mit $B_ε(x_n) ⊂ U_n ∩
-% B_{r_{n-1}} (x_{n-1})$ und $r_n = \frac ε 2$ ist wie gewünscht. Für $m
+% ist, also existiert $x_n$ und $\frac 1 n > \epsilon > 0$ mit $B_\epsilon (x_n) ⊂ U_n ∩
+% B_{r_{n-1}} (x_{n-1})$ und $r_n = \frac \epsilon 2$ ist wie gewünscht. Für $m
% \ge n$ impliziert (ii), dass $x_m ∈ B_{r_n}(x_n)$ und aus (i) folgt,
% dass die Folge $(x_n)_{n ∈ ℕ}$ damit eine Cauchyfolge ist. Damit
% konvergiert $(x_n)_{n ∈ ℕ}$ gegen ein $x ∈X$. Sei nun $N ∈ ℕ$ und $m >
% N$. Dann folgt aus $x_m ∈ B_{r_N}(x_N)$, dass
% \begin{align*}
-% x &= \lim_{m → ∞} x_m ∈ \cl{B_{r_N}(x_n)} ⊂ U_N ∩ B_{r_{N-1}}(x_{N-1}) \\
+% x &= \lim_{m → \infty } x_m ∈ \cl{B_{r_N}(x_n)} ⊂ U_N ∩ B_{r_{N-1}}(x_{N-1}) \\
% & ⊂ U_N ∩ B_{r_1}(x_1) ⊂ U_N ∩ W,
% \end{align*}
% also $x ∈ \bigcap_{n ∈ ℕ} U_N ∩ W$.
@@ -776,12 +777,12 @@ Erklärtes Ziel dieses Kapitels wird sein, die beiden Strukturen aus den vorheri
\section{Normierte Räume}
\begin{definition}
- Sei $X$ ein linearer Raum über $\K$. Die Abbildung $\norm\cdot: X → [0,∞)$
- heißt \emph{Norm} auf $X$, falls für alle $x, y ∈ X, α ∈ K$ gilt:
+ Sei $X$ ein linearer Raum über $\K$. Die Abbildung $\norm\cdot: X → [0,\infty )$
+ heißt \emph{Norm} auf $X$, falls für alle $x, y ∈ X, \alpha ∈ K$ gilt:
\begin{enumerate}
\item $\norm x = 0 \Longleftrightarrow x = 0$
\item
- $\norm{αx} = |α| \norm x$
+ $\norm{\alpha x} = |\alpha | \norm x$
\item
$\norm{x+y} \le \norm x + \norm y$
\end{enumerate}
@@ -789,7 +790,7 @@ Erklärtes Ziel dieses Kapitels wird sein, die beiden Strukturen aus den vorheri
\end{definition}
\begin{bemerkung}
- Durch $d(x,y) := \norm{x-y}$ wird ein normierter Raum auch ein metrischer, also insbesondere auch ein topologischer Raum.
+ Durch $d(x,y) \coloneq \norm{x-y}$ wird ein normierter Raum auch ein metrischer, also insbesondere auch ein topologischer Raum.
Diese induzierte Topologie auf $(X, \norm\cdot)$ heißt \emph{Normtopologie}.
Ohne die lineare Struktur macht der normierte Raum gar keinen Sinn, da für die Definition einiger der Normaxiome die Vektorraumoperationen verwendet werden.
@@ -798,35 +799,35 @@ Erklärtes Ziel dieses Kapitels wird sein, die beiden Strukturen aus den vorheri
\begin{beispiele}
\begin{enumerate}
\item
- Betrachte den $ℝ^n$ mit $\norm x _{p} := \left( \sum_{i=1}^n |x_i|^p \right)^{1/p}$ mit $1 \le p < ∞$ ist ein normierter Raum,
- genauso wie mit $\norm{x}_{∞} := \max_{1 \le i \le n} |x_i|$.
+ Betrachte den $ℝ^n$ mit $\norm x _{p} \coloneq \left( \sum_{i=1}^n |x_i|^p \right)^{1/p}$ mit $1 \le p < \infty $ ist ein normierter Raum,
+ genauso wie mit $\norm{x}_{\infty } \coloneq \max_{1 \le i \le n} |x_i|$.
Insbesondere gibt es im $ℝ^n$ überabzählbar viele verschiedene Normen.
Wir werden jedoch später sehen, dass diese Normen alle die gleiche Topologie erzeugen.
\item
- Der Raum aller stetigen Funktionen auf einem kompaktem Intervall $C[a,b]$ mit $\norm{x}_{∞} := \max_{t ∈ [a,b]} |x(t)|$ ist ein normierter Raum.
+ Der Raum aller stetigen Funktionen auf einem kompaktem Intervall $C[a,b]$ mit $\norm{x}_{\infty } \coloneq \max_{t ∈ [a,b]} |x(t)|$ ist ein normierter Raum.
Außerdem wird durch
\[
- \norm x := ∫_a^b |x(t)| dt
+ \norm x \coloneq ∫_a^b |x(t)| dt
\]
ebenfalls eine Norm definiert.
\item
Sei $\Omega ⊂ ℝ^n$ offen und beschränkt. Dann wird $C(\cl{\Omega})$ mit
\[
- \norm{x}_{∞} := \max_{t ∈ \cl \Omega} |x(t)|
+ \norm{x}_{\infty } \coloneq \max_{t ∈ \cl \Omega} |x(t)|
\]
auch zu einem normierten Raum.
\item
$L^p(\Omega) = \L^p(\Omega)/\mathcal N$, wobei $\mathcal N = \{ f: \Omega → \R, f(t) = 0 \text{ fast überall}\}$ ist mit
\[
- \norm x := \left(∫_{\Omega} |x(t)|^p dt \right)^{1/p}
+ \norm x \coloneq \left(∫_{\Omega} |x(t)|^p dt \right)^{1/p}
\]
- ein normierter Raum, wobei $1 \le p < ∞$.
+ ein normierter Raum, wobei $1 \le p < \infty $.
\item
$\ell^p$ mit
\[
- \norm x _{p} := \left( \sum_{i=1}^n |x_i|^p \right)^{1/p}
+ \norm x _{p} \coloneq \left( \sum_{i=1}^n |x_i|^p \right)^{1/p}
\]
- ist ebenfalls ein normierter Raum, wobei $1 \le p < ∞$.
+ ist ebenfalls ein normierter Raum, wobei $1 \le p < \infty $.
\end{enumerate}
\end{beispiele}
@@ -834,13 +835,13 @@ Erklärtes Ziel dieses Kapitels wird sein, die beiden Strukturen aus den vorheri
Sei $(X, \norm\cdot)$ ein normierter Raum. Dann sind die Abbildungen $+$, $\cdot$ und $\norm\cdot$ stetig.
\end{lemma}
\begin{proof}
- Für beliebige Folgen $(x_n)_{n ∈ ℕ},(y_n)_{n ∈ ℕ} ⊂ X, (α_n)_{n ∈ ℕ}$ mit $\lim x_n = x$, $\lim y_n = y$, $\lim α_n = α$ gelten
+ Für beliebige Folgen $(x_n)_{n ∈ ℕ},(y_n)_{n ∈ ℕ} ⊂ X, (\alpha _n)_{n ∈ ℕ}$ mit $\lim x_n = x$, $\lim y_n = y$, $\lim \alpha _n = \alpha $ gelten
\[
\norm{(x_n + y_n) - (x+y)} \le \norm{x-x_n} + \norm{y -y_n}
\]
sowie
\[
- \norm{α_nx_n - αx} \le |α_n| \norm{x_n-x} + \norm{x} |α_n - α|
+ \norm{\alpha _nx_n - \alpha x} \le |\alpha _n| \norm{x_n-x} + \norm{x} |\alpha _n - \alpha |
\]
und
\[
@@ -858,12 +859,12 @@ Erklärtes Ziel dieses Kapitels wird sein, die beiden Strukturen aus den vorheri
\section{Topologische lineare Räume}
\begin{bemerkung-nn}
Hierbei sei stetis die Topologie von $X×X$ die Produktopologie, bei den Körpern $\K = \begin{cases} ℝ \\ ℂ \end{cases}$ die übliche Topologie.
- Wir schreiben im Folgenden für Mengen $M, M_1, M_2 ⊂ X$ und $α ⊂ \K$ nun
+ Wir schreiben im Folgenden für Mengen $M, M_1, M_2 ⊂ X$ und $\alpha ⊂ \K$ nun
\[
- M_1 + M_2 := s(M_1,M_2) := \{x+y : x ∈ M_1, y ∈ M_2\},
+ M_1 + M_2 \coloneq s(M_1,M_2) \coloneq \{x+y : x ∈ M_1, y ∈ M_2\},
\]
\[
- A \cdot M := m(A,M) := \{ αx: α ∈ A, x ∈ M\}.
+ A \cdot M \coloneq m(A,M) \coloneq \{ \alpha x: \alpha ∈ A, x ∈ M\}.
\]
\end{bemerkung-nn}
@@ -890,11 +891,11 @@ Erklärtes Ziel dieses Kapitels wird sein, die beiden Strukturen aus den vorheri
\begin{enumerate}
\item Die Addition $m$ ist stetig.
\item
- Für beliebiges $α ∈ \K, x ∈ X$ gilt: Zu jeder Umgebung $O_{αx} ∈ \T$ existieren Umgebungen $O_x ∈ \T$ von $x$ und $O_α ∈ \T$ von $y$ mit $O_α × O_x ⊂ O_{αx}$.
+ Für beliebiges $\alpha ∈ \K, x ∈ X$ gilt: Zu jeder Umgebung $O_{\alpha x} ∈ \T$ existieren Umgebungen $O_x ∈ \T$ von $x$ und $O_\alpha ∈ \T$ von $y$ mit $O_\alpha × O_x ⊂ O_{\alpha x}$.
\end{enumerate}
\end{lemma}
-Betrachtet man insbesondere die Stetigkeit am Punke $α=0$ und $x ∈ X$ beliebig, dann gilt also:
+Betrachtet man insbesondere die Stetigkeit am Punke $\alpha =0$ und $x ∈ X$ beliebig, dann gilt also:
Für jede Umgebung $O_0 ∈ \U_0 ⊂ X$ existiert eine Umgebung $O_x ∈ \U_x$ und ein $r > 0$, so dass
\[
∀β: |β| <r: βO_x ⊂ O_0.
@@ -903,7 +904,7 @@ Unmittelbar daraus erhalten wir folgendes Korollar:
\begin{korollar}
Im topologischen Raum $(X,\T)$ gilt für $x ∈ X$ beliebig und $(β_n)_{n ∈ ℕ} ⊂ ℝ$
\[
- β_n \xrightarrow[n → ∞]{} 0 \implies β_nx \xrightarrow[n → ∞]{} 0.
+ β_n \xrightarrow[n → \infty ]{} 0 \implies β_nx \xrightarrow[n → \infty ]{} 0.
\]
\end{korollar}
@@ -912,12 +913,12 @@ Unmittelbar daraus erhalten wir folgendes Korollar:
\item
Zu $x_0 ∈ X$ fest definieren wir den Translationsoperator
\[
- T_{x_0} := X → X, x ↦ x + x_0.
+ T_{x_0} \coloneq X → X, x ↦ x + x_0.
\]
\item
- Zu $α_0 ∈ \K^*$ fest definieren wir den Multiplikationsoperator
+ Zu $\alpha _0 ∈ \K^*$ fest definieren wir den Multiplikationsoperator
\[
- M_{α_0} := X → X, x ↦ α_0\cdot x.
+ M_{\alpha _0} \coloneq X → X, x ↦ \alpha _0\cdot x.
\]
\end{enumerate}
\end{definition-nn}
@@ -964,36 +965,36 @@ k \]
Es genügt, da in metrischen Räumen Folgenstetigkeit und Stetigkeit äquivalent sind, zu zeigen, dass $\lim d(x_n + y_n, x + y ) = 0$, sofern $\lim d(x_n,x) = 0$ und $\lim d(y_n,y) = 0$.
Dazu ist
\[
- d(x_n+y_n,x+y) \le d(x_n+y_n) + d(x+y_n, x+y) = d(x_n,x) +d(y_n,y) \xrightarrow[n → ∞]{} 0.
+ d(x_n+y_n,x+y) \le d(x_n+y_n) + d(x+y_n, x+y) = d(x_n,x) +d(y_n,y) \xrightarrow[n → \infty ]{} 0.
\]
\end{proof}
\begin{beispiel-nn}
Sei $X = C(a,b)$ mit der Metrik
\[
- d(x,y) := \min\{ 1, \sum_{t ∈ (a,b)} |x(t)-y(t)|\}.
+ d(x,y) \coloneq \min\{ 1, \sup_{t ∈ (a,b)} |x(t)-y(t)|\}.
\]
Dann ist $d$ eine translationsinvariante Metrik, aber $X$ ist kein linearer Raum, da die Skalarmultiplikation nicht stetig ist.
\end{beispiel-nn}
-Für die Stetigkeit der skalaren Multiplikation im Punkt $(α,x) ∈ \K × X$ hat man (nach dem $ε-δ-Kriterium$)
+Für die Stetigkeit der skalaren Multiplikation im Punkt $(\alpha ,x) ∈ \K × X$ hat man (nach dem $\epsilon -\delta -Kriterium$)
\[
- ∀ε > 0 ∃ δ > 0 ∃ r> 0 ∀β ∈ \K ∀y ∈ X:
+ ∀\epsilon > 0 ∃ \delta > 0 ∃ r> 0 ∀β ∈ \K ∀y ∈ X:
\begin{rcases}
- |β - α| < r \\
- d(x,y) < δ
+ |β - \alpha | < r \\
+ d(x,y) < \delta
\end{rcases}
- \implies d(βy,αx) < ε
+ \implies d(βy,\alpha x) < \epsilon
\]
\begin{lemma}
\label{lemma-metrischer-linearer-raum-charak}
Sei $(X,d)$ ein metrischer Raum mit linearer Struktur und mit einer translationinvarianten Metrik.
- Dann ist $X$ mit der von $d$ erzeugten Topologie ein \emph{metrischer linearer Raum} genau dann, wenn für alle $α ∈ \K, x ∈ X$ und beliebige Nullfolgen $(x_n)_{n ∈ ℕ} ⊂ X, (α_n)_{n ∈ ℕ) ⊂ \K}$ gilt
+ Dann ist $X$ mit der von $d$ erzeugten Topologie ein \emph{metrischer linearer Raum} genau dann, wenn für alle $\alpha ∈ \K, x ∈ X$ und beliebige Nullfolgen $(x_n)_{n ∈ ℕ} ⊂ X, (\alpha _n)_{n ∈ ℕ)} ⊂ \K$ gilt
\begin{gather*}
- αx_n \xrightarrow[n → ∞]{} 0 \\
- αx_n \xrightarrow[n → ∞]{} 0 \\
- α_nx_n \xrightarrow[n → ∞]{} 0
+ \alpha x_n \xrightarrow[n → \infty ]{} 0 \\
+ \alpha x_n \xrightarrow[n → \infty ]{} 0 \\
+ \alpha _nx_n \xrightarrow[n → \infty ]{} 0
\end{gather*}
\end{lemma}
\begin{proof}
@@ -1002,28 +1003,29 @@ Für die Stetigkeit der skalaren Multiplikation im Punkt $(α,x) ∈ \K × X$ ha
„$⇐$“: Wegen der Äquivalenz von Stetigkeit und Folgenstetigkeit ist zu zeigen
\[
\begin{rcases}
- α_n \xrightarrow[n → ∞]{} α ∈ \K \\
- x_n \xrightarrow[n → ∞]{} x ∈ X
+ \alpha _n \xrightarrow[n → \infty ]{} \alpha ∈ \K \\
+ x_n \xrightarrow[n → \infty ]{} x ∈ X
\end{rcases}
- \implies α_n x_n \xrightarrow[n → ∞]{} αx.
+ \implies \alpha _n x_n \xrightarrow[n → \infty ]{} \alpha x.
\]
- Sei dazu $z_n := x_n - x ∈ X$, $γ_n := α_n - α ∈ \K$. Dann ist
+ Sei dazu $z_n \coloneq x_n - x ∈ X$, $γ_n \coloneq \alpha _n - \alpha ∈ \K$. Dann ist
\[
- γ_n z_n + γ_n x + α z_n = (α_n - α)(x_n-x) + (α_n-α) x + α(x_n-x)
- = α_n x_n - α×.
+ γ_n z_n + γ_n x + \alpha z_n = (\alpha _n - \alpha )(x_n-x) + (\alpha _n-\alpha ) x + \alpha (x_n-x)
+ = \alpha _n x_n - \alpha ×.
\]
Somit ist
\begin{align*}
- d(α_nx_n,αx) &= d(αnx_n - αx,0) = d(γ_nz_n + γnx + αz_n, 0) \\
- &\le \underbrace{d(γ_nz_n,0)}_{→ 0} + \underbrace{d(γ_nx, 0)}_{→ 0} + \underbrace{d(αz_n, 0)}_{→ 0} \xrightarrow{n → 0} 0.
+ d(\alpha _nx_n,\alpha x) &= d(\alpha nx_n - \alpha x,0) = d(γ_nz_n + γnx + \alpha z_n, 0) \\
+ &\le \underbrace{d(γ_nz_n,0)}_{→ 0} + \underbrace{d(γ_nx, 0)}_{→ 0} +
+\underbrace{d(\alpha z_n, 0)}_{→ 0} \xrightarrow{n → \infty } 0.
\end{align*}
Da die Addition ohnehin immer stetig ist, sind wir fertig.
\end{proof}
\begin{definition}
- Eine Abbildung $|\cdot|: X → [0,∞)$ heißt \emph{Quasi-Norm} auf dem Linearen
+ Eine Abbildung $|\cdot|: X → [0,\infty )$ heißt \emph{Quasi-Norm} auf dem Linearen
Raum $X$, falls gilt:
\begin{enumerate}[label=(Q\arabic*)]
\item
@@ -1033,11 +1035,11 @@ Für die Stetigkeit der skalaren Multiplikation im Punkt $(α,x) ∈ \K × X$ ha
\item
$|x+y| \le |x| + |y|$ für alle $x,y ∈ X$
\item
- $|αx_n| \xrightarrow[n → ∞]{} 0$ für $α ∈ \K$, falls $|x_n| → 0$
+ $|\alpha x_n| \xrightarrow[n → \infty ]{} 0$ für $\alpha ∈ \K$, falls $|x_n| → 0$
\item
- $|α_nx| \xrightarrow[n → ∞]{} 0$ für $x ∈ X$, falls $|α_n| → 0$
+ $|\alpha _nx| \xrightarrow[n → \infty ]{} 0$ für $x ∈ X$, falls $|\alpha _n| → 0$
\item
- $|α_nx_n| \xrightarrow[n → ∞]{} 0$ falls $|x_n| → 0$ und $|α_nx_n| → 0$
+ $|\alpha _nx_n| \xrightarrow[n → \infty ]{} 0$ falls $|x_n| → 0$ und $|\alpha _nx_n| → 0$
\end{enumerate}
$(X,|\cdot|)$ heißt dann \emph{quasi-normierter} Raum.
\end{definition}
@@ -1049,10 +1051,10 @@ Für die Stetigkeit der skalaren Multiplikation im Punkt $(α,x) ∈ \K × X$ ha
\begin{satz}
\begin{enumerate}
\item
- Ist $|\cdot|$ eine Quasi-Norm auf $X$, so wird durch $d(x,y) := |x-y|$ eine translationsinvariante Metrik definiert, welche $X$ zu einem metrischen linearen Raum macht.
+ Ist $|\cdot|$ eine Quasi-Norm auf $X$, so wird durch $d(x,y) \coloneq |x-y|$ eine translationsinvariante Metrik definiert, welche $X$ zu einem metrischen linearen Raum macht.
\item
Ist $(X,d)$ ein metrischer linearer Raum mit translationsinvarianter Metrik $d$, so ist
- $(X,|\cdot|)$ mit $|x| := d(x,0)$ ein quasi-normierter Raum.
+ $(X,|\cdot|)$ mit $|x| \coloneq d(x,0)$ ein quasi-normierter Raum.
\end{enumerate}
\end{satz}
\begin{proof}
@@ -1067,11 +1069,11 @@ Speziell für die Anwendung sehr wichtige metrische lineare Räume werden von Se
Eine Abbildung $p: X → ℝ$ heißt \emph{Semi-Norm} oder \emph{Halbnorm}, falls folgendes gilt:
\begin{enumerate}[label=(S\arabic*)]
\item
- $∀x ∈ X: p(x) ≥ 0$
+ $∀x ∈ X: p(x) \ge 0$
\item
- $∀ x ∈ X, α ∈ \K: p(αx) = |α| p(x)$
+ $∀ x ∈ X, \alpha ∈ \K: p(\alpha x) = |\alpha | p(x)$
\item
- $∀ x, y ∈ X: p(x+y) ≤ p(x) + p(y)$
+ $∀ x, y ∈ X: p(x+y) \le p(x) + p(y)$
\end{enumerate}
$(X,p)$ heißt dann \emph{semi-normierter} Raum.
\end{definition}
@@ -1092,7 +1094,7 @@ Speziell für die Anwendung sehr wichtige metrische lineare Räume werden von Se
\end{equation}
Dann ist
\[
- d(x,yr) := \sum_{n = 1}^∞ 2^{-n} \frac{p_n(x-y)}{1+p_n(x-y)}
+ d(x,y) \coloneq \sum_{n = 1}^\infty 2^{-n} \frac{p_n(x-y)}{1+p_n(x-y)}
\]
eine translationsinvariante Metrik auf $X$, welche $X$ zum metrischen linearen Raum macht.
\end{satz}
@@ -1108,24 +1110,24 @@ Speziell für die Anwendung sehr wichtige metrische lineare Räume werden von Se
\begin{satz}
\label{satz-umgebungsbasis-produkt-von-seminorm}
Sei $(X,d)$ der in \cref{satz-abzaehlbares-prod-seminormierter-raeume} gegebene metrische lineare Raum (mit der von der Metrik erzeugten Topologie).
- Dann bilden die Mengen ($ε_n > 0$)
+ Dann bilden die Mengen ($\epsilon _n > 0$)
\[
- U (p_n,ε_n) := \bigcup B^{p_n}_{ε_n}(0)
- = \{ x ∈ X: p_n(x) < ε_n\}
+ U (p_n,\epsilon _n) \coloneq \bigcup B^{p_n}_{\epsilon _n}(0)
+ = \{ x ∈ X: p_n(x) < \epsilon _n\}
\]
und deren endliche Durchschnitte eine Umgebungsbasis von $0 ∈ X$
\end{satz}
\begin{bemerkung-nn}
- Nach dem Invarianzprinzip ist damit durch $\bigcup B^{p_n}_{ε_n}$ die ganze Topologie bestimmt.
+ Nach dem Invarianzprinzip ist damit durch $\bigcup B^{p_n}_{\epsilon _n}$ die ganze Topologie bestimmt.
Mit anderen Worten: Die Topologie welche über die Metrik bestimmt ist, ist dieselbe wie die, welche von den
- $U(p_n,ε_n)$ und endlichen Schnitten davon erzeugt wird.
+ $U(p_n,\epsilon _n)$ und endlichen Schnitten davon erzeugt wird.
\end{bemerkung-nn}
\begin{proof}[\cref{satz-umgebungsbasis-produkt-von-seminorm}]
- Zunächst ist $U (p_n,ε_n) ∈ \T$:
- Sei $n ∈ ℕ$ und $ε_n > 0$ fest und $y ∈ U(p_n,ε_n)$ beliebig gegeben.
- Dann ist $p_n(y) < ε_n$. Dann wähle $ρ = ρ(y) > 0$, so dass $p_n(y) + ρ < ε_n$.
- Dann gilt für $r := 2^{-n} \frac{ρ}{1+ρ} > 0$:
+ Zunächst ist $U (p_n,\epsilon _n) ∈ \T$:
+ Sei $n ∈ ℕ$ und $\epsilon _n > 0$ fest und $y ∈ U(p_n,\epsilon _n)$ beliebig gegeben.
+ Dann ist $p_n(y) < \epsilon _n$. Dann wähle $ρ = ρ(y) > 0$, so dass $p_n(y) + ρ < \epsilon _n$.
+ Dann gilt für $r \coloneq 2^{-n} \frac{ρ}{1+ρ} > 0$:
\[
x ∈ B_r(y) \implies p_n(x+r) < ρ.
\]
@@ -1133,10 +1135,10 @@ Speziell für die Anwendung sehr wichtige metrische lineare Räume werden von Se
\[
\frac{p_n(x-y)}{1+p_n(x-y)} \le 2^n \underbrace{d(x,y)}_{< r} < 2^n r = \frac{ρ}{1+ρ},
\]
- also $p_n(x-y) < ρ$. Mit diesem $r$ gilt $B_r(y) ⊂ U(p_n,ε_n)$:
+ also $p_n(x-y) < ρ$. Mit diesem $r$ gilt $B_r(y) ⊂ U(p_n,\epsilon _n)$:
Sei $x ∈ B_r(y)$. Dann gilt
\[
- p_n(x) \le \underbrace{p_n(x-y)}_{< ρ} + p_n(y) < p_n(y) + ρ = ε_n
+ p_n(x) \le \underbrace{p_n(x-y)}_{< ρ} + p_n(y) < p_n(y) + ρ = \epsilon _n
\]
wie gewünscht.
@@ -1144,31 +1146,31 @@ Speziell für die Anwendung sehr wichtige metrische lineare Räume werden von Se
Sei $ B_r(0), r > 0$ gegeben.
Wähle $n_0 ∈ ℕ$ mit
\[
- \sum_{n=n_0}^∞ 2^{-n} < \frac r 2.
+ \sum_{n=n_0}^\infty 2^{-n} < \frac r 2.
\]
- mit $ε := \frac r 2 $ gilt dann
+ mit $\epsilon \coloneq \frac r 2 $ gilt dann
\[
- \bigcap_{n=1}^{n_0} U(p_(,ε) ⊂ B_r(0).
+ \bigcap_{n=1}^{n_0} U(p_(,\epsilon ) ⊂ B_r(0).
\]
- Sei dazu $x ∈ \bigcap_{n=1}^{n_0} U(p_n,ε)$ beliebig.
+ Sei dazu $x ∈ \bigcap_{n=1}^{n_0} U(p_n,\epsilon )$ beliebig.
Dann ist
\[
- d(x,0) \le \sum_{n=1}^{n_0} 2^{-n} \frac{p_n(x)}{1+p_n(x)} + \sum_{n=n_0}^∞ 2^{-n} < ε \sum_{n=1}^{n_0} 2^{-n} + \frac r 2 < ε + \frac r 2 = r,
+ d(x,0) \le \sum_{n=1}^{n_0} 2^{-n} \frac{p_n(x)}{1+p_n(x)} + \sum_{n=n_0}^\infty 2^{-n} < \epsilon \sum_{n=1}^{n_0} 2^{-n} + \frac r 2 < \epsilon + \frac r 2 = r,
\]
somit also $x ∈ B_r(0)$.
\end{proof}
\begin{bemerkung}
- Die Mengen $U(p_n,ε_n)$ und deren endlichen Schnitte sind konvexe Mengen, das heißt
+ Die Mengen $U(p_n,\epsilon _n)$ und deren endlichen Schnitte sind konvexe Mengen, das heißt
\[
- x, y ∈ U(p_n,ε_n),α ∈ [0,1] \implies αx+(1-α)y ∈ U(p_n,ε_n)
+ x, y ∈ U(p_n,\epsilon _n),\alpha ∈ [0,1] \implies \alpha x+(1-\alpha )y ∈ U(p_n,\epsilon _n)
\]
\end{bemerkung}
\begin{proof}
Es ist
\[
- p_n(αx + (1-α)y) \le |α| \underbrace{p_n(x)}_{< ε_n} + |1-α|\underbrace{p_n(y)}_{< ε_n} = ε_n.
+ p_n(\alpha x + (1-\alpha )y) \le |\alpha | \underbrace{p_n(x)}_{< \epsilon _n} + |1-\alpha |\underbrace{p_n(y)}_{< \epsilon _n} = \epsilon _n.
\]
\end{proof}
@@ -1185,10 +1187,10 @@ Also besitzt der in \cref{satz-abzaehlbares-prod-seminormierter-raeume} gewonne
\]
Dann sind die Mengen
\[
- U(p_i,ε_i) = \{ x ∈ X: p_{(x) < ε_i}\}, \quad ε_i > 0, i ∈ I
+ U(p_i,\epsilon _i) = \{ x ∈ X: p_{(x) < \epsilon _i}\}, \quad \epsilon _i > 0, i ∈ I
\]
und deren endliche Schnitte eine konvexe Umgebungsbasis von $0 ∈ X$.
- Die dadurch gewonne Topologie $\T$ macht $X$ zu einem \emph{lokalkonvexen Hausdorff-Raum}.
+ Die dadurch gewonne Topologie $\T$ macht $X$ zu einem \emph{lokalkonvexen Hausdorff"=Raum}.
\end{satz}
\section{Beispiele}
@@ -1197,9 +1199,9 @@ Wir werden die unten angegebenen Beispiele auch gleich auf Vollständigkeit unte
\begin{definition}
\begin{enumerate}
\item
- Ein metrischer linearer Raum $(X,d)$ der vollständig ist, heißt \emph{Fréchet-Raum}.
+ Ein metrischer linearer Raum $(X,d)$ der vollständig ist, heißt \emph{Fréchet"=Raum}.
\item
- Ein normierter Raum $(X,\norm\cdot)$, der vollständig ist, heißt \emph{Banach-Raum}.
+ Ein normierter Raum $(X,\norm\cdot)$, der vollständig ist, heißt \emph{Banach"=Raum}.
\end{enumerate}
\end{definition}
@@ -1207,119 +1209,122 @@ Wir werden die unten angegebenen Beispiele auch gleich auf Vollständigkeit unte
\begin{beispiel-nn}[$\ell^p$-Räume]
\begin{enumerate}
\item
- $(\ell^p,\norm\cdot_p)$, $1 \le p < ∞$ ist normierter Raum mit
+ $(\ell^p,\norm\cdot_p)$, $1 \le p < \infty $ ist normierter Raum mit
\[
- \norm x _p = \left( \sum_{i=1}^∞ |x_i|^p \right)^{1/p}.
+ \norm x _p = \left( \sum_{i=1}^\infty |x_i|^p \right)^{1/p}.
\]
\item
- $(\ell^∞,\norm\cdot_∝)$, ist normierter Raum mit $\norm x _∞ = \sup_{i ∈ ℕ} |x_i|$.
+ $(\ell^\infty ,\norm\cdot_\infty)$, ist normierter Raum mit $\norm x _\infty = \sup_{i ∈ ℕ} |x_i|$.
\item
$(\ell^p,|\cdot|_p = \norm\cdot_p^p)$, $0 \le p < 1$ ist quasi-normierter Raum.
\end{enumerate}
\end{beispiel-nn}
\begin{bemerkung}
- Für $0 < p < q \le ∞$ gilt $\ell^p ⊂ \ell^q ⊂ \ell^∞$.
+ Für $0 < p < q \le \infty $ gilt $\ell^p ⊂ \ell^q ⊂ \ell^\infty $.
\end{bemerkung}
\begin{beweis}
Sei $x ∈ \ell^p$ mit $|x| = 1 = \sum_{i ∈ ℕ} |x_i|^p$.
Dann ist für alle $i ∈ ℕ$ $|x_i|^p \le 1$, also auch $|x_i| < 1$.
- Dann folgt auch $\sum_{i ∈ ℕ} |x_i|^q < 1$, also $x ∈ \ell^q$ und $\sup_{i ∈ ℕ} |x_i| ≤ 1$, also $x ∈ \ell^∞$.
+ Dann folgt auch $\sum_{i ∈ ℕ} |x_i|^q < 1$, also $x ∈ \ell^q$ und $\sup_{i ∈ ℕ} |x_i| \le 1$, also $x ∈ \ell^\infty $.
\end{beweis}
\begin{satz}
- Für $1 \le p \le ∞$ ist $(\ell^p,\norm\cdot_p)$ ein Banachraum.
- Für $0 < p < ∞$ ist $(\ell^p,|\cdot|_p)$ ein Fréchet-Raum.
+ Für $1 \le p \le \infty $ ist $(\ell^p,\norm\cdot_p)$ ein Banachraum.
+ Für $0 < p < \infty $ ist $(\ell^p,|\cdot|_p)$ ein Fréchet-Raum.
\end{satz}
\begin{proof}
- Nur für $1 \le p < ∞$.
+ Nur für $1 \le p < \infty $.
Sei dazu $(x_n)_{n ∈ ℕ} ⊂ \ell^p$ eine Cauchy-Folge, also
- $x_n = (ξ_k^n)_{k ∈ ℕ}$ und für jedes $ε > 0$ gibt es ein $n_0$ mit
+ $x_n = (ξ_k^n)_{k ∈ ℕ}$ und für jedes $\epsilon > 0$ gibt es ein $n_0$ mit
\[
- ∀n,m > n_0: \norm{x_n-xm}_p = \left( \sum_{k=1}^∞ |ξ_k^n-ξ_k^m|^p \right)^{1/p} < ε.
+ ∀n,m > n_0: \norm{x_n-x_m}_p = \left( \sum_{k=1}^\infty |ξ_k^n-ξ_k^m|^p \right)^{1/p} < \epsilon .
\]
Sei $k_0 ∈ ℕ$ beliebig. Dann ist $(ξ_k^n)_{n ∈ ℕ}$
eine Cauchy-Folge in $\K$, besitzt also einen Grenzwert $ξ_{k_0}$.
- Setze nun $x := (ξ_k)_{k ∈ ℕ} ∈ \K^∞ = s$. Wir vermuten $x$ als Grenzwert unserer Cauchy-Folge.
+ Setze nun $x \coloneq (ξ_k)_{k ∈ ℕ} ∈ \K^\infty = s$. Wir vermuten $x$ als Grenzwert unserer Cauchy-Folge.
Also müssen wir zeigen, dass $x ∈ \ell^p$, und dass unsere Folge tatsächlich gegen $x$ konvergiert.
Es gilt
\[
- \norm{x_n}_! \le \underbrace{\norm{x_n-x_{n_0}}}_{< ε} + \norm{x_{n_0}} \quad \forall n \ge n_0
+ \norm{x_n}_! \le \underbrace{\norm{x_n-x_{n_0}}}_{< \epsilon } + \norm{x_{n_0}} \quad \forall n \ge n_0
\]
Deshalb existiert ein $M > 0$ mit $\norm{x_n}_p < M$ für alle $n ∈ ℕ$, also
\[
- \sum_{k=1}^N |ξ_k^n|p < \sum_{k =1}^∞ |ξ_k^n|^p \le M^p < ∞.
+ \sum_{k=1}^N |ξ_k^n|p < \sum_{k =1}^\infty |ξ_k^n|^p \le M^p < \infty .
\]
Also haben wir
\[
\sum_{k=1}^N |ξ_k^p| \le M^p \quad ∀ n ∈ ℕ,
\]
- also durch Grenzwertbildung $N → ∞$ auch $\norm{x}_p^p ≤ M^p$ bzw. $x ∈ \ell^p$.
+ also durch Grenzwertbildung $N → \infty $ auch $\norm{x}_p^p \le M^p$ bzw. $x ∈ \ell^p$.
Ferner haben wir
\[
- \sum_{k=1}^N |ξ_k^n-ξ_k^m|^p < ε^p \quad ∀ N ∈ ℕ, n, m ≥ n_0(ε).
+ \sum_{k=1}^N |ξ_k^n-ξ_k^m|^p < \epsilon ^p \quad ∀ N ∈ ℕ, n, m \ge n_0(\epsilon ).
\]
- Für $n → ∞$ folgt
+ Für $n → \infty $ folgt
\[
- \sum_{k=1}^N |ξ_k-ξ_k^m|^p < ε^p \quad ∀N ∈ ℕ, m ≥ n_0,
+ \sum_{k=1}^N |ξ_k-ξ_k^m|^p < \epsilon ^p \quad ∀N ∈ ℕ, m \ge n_0,
\]
- und mit $N → ∞$
+ und mit $N → \infty $
\[
- \sum_{k=1}^∞ |ξ_k-ξ_k^m|^p < ε^p \quad ∀m ≥ n_0,
+ \sum_{k=1}^\infty |ξ_k-ξ_k^m|^p < \epsilon ^p \quad ∀m \ge n_0,
\]
also die Konvergenz.
\end{proof}
\begin{beispiel-nn}
- Betrachte den Folgenraum $S = \K^∞ = \{x = (ξ_n)_{n ∈ ℕ}, ξ_n ∈ \K\}$.
+ Betrachte den Folgenraum $S = \K^\infty = \{x = (ξ_n)_{n ∈ ℕ}, ξ_n ∈ \K\}$.
Dann ist
\[
- p_n(x) := |ξ_n|, \quad p_n: \K^∞ → ℝ
+ p_n(x) \coloneq |ξ_n|, \quad p_n: \K^\infty → ℝ
\]
eine abzählbare Familie von Halbnormen mit
\[
- p_n(x) = 0 ∀n ∈ ℕ \implies x = 0 ∈ \K^∞
+ p_n(x) = 0 ∀n ∈ ℕ \implies x = 0 ∈ \K^\infty
\]
- Nach \cref{satz-abzaehlbares-prod-seminormierter-raeume} folgt, dass $(\K^∞, d)$ mit
+ Nach \cref{satz-abzaehlbares-prod-seminormierter-raeume} folgt, dass $(\K^\infty , d)$ mit
\[
- d(x,y) := \sum_{n ∈ ℕ} 2^{-n} \frac{p_n(x-y)}{1+p_n(x-y)}
+ d(x,y) \coloneq \sum_{n ∈ ℕ} 2^{-n} \frac{p_n(x-y)}{1+p_n(x-y)}
\]
ein metrischer linearer Raum ist.
Der Konvergenzbegriff entspricht gerade der komponentenweisen Konvergenz, das heißt, für eine Folge $(x_k)_{k ∈ ℕ}$ mit $x_k = (ξ^k_n)_{n ∈ ℕ}$ gilt
- \[
- x_k \xrightarrow[k→∞]{} 0 \; \Longleftrightarrow \; d(x_n,0) \xrightarrow[k→∞]{} 0 \; \Longleftrightarrow \; p_n(x_k) \xrightarrow[k→∞]{} ∀ n ∈ ℕ \; \Longleftrightarrow \; |ξ_n^k| \xrightarrow[k→∞] 0 ∀ n ∈ ℕ.
- \]
+ \begin{align*}
+ x_k \xrightarrow[k→\infty ]{} 0
+ &\gdw d(x_n,0) \xrightarrow[k→\infty ]{} 0 \\
+ &\gdw p_n(x_k) \xrightarrow[k→\infty ]{} ∀ n ∈ ℕ \\
+ &\gdw |ξ_n^k| \xrightarrow[k→\infty ]{} 0 ∀ n ∈ ℕ.
+ \end{align*}
- Wir fragen uns nun, ob auf dem $\K^∞$ auch eine Topologie existiert, so dass der induzierte Konvergenzbegriff der der gleichmäßigen Konvergenz in allen Komponenten entspricht?
+ Wir fragen uns nun, ob auf dem $\K^\infty $ auch eine Topologie existiert, so dass der induzierte Konvergenzbegriff der der gleichmäßigen Konvergenz in allen Komponenten entspricht.
Also
\[
- x_k \xrightarrow[k → ∞]{\text{glm}} 0 ∈ \K^∞ \gdw ∀ε > 0 ∃ k_0 ∈ ℕ: |ξ_n^k| < ε ∀ k \ge k_0 ∀n ∈ ℕ.
+ x_k \xrightarrow[k → \infty ]{\text{glm}} 0 ∈ \K^\infty \gdw ∀\epsilon > 0 ∃ k_0 ∈ ℕ: |ξ_n^k| < \epsilon ∀ k \ge k_0 ∀n ∈ ℕ.
\]
- Wenn $\K^∞$ ein topologischer linearer Raum sein soll, ist das nicht möglich. Notwendig wäre, dass für eine Folge $x  ∈ \K^∞$
+ Wenn $\K^\infty $ ein topologischer linearer Raum sein soll, ist das nicht möglich. Notwendig wäre, dass für eine Folge $x  ∈ \K^\infty $
\[
- α_k \xrightarrow[k → ∞]{} 0 \text{ in } \K \implies α_k x \xrightarrow[k→∞]{} \text{ in } X = \K^∞.
+ \alpha _k \xrightarrow[k → \infty ]{} 0 \text{ in } \K \implies \alpha _k x \xrightarrow[k→\infty ]{} \text{ in } X = \K^\infty .
\]
- Wähle dazu die Nullfolge $(α_k)_{k ∈ ℕ} = (1/k)_{k ∈ ℕ} ⊂ ℝ$, $x= (n)_{n ∈ ℕ} ⊂ X$. Dann ist
+ Wähle dazu die Nullfolge $(\alpha _k)_{k ∈ ℕ} = (1/k)_{k ∈ ℕ} ⊂ ℝ$, $x= (n)_{n ∈ ℕ} ⊂ X$. Dann ist
\[
- α_k x = (n/k)_{n ∈ ℕ} ∈ \K^∞
+ \alpha _k x = (n/k)_{n ∈ ℕ} ∈ \K^\infty
\]
- zwar eine Nullfolge in $\K^∝$ ist, diese Konvergenz ist jedoch nicht gleichmäßig in $n$.
- Man kann zeigen, dass $\K^∞$ mit $d$ vollständig, also ein Fréchet-Raum, ist.
- Ist $\K^∞$ auch normierbar?
- Also gibt es auf $\K^∞$ eine Norm, welche die gleiche Topologie erzeugt wie die $d$?
+ zwar eine Nullfolge in $\K^\infty$ ist, diese Konvergenz ist jedoch nicht gleichmäßig in $n$.
+ Man kann zeigen, dass $\K^\infty $ mit $d$ vollständig, also ein Fréchet-Raum, ist.
+ Ist $\K^\infty $ auch normierbar?
+ Also gibt es auf $\K^\infty $ eine Norm, welche die gleiche Topologie erzeugt wie die $d$?
Auch das ist nicht möglich:
\end{beispiel-nn}
\begin{lemma}
\label{lemma-s-metrikkugeln-enthalten-unterraeme}
- In $(\K^∞,d)$ gilt:
+ In $(\K^\infty ,d)$ gilt:
\begin{enumerate}
\item
- $B_1(0) = \K^∞$
+ $B_1(0) = \K^\infty $
\item
- Betrachte den linearen Unterraum $M_{n_0} := \{ x = (ξ_n)_{n ∈ ℕ}$ mit $ξ_n = 0$ für $n = 1,…,n_0 \}$.
+ Betrachte den linearen Unterraum $M_{n_0} \coloneq \{ x = (ξ_n)_{n ∈ ℕ}$ mit $ξ_n = 0$ für $n = 1,…,n_0 \}$.
Dann gibt es für jeden Radius $r > 0$ ein $n_0 ∈ ℕ$, so dass $M_{n_0} ⊂ B_{r}(0)$.
Das heißt, jede noch so kleine Metrikkugel enthält einen nichttrivialen Unterraum.
\end{enumerate}
@@ -1330,20 +1335,20 @@ Wir werden die unten angegebenen Beispiele auch gleich auf Vollständigkeit unte
Das ist trivial.
\item
Sei $r > 0$ gegeben.
- Wähle nun $n_0$, so dass $\sum_{n=n_0+1}^∞ 2^{-n} < r$.
+ Wähle nun $n_0$, so dass $\sum_{n=n_0+1}^\infty 2^{-n} < r$.
Dann gilt
\[
∀ x ∈ M_{n_0}: d(x,0) =
\sum_{n ∈ ℕ} 2^{-n} \frac{p_n(x)}{1+p_n(x)} =
- \sum_{n=n_0}^∞ 2^{-n} \frac{p_n(x)}{1+p_n(x)} \le
- \sum_{n=n_0}^∞ 2^{-n} < r.
+ \sum_{n=n_0}^\infty 2^{-n} \frac{p_n(x)}{1+p_n(x)} \le
+ \sum_{n=n_0}^\infty 2^{-n} < r.
\]
\end{enumerate}
\end{proof}
-Wäre nun die Topologie auf $(\K^∞,d)$ nun auch von einer Norm erzeugt, dann wären die Normkugeln
+Wäre nun die Topologie auf $(\K^\infty ,d)$ nun auch von einer Norm erzeugt, dann wären die Normkugeln
\[
- B_r^{\norm\cdot}(0) = \{ x ∈ \K^∞: \norm x < \tilde r \}
+ B_r^{\norm\cdot}(0) = \{ x ∈ \K^\infty : \norm x < \tilde r \}
\]
auch eine Umgebungsbasis der Null.
Das heißt insbesondere würden wir zu jedem $\tilde r$ ein $r$ finden, so dass $0 ∈ B_r^d(0) ⊂ B_r^{\norm\cdot} (0)$.
@@ -1352,19 +1357,19 @@ Mit \cref{lemma-s-metrikkugeln-enthalten-unterraeme} folgt also
M_{n_0} ⊂ B_r(0) ⊂ B_r^{\norm\cdot}(0)
\]
für ein geeignetes $n_0$.
-Sei nun ein $0 \ne x ∈ M_{n_0}$. Dann ist, da $M_{n_0}$ ein Unterraum ist, auch $αx ∈ M_{n_0}$ für alle $α ∈ \K$.
+Sei nun ein $0 \ne x ∈ M_{n_0}$. Dann ist, da $M_{n_0}$ ein Unterraum ist, auch $\alpha x ∈ M_{n_0}$ für alle $\alpha ∈ \K$.
Das heißt,
\[
- |α| \cdot \norm x = \norm{αx} < \tilde r \text{ für alle } α ∈ \K,
+ |\alpha | \cdot \norm x = \norm{\alpha x} < \tilde r \text{ für alle } \alpha ∈ \K,
\]
-was bereits $α = 0$ impliziert. Das ist ein Widerspruch.
+was bereits $\alpha = 0$ impliziert. Das ist ein Widerspruch.
\begin{beispiel-nn}[Räume beschränkter Funktionen]
- Sei $S$ eine beliebige Menge und $B(S) := \{ f: S → \K, f(s)$ ist beschränkt $\}$.
+ Sei $S$ eine beliebige Menge und $B(S) \coloneq \{ f: S → \K, f(s)$ ist beschränkt $\}$.
Dann wird $B(S)$ mit
\[
- \norm f _{B(S)} := \sup_{x ∈ S} |f(x)| < ∞,
+ \norm f _{B(S)} \coloneq \sup_{x ∈ S} |f(x)| < \infty ,
\]
der $\sup$-Norm, zu einem Banachraum.
Dabei ist offensichtlich, dass $\norm\cdot_{B(S)}$ tatsächlich eine Norm ist, und wir werden in einer Übung zeigen, dass die induzierte Metrik tatsächlich vollständig ist.
@@ -1393,14 +1398,14 @@ was bereits $α = 0$ impliziert. Das ist ein Widerspruch.
\]
ein normierter Raum mit
\[
- \norm{f}_{C(K)} = \norm{f}_{∞} = \max_{t ∈ K} |f(t)|,
+ \norm{f}_{C(K)} = \norm{f}_{\infty } = \max_{t ∈ K} |f(t)|,
\]
der Maximumsnorm.
Dieses Maximum wird tatsächlich immer angenommen, da $K$ kompakt ist (Satz von Minimum und Maximum).
Insbesondere sind alle stetigen Funktionen auf $K$ beschränkt. Damit gilt offensichtlich $C(K) ⊂ B(K)$ und $\norm{f}_{C(K)} = \norm{f}_{B(K)}$ für alle $f ∈ C(K)$.
Da jede stetige Funktion auf kompakten Teilmengen von metrischen Räumen auch gleichmäßig stetig ist, das heißt
\[
- ∀ ε > 0 ∃ δ > 0: \left( |t_1-t_2| < δ \implies |f(t_1)-f(t_2)| < ε \right) ∀ t_1,t_2 ∈ K
+ ∀ \epsilon > 0 ∃ \delta > 0: \left( |t_1-t_2| < \delta \implies |f(t_1)-f(t_2)| < \epsilon \right) ∀ t_1,t_2 ∈ K
\]
\end{beispiel-nn}
@@ -1409,19 +1414,19 @@ was bereits $α = 0$ impliziert. Das ist ein Widerspruch.
\end{lemma}
\begin{proof}
Sei $(f_i)_{i ∈ ℕ}$ eine konvergente (in $(B(K),\norm\cdot_{B(K)})$) Folge in $C(K)$.
- Dann existiert ein $f ∈ B(K)$ mit $f_i \xrightarrow[i → ∞]{\norm{\cdot}_{B(K)}} f$.
+ Dann existiert ein $f ∈ B(K)$ mit $f_i \xrightarrow[i → \infty ]{\norm{\cdot}_{B(K)}} f$.
Wir müssen zeigen, dass $f$ bereits stetig ist.
Für beliebige $t₁, t_2 ∈ K$ gilt
\[
- |f(t_1)-f(t_2) | \le \underbrace{|f_i(t_1)-f_i(t_2)|}_{< ε/3 \text{ für } |t_1-t_2| < δ^{(i)}(ε)} + 2 \underbrace{\norm{f_i - f}_{B(K)}}_{< ε/3 \text{ für } i > i_0} < ε.
+ |f(t_1)-f(t_2) | \le \underbrace{|f_i(t_1)-f_i(t_2)|}_{< \epsilon /3 \text{ für } |t_1-t_2| < \delta ^{(i)}(\epsilon )} + 2 \underbrace{\norm{f_i - f}_{B(K)}}_{< \epsilon /3 \text{ für } i > i_0} < \epsilon .
\]
Damit ist $f$ auch gleichmäßig stetig, also insbesondere auch stetig und in $C(K)$.
\end{proof}
-Das heißt, die Stetigkeit der Folgenglieder $(f_i)_{i ∈ ℕ} ⊂ C(K)$ überträgt sich auf die Grenzfunktion und Konvergenz in $(C(K),\norm\cdot_{∞})$ ist „gleichmäßig auf $K$“.
-Wegen dieser Eigenschaft ist die Maximumsnorm $\norm\cdot_∞$ auch die natürliche Norm auf $C(K)$.
+Das heißt, die Stetigkeit der Folgenglieder $(f_i)_{i ∈ ℕ} ⊂ C(K)$ überträgt sich auf die Grenzfunktion und Konvergenz in $(C(K),\norm\cdot_{\infty })$ ist „gleichmäßig auf $K$“.
+Wegen dieser Eigenschaft ist die Maximumsnorm $\norm\cdot_\infty $ auch die natürliche Norm auf $C(K)$.
Andere mögliche Normen (und damit andere Topologien) auf $C(K)$ wären z.B.
\[
- \norm{f}_p = \left( \int_K |f(t)|^p dt \right)^{1/p}, \quad 1 \le p < ∞.
+ \norm{f}_p = \left( \int_K |f(t)|^p dt \right)^{1/p}, \quad 1 \le p < \infty .
\]
Allerdings ist die Konvergenz in dieser Topologie impliziert keine Stetigkeit für die Grenzfunktion.
@@ -1429,7 +1434,7 @@ Allerdings ist die Konvergenz in dieser Topologie impliziert keine Stetigkeit f
\begin{beispiel-nn}
Sei $\Omega ⊂ ℝ^n$ offen und analog
\[
- C(\Omega) := \{ f: \Omega → \K, f \text { stetig }\}.
+ C(\Omega) \coloneq \{ f: \Omega → \K, f \text { stetig }\}.
\]
Hier können Funktionen aber auch unbeschränkt sein. Also braucht $\sup |f|$ nicht mehr zu existieren.
\end{beispiel-nn}
@@ -1447,7 +1452,7 @@ Man nehme z.B.
\[
K_m = \{ x ∈ \Omega ⊂ ℝ^n: \norm{x} \le m, \operatorname{dist}(x,∂\Omega) \ge 1/m\},
\]
-wobei $\operatorname{dist}(x,∂\Omega) := \inf\{ \norm{x-y}: y ∈ ∂\Omega\}$ und $∂M = \cl \Omega \setminus \Omega$.
+wobei $\operatorname{dist}(x,∂\Omega) \coloneq \inf\{ \norm{x-y}: y ∈ ∂\Omega\}$ und $∂M = \cl \Omega \setminus \Omega$.
Dann ist $C(\Omega)$ mit der Metrik
\[
@@ -1460,8 +1465,8 @@ ein Fréchetraum, also ein metrisierbarer linearer Raum nach \cref{satz-abzaehlb
Es gilt in diesem Raum
\[
- d(f_i,f) \xrightarrow[i → ∞]{} 0 \gdw
- \norm{f_i-f}_{C(K_m)} \xrightarrow[i → ∞]{} ∀m ∈ ℕ,
+ d(f_i,f) \xrightarrow[i → \infty ]{} 0 \gdw
+ \norm{f_i-f}_{C(K_m)} \xrightarrow[i → \infty ]{} ∀m ∈ ℕ,
\]
was ja gerade gleichmäßige Konvergenz auf jeder Kompakten Menge $K ⊂ \Omega$ bedeutet.
Damit ist Stetigkeit der Folgenglieder $(f_i)_{i ∈ ℕ} ⊂ C(\Omega)$ impliziert Stetigkeit der Grenzfunktion $f ∈ C(\Omega)$, da Stetigkeit nur eine lokale Eigenschaft ist.
@@ -1471,24 +1476,24 @@ Wir werden in der Übung sehen, dass $C(\Omega)$ mit dieser Metrik $d_{C(\Omega)
\begin{beispiel-nn}[Räume differenzierbarer Funktionen]
\begin{enumerate}
\item
- Betrachte die Menge $C^\ell(K) = \{ f: K → ℝ, D^α f$ existiert und ist stetig für$|α| < \ell \}$ der $\ell$-mal stetig differenzierbaren Funtktionen auf einer kompakten Menge $K ⊂ ℝ^n$ mit $\ell ∈ ℕ_0$
- Dabei ist $α = (α_1,…,α_n) ∈ ℕ_0^n$ ein Multiindex, $|α| = \sum_{i=1}^n α_i$ und
+ Betrachte die Menge $C^\ell(K) = \{ f: K → ℝ, D^\alpha f$ existiert und ist stetig für$|\alpha | < \ell \}$ der $\ell$-mal stetig differenzierbaren Funtktionen auf einer kompakten Menge $K ⊂ ℝ^n$ mit $\ell ∈ ℕ_0$
+ Dabei ist $\alpha = (\alpha _1,…,\alpha _n) ∈ ℕ_0^n$ ein Multiindex, $|\alpha | = \sum_{i=1}^n \alpha _i$ und
\[
- D^α f = \frac{∂^{|α|} f}{∂x_1^{α_1}\cdots∂x_n^{α_n}}.
+ D^\alpha f = \frac{∂^{|\alpha |} f}{∂x_1^{\alpha _1}\cdots∂x_n^{\alpha _n}}.
\]
Dann wird $C^\ell(K)$ mit der Norm
\[
- \norm{f}_{C^\ell(K)} = \max_{|α| \le l} \max_{x ∈ K} | D^α f(x)|
+ \norm{f}_{C^\ell(K)} = \max_{|\alpha | \le l} \max_{x ∈ K} | D^\alpha f(x)|
\]
zu einem Banachraum. Die meisten Eigenschaften sind klar, die Vollständigkeit folgt unmittelbar aus der Vollständigkeit von $C(K)$
Konvergenz in $C^\ell(K)$ bedeutet gerade gleichmäßige Konvergenz aller partiellen Ableitungen bis zur Ordnung $\ell$ auf ganz $K$.
\item
Sei $\Omega ⊂ ℝ^n$ offen und
- $\C^\ell(\Omega) = \{ f: \Omega → ℝ, D^α f$ existiert und ist stetig für$|α| < \ell \}$
+ $\C^\ell(\Omega) = \{ f: \Omega → ℝ, D^\alpha f$ existiert und ist stetig für$|\alpha | < \ell \}$
der Raum der $\ell$-mal stetig differenzierbaren Funtktionen auf $\Omega$ mit $\ell ∈ ℕ_0$.
$C^\ell(\Omega)$ wird mit der Metrik
\[
- d(f,g) := \sum_{m ∈ ℕ} 2^{-m} \frac{p_{m,l}(f-g)}{1+p_{m,l}(f-g)}, \quad p_{m,l}(f) = \max_{|α| \le \ell} \norm{D^α f}_{C(K_m)},
+ d(f,g) \coloneq \sum_{m ∈ ℕ} 2^{-m} \frac{p_{m,l}(f-g)}{1+p_{m,l}(f-g)}, \quad p_{m,l}(f) = \max_{|\alpha | \le \ell} \norm{D^\alpha f}_{C(K_m)},
\]
wobei die $K_m$ Ausschöpfungen von $\Omega$ mit kompakten Mengen sind, zu einem Fréchetraum.
Konvergenz in $C^\ell(\Omega)$ bedeutet gerade gleichmäßige Konvergenz aller partiellen Ableitungen bis zur Ordnung $\ell$ auf jedem Kompaktum, das in $\Omega$ enthalten ist.
@@ -1497,10 +1502,10 @@ Wir werden in der Übung sehen, dass $C(\Omega)$ mit dieser Metrik $d_{C(\Omega)
Wir betrachten nun einige Unterräume von $\C^\ell(\Omega)$:
\begin{enumerate}[label=(\roman*)]
\item
- $\C^\ell_B(\Omega) = \{ f: \Omega → ℝ, D^α f$ existiert, ist beschränkt und ist stetig für$|α| < \ell \}$
+ $\C^\ell_B(\Omega) = \{ f: \Omega → ℝ, D^\alpha f$ existiert, ist beschränkt und ist stetig für$|\alpha | < \ell \}$
wird zum normierten Raum mit
\[
- \norm{f}_{C^\ell_B(\Omega)} = \max_{|α| \le l} \sup_{x ∈ \Omega} | D^α f(x)|
+ \norm{f}_{C^\ell_B(\Omega)} = \max_{|\alpha | \le l} \sup_{x ∈ \Omega} | D^\alpha f(x)|
\]
Zwar gilt $C^\ell_B(\Omega) ⊂ C^\ell(\Omega)$ (als Mengen), jedoch besitzt $C^\ell_B(\Omega)$ nicht die Relativtopologie von $\C^\ell(\Omega)$, wie wir in einer Übung sehen werden.
\begin{definition}
@@ -1508,7 +1513,7 @@ Wir werden in der Übung sehen, dass $C(\Omega)$ mit dieser Metrik $d_{C(\Omega)
\item
Für $\Omega ⊂ ℝ^n$ offen und $f: \Omega → ℝ$ heißt
\[
- \supp f := \cl{ \{ x ∈ \Omega, f(x) \ne 0 \}}
+ \supp f \coloneq \cl{ \{ x ∈ \Omega, f(x) \ne 0 \}}
\]
der \emph{Träger} oder \emph{Support} von $f$.
\item
@@ -1533,16 +1538,16 @@ Wir werden in der Übung sehen, dass $C(\Omega)$ mit dieser Metrik $d_{C(\Omega)
\end{enumerate}
\item
Sei $\Omega ⊂ ℝ^n$ offen und
- $C^∞(\Omega) = \{ f: \Omega → ℝ, D^αf $ existiert und ist stetig für alle $α ∈ ℕ_0^n \} = \bigcap_{\ell ∈ ℕ}C^\ell(\Omega)$.
+ $C^\infty (\Omega) = \{ f: \Omega → ℝ, D^\alpha f $ existiert und ist stetig für alle $\alpha ∈ ℕ_0^n \} = \bigcap_{\ell ∈ ℕ}C^\ell(\Omega)$.
Wir definieren die Topologie wieder über eine Metrik durch Seminormen
\[
- d(f,g) := \sum_{m ∈ ℕ} 2^{-m} \frac{p_{m}(f-g)}{1+p_{m}(f-g)}, \quad p_{m}(f) = \max_{|α| \le m} \norm{D^α f}_{C(K_m)}.
+ d(f,g) \coloneq \sum_{m ∈ ℕ} 2^{-m} \frac{p_{m}(f-g)}{1+p_{m}(f-g)}, \quad p_{m}(f) = \max_{|\alpha | \le m} \norm{D^\alpha f}_{C(K_m)}.
\]
Mit dieser Metrik wird $C^\ell(\Omega)$ zum Fréchetraum.
- Konvergenz in $C^∞(\Omega)$ bedeutet gerade gleichmäßige Konvergenz aller partiellen Ableitungen auf jedem Kompaktum, das in $\Omega$ enthalten ist.
+ Konvergenz in $C^\infty (\Omega)$ bedeutet gerade gleichmäßige Konvergenz aller partiellen Ableitungen auf jedem Kompaktum, das in $\Omega$ enthalten ist.
Auch dieser Raum ist nicht normierbar mit einem analogem Argument wie bei den stetigen Funktionen auf $\Omega$.
\item
- Sei $\Omega ⊂ ℝ^n$ offen und $C_0^∞(\Omega) = \{ f ∈ C^∞(\Omega) : \supp f ⊂⊂ M \}$ der \emph{Raum der Testfunktionen}.
+ Sei $\Omega ⊂ ℝ^n$ offen und $C_0^\infty (\Omega) = \{ f ∈ C^\infty (\Omega) : \supp f ⊂⊂ M \}$ der \emph{Raum der Testfunktionen}.
Ein Beispiel für so eine Funktion ist
\[
f(x) =
@@ -1552,40 +1557,40 @@ Wir werden in der Übung sehen, dass $C(\Omega)$ mit dieser Metrik $d_{C(\Omega)
\end{cases},
\]
wobei $\Omega = B^{\norm\cdot_\infty}_2(0)$, $|\cdot| = \norm\cdot_2$ und $c ∈ ℝ$ konstant.
- Offensichtlich ist $C_0^∞(\Omega) ⊂ C^∞(\Omega)$.
- Wenn man auf $C_0^∞(\Omega)$ jedoch die Spurtopologie wählt, bekommt man später Probleme (bestimmte Funktionale auf $C_0^∞(\Omega)$ sind nicht mehr stetig, wie wir in einer Übungsaufgabe sehen werden.
- Man nennt Funktionale auf $C_0^∞(\Omega)$ auch Distributionen).
- Außerdem wäre der $C_0^∞(\Omega)$ mit dieser Metrik nicht vollständig -- der Träger der Grenzfunktion muss nicht mehr beschränkt sein.
+ Offensichtlich ist $C_0^\infty (\Omega) ⊂ C^\infty (\Omega)$.
+ Wenn man auf $C_0^\infty (\Omega)$ jedoch die Spurtopologie wählt, bekommt man später Probleme (bestimmte Funktionale auf $C_0^\infty (\Omega)$ sind nicht mehr stetig, wie wir in einer Übungsaufgabe sehen werden.
+ Man nennt Funktionale auf $C_0^\infty (\Omega)$ auch Distributionen).
+ Außerdem wäre der $C_0^\infty (\Omega)$ mit dieser Metrik nicht vollständig -- der Träger der Grenzfunktion muss nicht mehr beschränkt sein.
\begin{definition-nn}
Sei $M ⊂ X$ und $X$ ein linearer Raum. Dann heißt
\[
- \conv (M) := \{ x: ∃α_i > 0, x_i ∈ M, i ∈ \{1,…,k\}: \sum_{i=1}^k α_i = 1, \sum_{i=1}^k α_i x_i = x \}
+ \conv (M) \coloneq \{ x: ∃\alpha _i > 0, x_i ∈ M, i ∈ \{1,…,k\}: \sum_{i=1}^k \alpha _i = 1, \sum_{i=1}^k \alpha _i x_i = x \}
\]
die \emph{konvexe Hülle} von $M$.
\end{definition-nn}
- Aus Gründen, die erst später zu verstehen sind, wählt man auf $C^∞_0(\Omega)$ folgende lokalkonvxe Topologie:
+ Aus Gründen, die erst später zu verstehen sind, wählt man auf $C^\infty _0(\Omega)$ folgende lokalkonvxe Topologie:
Setze
\[
- p(\xi) := \sum_{k ∈ ℕ} 2^{-k} \frac{\norm \xi _{C^k(\Omega)}}{1 + \norm \xi _{C^k(\Omega)}}, \quad \xi ∈ C_0^∞(\Omega)
+ p(\xi) \coloneq \sum_{k ∈ ℕ} 2^{-k} \frac{\norm \xi _{C^k(\Omega)}}{1 + \norm \xi _{C^k(\Omega)}}, \quad \xi ∈ C_0^\infty (\Omega)
\]
Sei $(D_j)_{j ∈ ℕ}$ eine Ausschöpfung von $\Omega$ mit offenen Mengen, also $D_j ⊂ D_{j+1}, D_j ⊂⊂ \Omega, \bigcup_{j ∈ ℕ} D_j = \Omega$.
Eine mögliche Wahl wäre beispielsweise $D_j = K_j^\circ$, wobei die $K_j$ wie oben sind.
- Für $ε = (ε_j)_{j ∈ ℕ} ∈ ℝ^∞, ε_j > 0$ für alle $ℕ$ definieren wir eine Umgebungsbasis der $0 ∈ C_0^∞(\Omega)$ durch alle Mengen
+ Für $\epsilon = (\epsilon _j)_{j ∈ ℕ} ∈ ℝ^\infty , \epsilon _j > 0$ für alle $ℕ$ definieren wir eine Umgebungsbasis der $0 ∈ C_0^\infty (\Omega)$ durch alle Mengen
\[
- U_ε := \conv \left[ \bigcup_{j ∈ ℕ} \{ \xi ∈ C^∞_0 : p(\xi) < ε_j \} \right] ⊂ C_0^∞(\Omega).
+ U_\epsilon \coloneq \conv \left[ \bigcup_{j ∈ ℕ} \{ \xi ∈ C^\infty _0 : p(\xi) < \epsilon _j \} \right] ⊂ C_0^\infty (\Omega).
\]
- mit $ε = (ε_j)_{j ∈ ℕ} ∈ ℝ^∞, ε_j > 0$ und endliche Schnitte davon. Andere Umgebungen umgeben sich durch Translation.
- Die so definierte Topologie nennen wir $\T_\D$ und den Raum $C_1^∞(\Omega)$ auch $\D(\Omega)$.
+ mit $\epsilon = (\epsilon _j)_{j ∈ ℕ} ∈ ℝ^\infty , \epsilon _j > 0$ und endliche Schnitte davon. Andere Umgebungen umgeben sich durch Translation.
+ Die so definierte Topologie nennen wir $\T_\D$ und den Raum $C_1^\infty (\Omega)$ auch $\D(\Omega)$.
Es stellt sich heraus, dass diese Topologie tatsächlich unabhängig von der gewählten Ausschöpfung ist.
Außerdem ist $(\D(\Omega),\T_\D)$ ein topologischer linearer Raum, das heißt, die Vektorraumoperationen sind stetig.
\begin{lemma}[Charakterisierung offener Mengen in $\D(\Omega)$]
Es gilt
\[
- O ∈ \T_\D \iff ∀ ξ ∈ O ∃ ε=(e_j)_{j ∈ ℕ} ∈ ℝ^∞, e_j > 0: e+U_ε ⊂ O.
+ O ∈ \T_\D \iff ∀ ξ ∈ O ∃ \epsilon =(e_j)_{j ∈ ℕ} ∈ ℝ^\infty , e_j > 0: e+U_\epsilon ⊂ O.
\]
Das heißt, die Topologie $\T_\D$ und die Topologie
\[
- \tilde T_\D = \{ O ⊂ C_0^∞(\Omega): ∀ ξ ∈ O ∃ ε = (ε_j)_{j ∈ ℕ} ∈ ℝ^∞, ε_j > 0: ε+ U_ε ⊂ O \}
+ \tilde T_\D = \{ O ⊂ C_0^\infty (\Omega): ∀ ξ ∈ O ∃ \epsilon = (\epsilon _j)_{j ∈ ℕ} ∈ ℝ^\infty , \epsilon _j > 0: \epsilon + U_\epsilon ⊂ O \}
\]
sind gleich.
\end{lemma}
@@ -1593,53 +1598,53 @@ Wir werden in der Übung sehen, dass $C(\Omega)$ mit dieser Metrik $d_{C(\Omega)
Übung.
\end{proof}
\begin{korollar}
- Die Mengen $U_∈$ sind bereits eine Umgebungsbasis der Null.
+ Die Mengen $U_\epsilon$ sind bereits eine Umgebungsbasis der Null.
Nach Definition sind sie aber auch Konvex, das heißt $(\D(\Omega),\T_\D)$ ist ein lokalkonvexer Hausdorff-Raum.
\end{korollar}
\begin{satz}
- $ξ_m \xrightarrow[m → ∞]{} 0 \gdw$
+ $ξ_m \xrightarrow[m → \infty ]{} 0 \gdw$
\[
\begin{cases}
(i), & \text{Es existiert $D$ offen mit $D ⊂⊂ \Omega$ und
- $ξ_m ∈ C_0^∞(D)$ für alle $m ∈ ℕ$} \\
+ $ξ_m ∈ C_0^\infty (D)$ für alle $m ∈ ℕ$} \\
(ii), & \text{Für jedes $k ∈ ℕ$ gilt:
- $\norm{ξ_m}_{C^k(\cl{\Omega})} \xrightarrow[m → ∞]{} 0$}
+ $\norm{ξ_m}_{C^k(\cl{\Omega})} \xrightarrow[m → \infty ]{} 0$}
\end{cases}
\]
\end{satz}
\begin{proof}
- Zeige nur „$\Longleftarrow$“. Sei dazu $(ξ_m)_{m ∈ ℕ}$ eine Folge mit (i) und (ii).
+ Zeige nur „$\Leftarrow$“. Sei dazu $(ξ_m)_{m ∈ ℕ}$ eine Folge mit (i) und (ii).
Wähle nun $D_j$ von oben mit $D ⊂ D_j$ ($j$ ist fest).
- Sei nun $ε=(ε_i)_{i ∈ ℕ}, ε_i > 0$ gegeben. Dann müssen wir zeigen, dass für alle $m > m_0$ schon $ξ_m ∈ U_ε$ gilt.
- Zunächst sind nach (i) $ξ_m ∈ C^∞_0(D_j)$ .
+ Sei nun $\epsilon =(\epsilon _i)_{i ∈ ℕ}, \epsilon _i > 0$ gegeben. Dann müssen wir zeigen, dass für alle $m > m_0$ schon $ξ_m ∈ U_\epsilon $ gilt.
+ Zunächst sind nach (i) $ξ_m ∈ C^\infty _0(D_j)$ .
Außerdem gilt
\[
- p(\xi_m) \le \underbrace{\sum_{k=1}^N 2^{-k} \frac{ \norm{\xi_m}_{C^k(\cl \Omega)} } {1+ \norm{\xi_m}_{C^k(\cl \Omega)} }}_{\text{wegen (i)} < ε_j/2 \text{ für $m \ge m_0(ε_j,N)$}} + \underbrace{\sum_{k=N+1} 2^{-k}}_{<ε_j/2 \text{ für $n$ groß genug}} < ε_j.
+ p(\xi_m) \le \underbrace{\sum_{k=1}^N 2^{-k} \frac{ \norm{\xi_m}_{C^k(\cl \Omega)} } {1+ \norm{\xi_m}_{C^k(\cl \Omega)} }}_{\text{wegen (i)} < \epsilon _j/2 \text{ für $m \ge m_0(\epsilon _j,N)$}} + \underbrace{\sum_{k=N+1} 2^{-k}}_{<\epsilon _j/2 \text{ für $n$ groß genug}} < \epsilon _j.
\]
\end{proof}
\item
Betrachten wir nun Lebesgue-integrierbare Funktionen.
- Bereits eingeführt wurden die Räume $\L^p(\Omega)$ und $L^p(\Omega)$, $0 < p < ∞$, wobei $\Omega ⊂ ℝ^n$ offen.
- Diese sind für $1 \le p < ∞$ normiert, und für $0 < p < 1$ quasi-normiert.
- Für $p = ∞$ setzen wir
+ Bereits eingeführt wurden die Räume $\L^p(\Omega)$ und $L^p(\Omega)$, $0 < p < \infty $, wobei $\Omega ⊂ ℝ^n$ offen.
+ Diese sind für $1 \le p < \infty $ normiert, und für $0 < p < 1$ quasi-normiert.
+ Für $p = \infty $ setzen wir
\[
- \L^∞(\Omega) := \{ f: \Omega → ℝ ∪ \{ -∞, ∞ \}, f \text{ messbar und fast überall beschränkt} \}.
+ \L^\infty (\Omega) \coloneq \{ f: \Omega → ℝ ∪ \{ -\infty , \infty \}, f \text{ messbar und fast überall beschränkt} \}.
\]
Damit haben wir offenbar
\[
- C(\Omega) ∩ B(\Omega) ⊂ \L^∞(\omega).
+ C(\Omega) ∩ B(\Omega) ⊂ \L^\infty (\omega).
\]
Sei
\[
- \norm f _{\L^∞(\Omega)} := \supess_{t ∈ \Omega} |f(t)| := \inf_{M ⊂ \Omega \text{ NM}} \sup_{t ∈ \Omega \setminus M} |f(t)|.
+ \norm f _{\L^\infty (\Omega)} \coloneq \supess_{t ∈ \Omega} |f(t)| \coloneq \inf_{M ⊂ \Omega \text{ NM}} \sup_{t ∈ \Omega \setminus M} |f(t)|.
\]
- Dann gilt für $f ∈ \L^∞(\Omega)$
+ Dann gilt für $f ∈ \L^\infty (\Omega)$
\[
\norm f = 0 \gdw f = 0 \text{ fast überall}
\]
- Mit $N := \{ f ∈ \L^∞(\Omega) : \norm f = 0 \}$ wird
+ Mit $N \coloneq \{ f ∈ \L^\infty (\Omega) : \norm f = 0 \}$ wird
\[
- L^∞(\Omega) := \left( \L^∞(\Omega)/N, \norm\cdot_{L^∞(\Omega)} \right)
+ L^\infty (\Omega) \coloneq \left( \L^\infty (\Omega)/N, \norm\cdot_{L^\infty (\Omega)} \right)
\]
zu einem normiertem Raum.
\end{enumerate}
@@ -1648,21 +1653,21 @@ Wir werden in der Übung sehen, dass $C(\Omega)$ mit dieser Metrik $d_{C(\Omega)
{ \LARGE Vorlesung vom Donnerstag, 9. November fehlt (genauso wie vermutlich alle weiteren Donnerstagsvorlesungen ab jetzt)}
-Seien $f_n → f ∈ L^p(\Omega), h ∈ C_0^∞(\Omega)$.
+Seien $f_n → f ∈ L^p(\Omega), h ∈ C_0^\infty (\Omega)$.
Dann
\[
- \lim_{n → ∞} ∫_Ω f_n(t) h(t) dt = ∫_Ω f(t) h(t) dt,
+ \lim_{n → \infty } ∫_\Omega f_n(t) h(t) dt = ∫_\Omega f(t) h(t) dt,
\]
denn
\begin{align*}
- ∫_Ω (f_n(t) - f(t)) h(t) dt &\le ∫_{\supp h} M |f_n(t) - f(t)| dt \\
+ ∫_\Omega (f_n(t) - f(t)) h(t) dt &\le ∫_{\supp h} M |f_n(t) - f(t)| dt \\
& \stackrel{\mathclap{\text{Hölder}}}{\le} \; M [ \supp(h)]^{1/q} \norm{f_n-f}_{L^p(\Omega)} → 0.
\end{align*}
\section{Beschränkte und kompakte Mengen in metrischen linearen Räumen}
Wir wissen bereits nach dem Satz von Heine-Borel, dass eine Teilmenge $K ⊂ ℝ^n$ genau dann kompakt ist, wenn sie abgeschlossen und beschränkt ist.
-Beschränktheit bedeutet hier in einer (beliebigen, da alle äquivalent) Norm.
+Beschränktheit bedeutet hier Beschränktheit in einer (beliebigen, da alle äquivalent) Norm.
Nun wollen wir so ein Konzept für Beschränktheit auch in allgemeinen metrischen (topologischen) linearen Räumen finden.
@@ -1673,12 +1678,12 @@ Nun wollen wir so ein Konzept für Beschränktheit auch in allgemeinen metrische
\item
In einigen metrischen Räumen gilt ohnehin $d(x,0) \le 1$ für alle $x ∈ X$.
\item
- Ist $d$ eine Metrik auf $X$. Dann ist $\tilde d := \frac d {1+d} \le 1$ eine zu $d$ äquivalente Metrik auf $X$, wie wir in Topologie gesehen haben.
+ Ist $d$ eine Metrik auf $X$. Dann ist $\tilde d \coloneq \frac d {1+d} \le 1$ eine zu $d$ äquivalente Metrik auf $X$, wie wir in Topologie gesehen haben.
\end{enumerate}
\end{problem-nn}
\begin{definition}
- Sei $(X,\T)$ ein topologischer linearer Raum, $B ⊂ X$ heißt \emph{beschränkt}, falls zu jeder offenen Umgebung $U$ von $0 ∈ X$ ein $α > 0$ existiert, so dass $B ⊂ αU = \{αu: u ∈ U\}$, das heißt jede Nullumgebung lässt sich so weit „aufblasen“, dass sie $B$ überdeckt.
+ Sei $(X,\T)$ ein topologischer linearer Raum, $B ⊂ X$ heißt \emph{beschränkt}, falls zu jeder offenen Umgebung $U$ von $0 ∈ X$ ein $\alpha > 0$ existiert, so dass $B ⊂ \alpha U = \{\alpha u: u ∈ U\}$, das heißt jede Nullumgebung lässt sich so weit „aufblasen“, dass sie $B$ überdeckt.
\end{definition}
\begin{bemerkung-nn}
@@ -1692,33 +1697,33 @@ Nun wollen wir so ein Konzept für Beschränktheit auch in allgemeinen metrische
\end{satz}
\begin{proof}
„⇒“: Sei $k ∈ ℕ$ gegeben.
- Setze $r_k := \frac 1 {2^{k+1}}$ und $U := B_{r_k}(0)$.
- Da $B$ beschränkt ist, gibt es $α = α_k > 0$, dass
+ Setze $r_k \coloneq \frac 1 {2^{k+1}}$ und $U \coloneq B_{r_k}(0)$.
+ Da $B$ beschränkt ist, gibt es $\alpha = \alpha _k > 0$, dass
\begin{align*}
- & B ⊂ αU = α B_{r_k}(0) \\
- \iff & α^{-1} B ⊂ B_{r_k} (0) \\
- \iff d(α^{-1} x, 0) < r_k ∀ x ∈ B
+ & B ⊂ \alpha U = \alpha B_{r_k}(0) \\
+ \gdw & \alpha ^{-1} B ⊂ B_{r_k} (0) \\
+ \gdw &d(\alpha ^{-1} x, 0) < r_k ∀ x ∈ B
\end{align*}
- Dann gilt schon $p_k(x) \le M_k := α_k$ für alle $x ∈ B$, denn
+ Dann gilt schon $p_k(x) \le M_k \coloneq \alpha _k$ für alle $x ∈ B$, denn
\[
- \frac 1 {2^{k+1}} = r_k > d(α_k^{-1} x, 0
- \ge 2^k \frac {p_k(α_k^{-1}x)}{1+p_k(α_k^{-1} x)}
- = 2^{-k} \frac{α_k^{-1} p_k(x)}{1+α_k^{-1} p_k(x)}.
+ \frac 1 {2^{k+1}} = r_k > d(\alpha _k^{-1} x, 0
+ \ge 2^k \frac {p_k(\alpha _k^{-1}x)}{1+p_k(\alpha _k^{-1} x)}
+ = 2^{-k} \frac{\alpha _k^{-1} p_k(x)}{1+\alpha _k^{-1} p_k(x)}.
\]
- Also mit $\eta := α_k^{-1} p_k(x)$ gilt $\frac 1 2 > \frac \eta {1+\eta}$, also $\eta < 1$ oder $p_k(x) \le M_k$ für alle $x ∈ B$.
+ Also mit $\eta \coloneq \alpha _k^{-1} p_k(x)$ gilt $\frac 1 2 > \frac \eta {1+\eta}$, also $\eta < 1$ oder $p_k(x) \le M_k$ für alle $x ∈ B$.
„⇐“:
Sei also $p_k(x) \le M_k$ für alle $x ∈ B$ und $k ∈ ℕ$.
- Wir müssen nun zeigen, dass es für jedes $r > 0$ ein $α > 0$ gibt mit $B ⊂ αB_r(0)$, also $α^{-1} B ⊂ B_r(0)$.
+ Wir müssen nun zeigen, dass es für jedes $r > 0$ ein $\alpha > 0$ gibt mit $B ⊂ \alpha B_r(0)$, also $\alpha ^{-1} B ⊂ B_r(0)$.
Sei also $r > 0$ gegeben.
- Wähle nun $m_0 ∈ ℕ$ mit $\sum_{n=m_0+1}^∞ 2^{-n} < r/2$.
- Wähle $α > 0$ mit $\sum_{n=1}^{m_0} 2^{-n} \frac{α^{-1} M_k}{1+α^{-1} M_k} < r/2$.
+ Wähle nun $m_0 ∈ ℕ$ mit $\sum_{n=m_0+1}^\infty 2^{-n} < r/2$.
+ Wähle $\alpha > 0$ mit $\sum_{n=1}^{m_0} 2^{-n} \frac{\alpha ^{-1} M_k}{1+\alpha ^{-1} M_k} < r/2$.
Dann gilt für alle $x ∈ B$
- \[
- d(α^{-1} x, 0) =
- \sum_{n ∈ ℕ} 2^{-n} \frac{α^{-1} p_n(x)}{1+α^{-1} p_n(x)}
- \le \sum_{n=1}^{m_0} 2^{-n} \frac{α^{-1} p_n(x)}{1+α^{-1} p_n(x)} + \sum_{n=m_0+1}^∞ 2^{-n} < r/2 + r/2 = r.
- \]
+ \begin{align*}
+ d(\alpha ^{-1} x, 0) &=
+ \sum_{n ∈ ℕ} 2^{-n} \frac{\alpha ^{-1} p_n(x)}{1+\alpha ^{-1} p_n(x)} \\
+ &\le \sum_{n=1}^{m_0} 2^{-n} \frac{\alpha ^{-1} p_n(x)}{1+\alpha ^{-1} p_n(x)} + \sum_{n=m_0+1}^\infty 2^{-n} < r/2 + r/2 = r.
+ \end{align*}
\end{proof}
\begin{korollar}
@@ -1733,7 +1738,7 @@ Nun wollen wir so ein Konzept für Beschränktheit auch in allgemeinen metrische
\begin{bemerkung}
Kugeln $B_r(0)$ in $(X,d)$, wobei $d$ wie in \cref{satz-abzaehlbares-prod-seminormierter-raeume}, sinid also immer unbeschränkt,
weil nichttriviale Unterräume $M_{n_0} ⊂ B_r(x)$ existieren.
- Insbesondere ist dies gültig in den Räumen $\K^∞, C(Ω), C^\ell(Ω)$ und $C^∞(Ω)$.
+ Insbesondere ist dies gültig in den Räumen $\K^\infty , C(\Omega), C^\ell(\Omega)$ und $C^\infty (\Omega)$.
\end{bemerkung}
\begin{definition}
@@ -1750,7 +1755,7 @@ Nun wollen wir so ein Konzept für Beschränktheit auch in allgemeinen metrische
\end{satz}
\begin{beispiel-nn}
- Die Räume $\K^∞, C(Ω), C^\ell(Ω)$ und $C^∞(Ω)$ sind nicht lokalbeschränkt, aber lokalkonvex. Somit sind sie auch nicht normierbar.
+ Die Räume $\K^\infty , C(\Omega), C^\ell(\Omega)$ und $C^\infty (\Omega)$ sind nicht lokalbeschränkt, aber lokalkonvex. Somit sind sie auch nicht normierbar.
Auch $L^p(0,1)$ mit $0 < p < 1$ ist nicht lokalkonvex, aber lokalbeschränkt, also nicht normierbar.
\end{beispiel-nn}
@@ -1772,7 +1777,7 @@ Nun wollen wir so ein Konzept für Beschränktheit auch in allgemeinen metrische
\begin{warnung-nn}
Metrikkugeln müssen im Allgemeinen nicht kreisförmig sein (obwohl die uns bekannten Kugeln dies sind).
- Gegenbeispiel: $X = ℝ$, $d(x,y) := \left| ∫_x^y 1+\ind{ℝ_-}(s)\; ds \right|$.
+ Gegenbeispiel: $X = ℝ$, $d(x,y) \coloneq \left| ∫_x^y 1+\ind{ℝ_-}(s)\; ds \right|$.
\end{warnung-nn}
\begin{lemma}
@@ -1787,7 +1792,7 @@ Nun wollen wir so ein Konzept für Beschränktheit auch in allgemeinen metrische
„⊂“: Sei $x ∈ X$. Setze $β_n = 1/n, n ∈ ℕ$. Dann gilt
\[
- β_n x \xrightarrow[n → ∞]{} 0,
+ β_n x \xrightarrow[n → \infty ]{} 0,
\]
also $β_n ∈ V$ für $n \ge n_0$. Damit haben wir aber $x ∈ n_0 V$.
\end{proof}
@@ -1815,7 +1820,7 @@ Nun wollen wir so ein Konzept für Beschränktheit auch in allgemeinen metrische
\[
K ⊂ \bigcup_{i=1}^s n_i W \stackrel{(*)}= n_s W, \quad n_1 < n_2 < … < n_s,
\]
- also folgt die Behauptung mit $α = n_s$. $(*)$ gilt wegen der Kreisförmigkeit und $\left|\frac {n_i}{n_s}\right| \le 1$.
+ also folgt die Behauptung mit $\alpha = n_s$. $(*)$ gilt wegen der Kreisförmigkeit und $\left|\frac {n_i}{n_s}\right| \le 1$.
\end{proof}
\begin{bemerkung-nn}
Ohne die Hausdorff-Eigenschaft gilt dies nicht. Gegenbeispiel: $X = ℝ$ mit der Klumpentopologie.
@@ -1827,7 +1832,7 @@ Nun wollen wir so ein Konzept für Beschränktheit auch in allgemeinen metrische
\item
In einem topologischen Raum $(X,\T)$ heißt eine Menge $A ⊂ X$ \emph{relativ kompakt}, falls $\cl A$ kompakt ist.
\item
- In einem metrischen Raum $(X,d)$ heißt eine Menge $A ⊂ X$ \emph{präkompakt}, falls für jedes $ε > 0$ die Menge $A$ von endlich vielen Bällen mit Radius $ε$ überdeckt werden kann.
+ In einem metrischen Raum $(X,d)$ heißt eine Menge $A ⊂ X$ \emph{präkompakt}, falls für jedes $\epsilon > 0$ die Menge $A$ von endlich vielen Bällen mit Radius $\epsilon $ überdeckt werden kann.
\end{enumerate}
\end{definition}
@@ -1847,8 +1852,8 @@ Nun wollen wir so ein Konzept für Beschränktheit auch in allgemeinen metrische
Sei $(x_n)_{n ∈ ℕ} ⊂ A$ eine Cauchy-Folge. Nach (b) besitzt $(x_n)_{n ∈ ℕ}$ einen Häufungspunkt $x^*$.
Da $(x_n)_{n ∈ ℕ}$ Cauchy-Folge ist, konvergiert $(x_n)_{n ∈ ℕ}$ schon gegen $x^*$. Damit ist $A$ vollständig.
- Angenommen, $A$ wäre nicht präkompakt. Dann gibt es $ε > 0$, so dass $A$ keine endliche Überdeckung mit $ε$-Kugeln besitzt.
- Dadurch kann man eine Folge $(x_k)_{k ∈ K}$ definieren, mit $d(x_k,x_j) > ε$ für $k \ne j$.
+ Angenommen, $A$ wäre nicht präkompakt. Dann gibt es $\epsilon > 0$, so dass $A$ keine endliche Überdeckung mit $\epsilon $-Kugeln besitzt.
+ Dadurch kann man eine Folge $(x_k)_{k ∈ K}$ definieren, mit $d(x_k,x_j) > \epsilon $ für $k \ne j$.
Dann besitzt $(x_k)_{k ∈ K}$ offensichtlich keine Cauchy-Teilfolge, also auch keinen Häufungspunkt.
Also $A$ präkompakt.
\end{proof}
@@ -1867,18 +1872,18 @@ Hier gilt $M = \inf \{ c \ge 0:$ mit $C$ gilt (5) $\}$.
$(3) \iff (4)$ klar durch die Charakterisierung von beschränkten Mengen in
normierten Räumen und Ausnutzung der Linearität.
- $(2) \Rightarrow (4)$. Sei $T$ stetig in $x^*$. Wähle $ε > 0$, so dass $T(\cl B_ε(x^*)) ⊂ B_1(T(x^*))$.
+ $(2) \Rightarrow (4)$. Sei $T$ stetig in $x^*$. Wähle $\epsilon > 0$, so dass $T(\cl B_\epsilon (x^*)) ⊂ B_1(T(x^*))$.
Dann gilt für alle $x ∈ \cl B _1 (0)$
\[
- x^* + ε x ∈ \cl B_ε(x^*)
+ x^* + \epsilon x ∈ \cl B_\epsilon (x^*)
\]
- und $T(x^*) + εT(x) = T(x^* + εx) ∈ B_1(T(x^*))$, das heißt $ε T(x) ∈ B_1(0)$ oder $\norm{T(x)}_Y \le \frac 1 {ε} =: M$
+ und $T(x^*) + \epsilon T(x) = T(x^* + \epsilon x) ∈ B_1(T(x^*))$, das heißt $\epsilon T(x) ∈ B_1(0)$ oder $\norm{T(x)}_Y \le \frac 1 {\epsilon } =: M$
$(4) \Rightarrow (5)$. Für $x \ne 0$ gilt
\[
\norm{T(x)} \le \norm x \norm{T\left( \frac x {\norm x} \right)} \le M \norm x,
\]
- also gilt die Aussage mit $C := M$.
+ also gilt die Aussage mit $C \coloneq M$.
$(5) \Rightarrow (1)$. Für $x, x_1 ∈ X$ gilt
\[
@@ -1916,7 +1921,7 @@ In topologischen linearen Räumen gilt dies jedoch nciht.
3.6.7
\end{satz}
\begin{proof}
- Nur „$\Leftarrow$“: Nach 6.6 reicht es, Beschränktheit von $T$ zu zeigen, also dass, wenn $B ⊂ X$ beschränkt ist, auch $TN?) ⊂ Y$ beschränkt ist.
+ Nur „$\Leftarrow$“: Nach 6.6 reicht es, Beschränktheit von $T$ zu zeigen, also dass, wenn $B ⊂ X$ beschränkt ist, auch $T(B) ⊂ Y$ beschränkt ist.
$B ⊂ X$ ist genau dann beschränkt, wenn für alle $k ∈ ℕ$ $C_k > 0$ existieren mit $p_k(x) \le C_k$ für alle $x ∈ B$.
Nach Voraussetzung ist dann aber auch für alle $x ∈ B$
\[
@@ -1926,8 +1931,8 @@ In topologischen linearen Räumen gilt dies jedoch nciht.
\end{proof}
\begin{definition}
- Seien $X, Y$ topologische lineare Räume. Dann bezeichnet $\L(X, Y) := \{ T: X → Y: T$ linear und stetig $\}$ den \emph{Raum der stetigen (beschränkten) Operatoren}.
- Im Spezialfall $Y = \K$ sei $X' := \L(X, \K)$ der \emph{Raum der stetigen Funktionale} oder auch der \emph{Dualraum von $X$}.
+ Seien $X, Y$ topologische lineare Räume. Dann bezeichnet $\L(X, Y) \coloneq \{ T: X → Y: T$ linear und stetig $\}$ den \emph{Raum der stetigen (beschränkten) Operatoren}.
+ Im Spezialfall $Y = \K$ sei $X' \coloneq \L(X, \K)$ der \emph{Raum der stetigen Funktionale} oder auch der \emph{Dualraum von $X$}.
\end{definition}
\begin{bemerkung-nn}
\begin{enumerate}
@@ -1941,9 +1946,9 @@ In topologischen linearen Räumen gilt dies jedoch nciht.
Ist $X$ jedoch normierbar, so folgt aus den Hahn-Banach-Sätzen, dass $X'$ nichttrivial ist.
\item
Falls $X$ und $Y$ normierte Räume sind, dann wird $\L(X, Y)$ ebenfalls zu einem normierten Raum mit der Operatornorm
- \[
- \norm T := \norm T _{\L(X,Y)} := \sup \{\norm x _X \le 1\} \norm {Tx}_Y = \inf \{ C ≥ 0: ∀x ∈ X: \norm {Tx} \le C \norm x \}.
- \]
+ \begin{align*}
+ \norm T &\coloneq \norm T _{\L(X,Y)} \coloneq \sup \{\norm x _X \le 1\} \norm {Tx}_Y \\ &= \inf \{ C \ge 0: ∀x ∈ X: \norm {Tx} \le C \norm x \}.
+ \end{align*}
Das heißt, wir haben
\begin{equation}
\label{eq:61}
@@ -1970,8 +1975,8 @@ In topologischen linearen Räumen gilt dies jedoch nciht.
\begin{proof}
Es ist nur noch die Vollständigkeit zu zeigen.
Sei dazu $(T_n)_{n ∈ ℕ}$ eine Cauchy-Folge in $\L(X,Y)$.
- Das heißt, für jedes $ε > 0$ existiert ein $N_0$ mit $\norm {T_n - T_m} < ε$ für $n, m > N_0$.
- Also mit \eqref{eq:61} $\norm {T_n x - T_mx} \le \norm {T_n - T_m} \norm x < ε \norm x$ für alle $x ∈ X$ und $n,m > N_0$.
+ Das heißt, für jedes $\epsilon > 0$ existiert ein $N_0$ mit $\norm {T_n - T_m} < \epsilon $ für $n, m > N_0$.
+ Also mit \eqref{eq:61} $\norm {T_n x - T_mx} \le \norm {T_n - T_m} \norm x < \epsilon \norm x$ für alle $x ∈ X$ und $n,m > N_0$.
Insbesondere ist $(T_nx)_{n ∈ ℕ}$ eine Cauchy-Folge in $Y$. Da $Y$ vollständig ist, besitzt diese Folge einen Grenzwert $y_x ∈ Y$.
Wir definieren eine Abbildung
\[
@@ -1980,39 +1985,39 @@ In topologischen linearen Räumen gilt dies jedoch nciht.
Dann ist $T$ linear, weil alle $T_n$ linear sind. Also ist nur die Stetigkeit von $T$ und die Konvergenz von $(T_n)_{n ∈ ℕ}$ gegen $T$ zu zeigen.
Für die Stetigkeit bekommt man unter Verwendung der Dreicksunglechung direkt
\[
- \left| \norm {T_n} - \norm{T_m} \right| \le \norm {T_n - T_m} < ε \quad ∀ n, m ≥ N_0,
+ \left| \norm {T_n} - \norm{T_m} \right| \le \norm {T_n - T_m} < \epsilon \quad ∀ n, m \ge N_0,
\]
also eine Cauchyfolge $\left( \norm{T_n} \right)_{n ∈ ℕ}$ in $ℝ$, die wegen der Vollständigkeit von $ℝ$ konvergent, also insbesondere auch beschränkt ist.
Damit gibt es $M > 0$ mit $\norm {T_n} \le M$ für alle $n ∈ ℕ$, also mit~\eqref{eq:61}
\[
- \norm{Tx} \xleftarrow[n → ∞]{} \norm{T_nx } \le M \norm x, ∀ x ∈ X,
+ \norm{Tx} \xleftarrow[n → \infty ]{} \norm{T_nx } \le M \norm x, ∀ x ∈ X,
\]
also die stetigkeit von $T$.
Jetzt zur Konvergenz:
Für $\norm x \le$ 1 gilt
\[
- \norm {T_n x - T_m x } < ε, \quad ∀n, m ≥ N_0,
+ \norm {T_n x - T_m x } < \epsilon , \quad ∀n, m \ge N_0,
\]
- also durch Grenzwertbildung $n → ∞$
+ also durch Grenzwertbildung $n → \infty $
\[
- \norm {T_n x - T x } < ε, \quad ∀n ≥ N_0,
+ \norm {T_n x - T x } < \epsilon , \quad ∀n \ge N_0,
\]
und mit~\eqref{eq:61}
\[
- \norm {T_n -T} = \sup_{\norm x \le 1} \norm {T_n x - T_x} < ε, \quad ∀ n ≥ N_0,
+ \norm {T_n -T} = \sup_{\norm x \le 1} \norm {T_n x - T_x} < \epsilon , \quad ∀ n \ge N_0,
\]
das heißt $T_n → T$ wie gewünscht.
Für den Zusatz haben wir
\[
- \norm {S(Tx)} ≤ \norm S \norm {Tx} \le \norm S \norm T \norm x.
+ \norm {S(Tx)} \le \norm S \norm {Tx} \le \norm S \norm T \norm x.
\]
- Da das für alle $x ∈ X$ gilt, haben wir $\norm {ST} ≤ \norm S \norm T$.
+ Da das für alle $x ∈ X$ gilt, haben wir $\norm {ST} \le \norm S \norm T$.
\end{proof}
\begin{korollar}
- Ist $X$ ein Banachraum, dann ist $\L(X) := \L(X,X)$ eine \emph{Banachalgebra}, das heißt ein vollständiger normierter Vektorraum mit einer Multiplikation, so dass für $T, S ∈ \L(X)$ gilt:
+ Ist $X$ ein Banachraum, dann ist $\L(X) \coloneq \L(X,X)$ eine \emph{Banachalgebra}, das heißt ein vollständiger normierter Vektorraum mit einer Multiplikation, so dass für $T, S ∈ \L(X)$ gilt:
\[
\norm {TS} \le \norm T \norm S.
\]
@@ -2020,27 +2025,27 @@ In topologischen linearen Räumen gilt dies jedoch nciht.
\begin{bemerkung}
Ist $T ∈ \L(X,Y)$, so ist $\ker T$ als Urbild der abgeschlossenen Menge $\{ 0\}$ stets abgeschlossen in $X$.
- Das Bild hingegen $R(T) := \im T$ ist im Allgemeinen jedoch nicht abgeschlossen.
+ Das Bild hingegen $R(T) \coloneq \im T$ ist im Allgemeinen jedoch nicht abgeschlossen.
Wann sind Elemente in $\L(X)$ invertierbar?
\end{bemerkung}
\begin{satz}
- Sei $X$ ein Banachraum und $\T ∈ \L(X)$ mit $\limsup\limits_{m → ∞} \norm{T}^{1/m} < 1$. Dann ist $(\id - T)^{-1} ∈ \L(X)$ und es gilt
+ Sei $X$ ein Banachraum und $\T ∈ \L(X)$ mit $\limsup\limits_{m → \infty } \norm{T}^{1/m} < 1$. Dann ist $(\id - T)^{-1} ∈ \L(X)$ und es gilt
\[
- (\id-T)^{-1} = \ lim_{m → ∞} \sum_{n = 0}^m T^n =: \sum_{n = 0}^∞ T^n ∈ \L(X).
+ (\id-T)^{-1} = \ lim_{m → \infty } \sum_{n = 0}^m T^n =: \sum_{n = 0}^\infty T^n ∈ \L(X).
\]
mit Konvergenz in $\L(X)$.
\end{satz}
\begin{proof}
- Wähle $m_0$ und $Θ < 1$ mit $\norm {T^n} < Θ^n$ für $n ≥ m_0$.
- Für $S_k \sum_{n=0}^k T^n$ gilt dann für $m_0 \le k < l$
+ Wähle $m_0$ und $\Theta < 1$ mit $\norm {T^n} < \Theta ^n$ für $n \ge m_0$.
+ Für $S_k = \sum_{n=0}^k T^n$ gilt dann für $m_0 \le k < l$
\[
- \norm{ S_l - S_k} = \norm { \sum_{n=k+1}^l T^n} \le \sum_{n=k+1}^l \norm{ T^k} \le \sum_{n=k+1}^l Θ^n < ε, \quad k, l ≥ N_0.
+ \norm{ S_l - S_k} = \norm { \sum_{n=k+1}^l T^n} \le \sum_{n=k+1}^l \norm{ T^k} \le \sum_{n=k+1}^l \Theta ^n < \epsilon , \quad k, l \ge N_0.
\]
Damit ist $(S_k)_{k ∈ ℕ}$ eine Cauchy-Folge in $\L(X)$ und somit konvergent.
- Sei $S$ der Grenzwert. Dann gilt für jedes $x ∈ X$ auch $S_k x \xrightarrow[k → ∞]{\norm\cdot_{X}} Sx$, also damit ist für alle $x∈ X$
+ Sei $S$ der Grenzwert. Dann gilt für jedes $x ∈ X$ auch $S_k x \xrightarrow[k → \infty ]{\norm\cdot_{X}} Sx$, also damit ist für alle $x∈ X$
\[
- (\id - T) Sx = \lim_{k → ∞} (\id -T) S_k x = \lim_{k → ∞} \sum_{n=0}^k (T^n -T^{n-1})x = \lim_{k→∞} x - T^{k+1}x = x.
+ (\id - T) Sx = \lim_{k → \infty } (\id -T) S_k x = \lim_{k → \infty } \sum_{n=0}^k (T^n -T^{n-1})x = \lim_{k→\infty } x - T^{k+1}x = x.
\]
Damit ist $(\id -T)S = \id$. Da sich analog $S(\id-T) = \id$ auch zeigen lässt, folgt die Behauptung.
\end{proof}
@@ -2050,13 +2055,13 @@ In topologischen linearen Räumen gilt dies jedoch nciht.
3.7.6
\end{lemma}
\begin{bemerkung-nn}
- Mit $Θ = 1$ geht es nicht immer. Gegenbeispiel: Sei $X = C[0,1] ∩ \{ x(0) =
+ Mit $\Theta = 1$ geht es nicht immer. Gegenbeispiel: Sei $X = C[0,1] ∩ \{ x(0) =
0 \}$ und $M = \{ x ∈ X : g∫_0^1 x(t) dt = 0 \}$.
Dann ist $M$ ein abgeschlossener linearer Unterraum, weil $T: X → ℝ, ∫_0^1 \cdot$ stetig ist und somit $M = T^{-1}(\{0\})$ als Urbild einer abgeschlossenen Menge in $ℝ$ abgeschlossen ist.
- Angenommen, ($Θ=1$), es existierte ein $x_Θ = x_ ∈ X$ mit $\norm x_1 = $ und $\norm {x-x_1} \ge 1 $ für alle $x ∈ M$.
+ Angenommen, ($\Theta =1$), es existierte ein $x_\Theta = x ∈ X$ mit $\norm x_1 = $ und $\norm {x-x_1} \ge 1 $ für alle $x ∈ M$.
Dann setze
\[
- c(y) := \frac{∫_0^1 x_1(t) dt}{∫_0^1 y(t) dt} ∈ ℝ
+ c(y) \coloneq \frac{∫_0^1 x_1(t) dt}{∫_0^1 y(t) dt} ∈ ℝ
\]
für alle $y \not\in M$. Man beachte, dass dies wohldefiniert ist.
Dann ist $x_1 - c(y)y ∈ M$, also $1 \le \norm{ x_1 - c(y)y - x_1} = |c(y)|\norm y$.
@@ -2072,14 +2077,14 @@ In topologischen linearen Räumen gilt dies jedoch nciht.
\begin{proof}
„⇐“ war Korollar 7.4.
- „⇒“. Angenommen, $\dim X = \infty.$ Sei $S^1 := \{ x ∈ X: \norm x = 1\}$.
+ „⇒“. Angenommen, $\dim X = \infty.$ Sei $S^1 \coloneq \{ x ∈ X: \norm x = 1\}$.
Da $S^1$ abgeschlossen und beschränkt ist, ist $S^1$ nach Annahme kompakt.
- Wähle $x_1 ∈ S^1$ und $M_1 := \lspan \{ x_1 \} \subsetneq X$.
+ Wähle $x_1 ∈ S^1$ und $M_1 \coloneq \lspan \{ x_1 \} \subsetneq X$.
$M_1$ ist ein abgeschlossener Unterraum nach Korollar 7.5.
- Nach Ries existiert ein $x_2 ∈ S_1$ mit $\norm {x_2-x_1} \ge Θ := \frac 1 2 $.
- Setze nun $M_2 := \lspan \{x_1,x_2\}$.
- Da $M_2$ ein abgeschlossener Unterraum ist, existiert ein $x_3 ∈ S_1$ mit $\norm {x_3 - x} \ge Θ$ für alle $x ∈ M_2$, also insbesondere $\norm {x_3-x_1} \ge Θ = \frac 1 2$ und $\norm {x_3-x_2} \ge Θ = \frac 1 2$.
- Iterativ (da $\dim X = ∞ $) existiert $x_n ∈ S_1$ mit $\norm {x_m - x_n} \ge \frac 1 2$ für $m \ge n$.
+ Nach Ries existiert ein $x_2 ∈ S_1$ mit $\norm {x_2-x_1} \ge \Theta \coloneq \frac 1 2 $.
+ Setze nun $M_2 \coloneq \lspan \{x_1,x_2\}$.
+ Da $M_2$ ein abgeschlossener Unterraum ist, existiert ein $x_3 ∈ S_1$ mit $\norm {x_3 - x} \ge \Theta $ für alle $x ∈ M_2$, also insbesondere $\norm {x_3-x_1} \ge \Theta = \frac 1 2$ und $\norm {x_3-x_2} \ge \Theta = \frac 1 2$.
+ Iterativ (da $\dim X = \infty $) existiert $x_n ∈ S_1$ mit $\norm {x_m - x_n} \ge \frac 1 2$ für $m \ge n$.
Somit haben wir eine Folge $(x_n)_{n ∈ ℕ}$ ohne Häufungspunkt in $S^1$ gefunden im Widerspruch zu $S^1$ kompakt.
\end{proof}
@@ -2091,7 +2096,7 @@ Damit sind in unendlich-dimensionalen normierten Räumen weder die Sphären noch
\end{definition}
\begin{korollar}
- Sei $X$ normiert, $\dim X = ∞$. Dann ist $X$ nicht lokalkompakt.
+ Sei $X$ normiert, $\dim X = \infty $. Dann ist $X$ nicht lokalkompakt.
\end{korollar}
\begin{proof}
Angenommen, dass doch. Dann gibt es $r > 0$, so dass $S_r = \{ x ∈ X : \norm x = r\} ⊂ \cl U$.
@@ -2112,7 +2117,7 @@ Sei wieder $\K = \R$ oder $\K = ℂ$.
\item
$\langle x, y \rangle = \cl {\langle y, x \rangle}$ für alle $x, y ∈ X$.
\item
- $\langle x, αy + β z \rangle = α \langle x, y \rangle + β \langle x,z \rangle$ für alle $α, β ∈ \K$, $x,y,z ∈ X$.
+ $\langle x, \alpha y + β z \rangle = \alpha \langle x, y \rangle + β \langle x,z \rangle$ für alle $\alpha , β ∈ \K$, $x,y,z ∈ X$.
\end{enumerate}
$(X,\langle -,- \rangle)$ heißt \emph{Skalarproduktraum}, \emph{unitärer Raum} oder \emph{Prähilbertraum}.
\end{definition}
@@ -2125,7 +2130,7 @@ Sei wieder $\K = \R$ oder $\K = ℂ$.
Sei $(X, \langle -,- \rangle)$ ein unitärer Raum. Dann gelten die folgenden Aussagen:
\begin{enumerate}
\item
- Durch $\norm x := \sqrt{\langle x, x \rangle}$ wird eine Norm definiert.
+ Durch $\norm x \coloneq \sqrt{\langle x, x \rangle}$ wird eine Norm definiert.
Dadurch wird jeder unitäre Raum auf natürliche Art und Weise normiert und trägt dadurch die induzierte natürliche Topologie.
\item
$|\langle x,y \rangle| \le \norm x \norm y$ mit Gleichheit genau dann, wenn $x$ und $y$ linear abhängig (Cauchy-Schwarz-Ungleichung).
@@ -2147,14 +2152,14 @@ Sei wieder $\K = \R$ oder $\K = ℂ$.
\item
Einfaches Nachrechnen unter Verwendung von (b)
\item
- Für $y = 0$ ist die Behauptung klar. Sei also $y \ne 0, α ∈ ℂ$.
+ Für $y = 0$ ist die Behauptung klar. Sei also $y \ne 0, \alpha ∈ ℂ$.
Dann
\[
- \langle x + αy, x+αy \rangle = \langle x, x \rangle + \cl \alpha \langle y, x \rangle + α \langle x,y \rangle + |α|^2 \langle y,y \rangle.
+ \langle x + \alpha y, x+\alpha y \rangle = \langle x, x \rangle + \cl \alpha \langle y, x \rangle + \alpha \langle x,y \rangle + |\alpha |^2 \langle y,y \rangle.
\]
- Speziell für $\cl \alpha := - \frac{\langle x,y \rangle}{\langle y,y \rangle}$ ergibt sich
+ Speziell für $\cl \alpha \coloneq - \frac{\langle x,y \rangle}{\langle y,y \rangle}$ ergibt sich
\[
- 0 \le \langle x + αy, x+α+ \rangle = \langle x,x \rangle - \frac{|\langle x,y \rangle^2|}{\langle y,y \rangle} - \frac{|\langle x,y \rangle^2|}{\langle y,y \rangle} + \frac{|\langle x,y \rangle^2|}{\langle y,y \rangle} = \langle x,x \rangle - \frac{|\langle x,y \rangle^2|}{\langle y,y \rangle}.
+ 0 \le \langle x + \alpha y, x+\alpha + \rangle = \langle x,x \rangle - \frac{|\langle x,y \rangle^2|}{\langle y,y \rangle} - \frac{|\langle x,y \rangle^2|}{\langle y,y \rangle} + \frac{|\langle x,y \rangle^2|}{\langle y,y \rangle} = \langle x,x \rangle - \frac{|\langle x,y \rangle^2|}{\langle y,y \rangle}.
\]
Durch Umstellen ergibt sich
\[
@@ -2163,7 +2168,7 @@ Sei wieder $\K = \R$ oder $\K = ℂ$.
Die CSU erhält man durch Wurzel ziehen.
Gleichheit gilt genau dann, wenn
\[
- \langle x+ α y, x+αy \rangle = 0 \gdw x + αy = 0,
+ \langle x+ \alpha y, x+\alpha y \rangle = 0 \gdw x + \alpha y = 0,
\]
also wenn $x$ und $y$ linear abhängig sind.
\item
@@ -2174,9 +2179,10 @@ Sei wieder $\K = \R$ oder $\K = ℂ$.
Addieren dieser Gleichungen für $+$ und $-$ ergibt die Behauptung.
\item
Es gilt
- \[
- \norm {x+y}^2 - \norm{x-y}^2 = (\norm x^2 + 2 \Re \langle x,y\rangle + \norm y^2) - (\norm x^2 - 2 \Re \langle x,y \rangle + \norm y^2) = 4 \Re \langle x,y \rangle.
- \]
+ \begin{align*}
+ \norm {x+y}^2 - \norm{x-y}^2 &= (\norm x^2 + 2 \Re \langle x,y\rangle + \norm y^2) - (\norm x^2 - 2 \Re \langle x,y \rangle + \norm y^2) \\
+ & = 4 \Re \langle x,y \rangle.
+ \end{align*}
Analog haben wir
\[
-i \norm{x+iy}^2 + i \norm{x-iy}^2 = … = 4i \Im \langle x,y \rangle,
@@ -2203,16 +2209,17 @@ Sei wieder $\K = \R$ oder $\K = ℂ$.
\begin{bemerkung}
\begin{enumerate}
- \item Die Paralellogramgleichung ist also charakteristisch für unitäre Räume.
\item
- $(C(S),\norm\cdot_∞)$ mit $S ⊂ ℝ^n$ kompakt erfüllt dies nicht.
+ Die Paralellogrammgleichung ist somit charakteristisch für unitäre Räume.
\item
- Die Abbildung $\langle -,- \rangle$ in unitären Räumen ist stetig in beiden Komponenten als unmittelbare Konsequenz aus der Stetigkeit der Norm.
+ $(C(S),\norm\cdot_\infty )$ mit $S ⊂ ℝ^n$ kompakt erfüllt dies nicht.
+ \item
+ Die Abbildung $\langle \cdot,\cdot \rangle$ in unitären Räumen ist stetig in beiden Komponenten als unmittelbare Konsequenz aus der Stetigkeit der Norm.
\end{enumerate}
\end{bemerkung}
\begin{definition}
- Ein bezüglich der Norm $\norm - := \sqrt{ \langle -,- \rangle}$ vollständiger unitärer Raum $(X,\langle -,- \rangle)$ heißt \emph{Hilbertraum}.
+ Ein bezüglich der Norm $\norm \cdot \coloneq \sqrt{ \langle \cdot,\cdot \rangle}$ vollständiger unitärer Raum $(X,\langle \cdot,\cdot \rangle)$ heißt \emph{Hilbertraum}.
\end{definition}
@@ -2226,15 +2233,17 @@ Hier fehlt eine VL.
Sei also $\hat y ∈ Y$ mit $x-\hat y \perp Y$, also $x-\hat y \perp (\hat y - y)$ für $y ∈ Y$ beliebig.
Dann gilt mit Pythagoras
\[
- \norm{x-y}^2 = \norm{x-\hat y + \hat y - y}^2 = \norm{x-\hat y}^2 + \norm{\hat y - y}^2 ≥ \norm{x-\hat y}^2,
+ \norm{x-y}^2 = \norm{x-\hat y + \hat y - y}^2 = \norm{x-\hat y}^2 + \norm{\hat y - y}^2 \ge \norm{x-\hat y}^2,
\]
was die Behauptung impliziert.
\end{proof}
\begin{bemerkung-nn}
- Damit gilt im Hilbertraum das Riesz'sche Lemma (3.7.6) mit $Θ = 1$.
+ Damit gilt im Hilbertraum das Riesz'sche Lemma (3.7.6) mit $\Theta = 1$.
Setze dazu
- $ x_{Θ=1} := \frac{x-\hat y }{\norm{x-\hat y}} $
- für ein $x \notin Y$. Dann ist $\norm{x_Θ} = 1$ und für alle $z ∈ Y$ gilt $\norm {z-x_Θ}^2 + 2 \Re \langle z,x_Θ \rangle + \norm{x_Θ}^2 ≥ 1 = Θ$.
+ $ x_{\Theta =1} \coloneq \frac{x-\hat y }{\norm{x-\hat y}} $
+ für ein $x \notin Y$. Dann ist $\norm{x_\Theta } = 1$ und für alle $z ∈ Y$
+gilt $\norm {z-x_\Theta }^2 + 2 \Re \langle z,x_\Theta \rangle + \norm{x_\Theta
+}^2 \ge 1 = \Theta $.
\end{bemerkung-nn}
\begin{satz}
Es sei $Y$ ein vollständiger Unterraum eines unitären Raums $X$.
@@ -2315,17 +2324,17 @@ Zentral in der Hilbertraumtheorie ist der Begriff der Hilbertraumbasis.
\begin{enumerate}
\item Für alle $x ∈ X$ gilt die Vollständigkeitsrelation
\[
- \lim_{n → ∞} \norm{x - \sum_{k=1}^n \langle \hat e_k, x \rangle \hat e_k} = 0
+ \lim_{n → \infty } \norm{x - \sum_{k=1}^n \langle \hat e_k, x \rangle \hat e_k} = 0
\]
\item
Für alle $x, y ∈ X$ ist
\[
- \langle x,y \rangle = \sum_{k=1}^∞ \cl{\langle \hat e_k. x \rangle} \langle \hat e_k, y \rangle.
+ \langle x,y \rangle = \sum_{k=1}^\infty \cl{\langle \hat e_k. x \rangle} \langle \hat e_k, y \rangle.
\]
\item
Für alle $x ∈ X$ gilt die Parseval-Gleichung
\[
- \norm{x}^2 = \sum_{k=1}^∞ \left| \langle \hat e_k, x \rangle \right|^2.
+ \norm{x}^2 = \sum_{k=1}^\infty \left| \langle \hat e_k, x \rangle \right|^2.
\]
\end{enumerate}
\end{definition}
@@ -2336,8 +2345,8 @@ Zentral in der Hilbertraumtheorie ist der Begriff der Hilbertraumbasis.
\begin{enumerate}
\item Statt (a) kann man auch
\[
- x = \lim_{n → ∞} \sum_{k=1}^n \langle \hat e_k, x \rangle \hat e_k
- = \sum_{k=1}^∞ \langle \hat e_k,x \rangle \hat e_k
+ x = \lim_{n → \infty } \sum_{k=1}^n \langle \hat e_k, x \rangle \hat e_k
+ = \sum_{k=1}^\infty \langle \hat e_k,x \rangle \hat e_k
\]
schreiben. Dies nennt man die Fourier-Reihe von $x$.
\item
@@ -2364,14 +2373,14 @@ Zentral in der Hilbertraumtheorie ist der Begriff der Hilbertraumbasis.
\item
Sei $S$ wie oben. Sei $x ∈ X$ mit $x \perp S$. Nach (c) gilt dann
\[
- \sum_{k=1}^∞ \big| \underbrace{\langle \hat e_k^∞, x \rangle}_{=0} \big|^2 = \norm x ^2,
+ \sum_{k=1}^\infty \big| \underbrace{\langle \hat e_k^\infty , x \rangle}_{=0} \big|^2 = \norm x ^2,
\]
also $\norm x = 0$ und $x = 0$.
\item
Sei nun $S$ ein abzählbares vollständiges Orthonormalensystem und $X$ ein Hilbertraum.
Führe den Beweis indirekt.
Angenommen, $S$ wäre keine Hilbertraumbasis.
- Dann gelten die Eigenschaften (a)-(c) aus der Definition nicht und wegen der obigen Bemerkung ist dann $Y := \cl{\lspan S} \subsetneq X$.
+ Dann gelten die Eigenschaften (a)-(c) aus der Definition nicht und wegen der obigen Bemerkung ist dann $Y \coloneq \cl{\lspan S} \subsetneq X$.
$Y$ ist also ein abgeschlossener Unterraum von $X$, und da $X$ Hilbertraum ist, damit vollständig.
Nach Satz 2.9 ist $X = Y \oplus Y^\perp$.
Insbesondere ist also $Y^\perp \ne \{ 0\}$.
@@ -2383,7 +2392,7 @@ Zentral in der Hilbertraumtheorie ist der Begriff der Hilbertraumbasis.
\end{enumerate}
\end{proof}
\begin{frage-nn}
- Hat jeder Hilbertraum $H$ mit $\dim H = ∞$ ein abzählbares vollständiges ONS (also eine Hilbertbasis)?
+ Hat jeder Hilbertraum $H$ mit $\dim H = \infty $ ein abzählbares vollständiges ONS (also eine Hilbertbasis)?
\end{frage-nn}
Die Antwort darauf ist nein, aber falls $H$ zusätzlich separabel ist, dann ist sie ja.
Dagegen ist die Existenz eines vollständigen Orthonormalensystems (also eventuell überabzählbar, also keine ONB) kein Problem:
@@ -2400,48 +2409,47 @@ Dagegen ist die Existenz eines vollständigen Orthonormalensystems (also eventue
Sei $X = L^2(0,2\pi), \K = ℝ$.
Dann ist ein VONS in $X$ gegeben durch
\[
- S = \left\{ \frac 1 {\sqrt{2π}}\right\}
- ∪ \left\{ \frac 1 {\sqrt{π}} \cos(nx) : n ∈ ℕ\right\}
- ∪ \left\{ \frac 1 {\sqrt{π}} \sin(nx) : n ∈ ℕ\right\}.
+ S = \left\{ \frac 1 {\sqrt{2\pi }}\right\}
+ ∪ \left\{ \frac 1 {\sqrt{\pi }} \cos(nx) : n ∈ ℕ\right\}
+ ∪ \left\{ \frac 1 {\sqrt{\pi }} \sin(nx) : n ∈ ℕ\right\}.
\]
In der klassischen Fourieranalysis werden Entwicklungen nach diesem VONS $S$ untersucht.
- Man zeigt dort, dass $\lspan S$ bezüglich $\norm\cdot_∞$ dicht liegt in $C_\text{per}([0,2\pi]) = \{ f: ℝ → ℝ: f$ ist stetig und $2π$-periodisch $\}$.
- Die Aussage von 2.13(2) und (2.10) liefert nur die Begründung für die Dichtheit von $\lspan S$ in $\norm-_{L^2}$.
+ Man zeigt dort, dass $\lspan S$ bezüglich $\norm\cdot_\infty $ dicht liegt in $C_{\text{per}}([0,2\pi]) = \{ f: \R → \R: f$ ist stetig und $2\pi $-periodisch $\}$.
+ Die Aussage von 2.13(2) und (2.10) liefert nur die Begründung
+für die Dichtheit von $\lspan S$ in $\norm-_{L^2}$.
\item
- Durch $(f,g)_μ := ∫_a^b μ(t) f(t) g(t)\; dt $, wobei $μ > 0$ und stetig auf $(a,b)$, ist auf $L^2(a,b)$ ein reelles Skalarprodukt definiert.
- Für verschiedene Gewichtsfunktionen $μ$ und verschiedene Wahlen von $(a,b)$ erhält man $μ$-orthogonale Polynomsysteme durch Anwendung des Gram-Schmidt-Verfahrens auf die Monome $\{t^i: i ∈ ℕ_0\}$.
+ Durch $(f,g)_\mu \coloneq ∫_a^b \mu (t) f(t) g(t)\; dt $, wobei $\mu > 0$ und stetig auf $(a,b)$, ist auf $L^2(a,b)$ ein reelles Skalarprodukt definiert.
+ Für verschiedene Gewichtsfunktionen $\mu $ und verschiedene Wahlen von $(a,b)$ erhält man $\mu $-orthogonale Polynomsysteme durch Anwendung des Gram-Schmidt-Verfahrens auf die Monome $\{t^i: i ∈ ℕ_0\}$.
\begin{enumerate}[label=(\roman*)]
\item
- $a=-1, b=1$, $μ(t) = 1$ liefert die Legendre-Polynome.
+ $a=-1, b=1$, $\mu (t) = 1$ liefert die Legendre-Polynome.
\item
- $a=-1, b=1$, $μ(t) = 1$ liefert die Tschebyscheff-Polynome.
- Das stimmt nicht, danke, dass du die Folie so lange gezeigt hast.
+ $a=-1, b=1$, $\mu (t) = \frac 1 {\sqrt{1-t^2}}$ liefert die Tschebyscheff-Polynome.
\item
- $a=0, b=∞$, $μ(t) = 1$ liefert die Laguerre-Polynome.
- Das stimmt nicht, danke, dass du die Folie so lange gezeigt hast.
+ $a=0, b=\infty $, $\mu (t) = \exp(-t)$ liefert die Laguerre-Polynome.
\item
- $a=-∞, b=∞$, $μ(t) = \exp(-t^2)$ liefert die Hermite-Polynome.
+ $a=-\infty , b=\infty $, $\mu (t) = \exp(-t^2)$ liefert die Hermite-Polynome.
\end{enumerate}
\item
Ist $X$ ein unitärer Raum mit ONB, kann er formal vervollständigt werden:
Sei also $(\hat e_k)_{k ∈ ℕ} ⊂ X$ diese ONB, dann ist
\[
- H := \left\{ \sum_{k=1}^∞ c_k \hat e_k: (c_k)_{k ∈ ℕ} ∈ \ell^2 \right\}
+ H \coloneq \left\{ \sum_{k=1}^\infty c_k \hat e_k: (c_k)_{k ∈ ℕ} ∈ \ell^2 \right\}
\]
ist ein Hilbertraum, den man die Vervollständigung von $X$ nennt.
Das Skalarprodukt zwischen $x = \sum_{k ∈ ℕ} c_k \hat e_k$ und $y = \sum_{k ∈ ℕ} d_k \hat e_k$
wird definiert als
\[
- \langle x,y \rangle := \sum_{k=1}^∞ \cl{c_k} d_k.
+ \langle x,y \rangle \coloneq \sum_{k=1}^\infty \cl{c_k} d_k.
\]
Tatsächlich kann $H$ mit dem Koordinatenraum $\ell^2 = \ell^2(ℕ)$ identifiert werden.
Die Abbildung
\[
- \Phi: \ell^2(ℕ) → H, (c_k)_{k ∈ℕ} ↦ \sum_{k=1}^∞ c_k \hat e_k
+ \Phi: \ell^2(ℕ) → H, (c_k)_{k ∈ℕ} ↦ \sum_{k=1}^\infty c_k \hat e_k
\]
ist linear, bijektiv und normerhaltend wegen der Parsevalgleichung
\[
- \norm{x}^2 = \sum_{k=1}^∞ \left| \langle \hat e_k, x \rangle \right|^2.
+ \norm{x}^2 = \sum_{k=1}^\infty \left| \langle \hat e_k, x \rangle \right|^2.
\]
Also $\ell^2(ℕ)$ und $H$ isometrisch und insbesondere $H$ vollständig.
\end{enumerate}
@@ -2450,11 +2458,11 @@ Dagegen ist die Existenz eines vollständigen Orthonormalensystems (also eventue
% VL NÄCHSTE WOCHE
-Der Satz 4.1 liefert also, dass die Abbildung $J_x: X → X,', y ↦ y'$ definiert
+Der Satz 4.1 liefert also, dass die Abbildung $J_x: X → X', y ↦ y'$ definiert
durch $y': X → \K, x ↦ \langle y,x \rangle$ bijektiv ist.
Wir schreiben nun
\[
-\langle \langle J_x(y),x \rangle \rangle = \langle \langle J_x(y),x \rangle \rangle_{X'×X} := J(x)(y)[x]
+\lAngle J_x(y),x \rAngle = \lAngle J_x(y),x \rAngle_{X'×X} \coloneq J(x)(y)[x]
= \langle y,x \rangle.
\]
Diese Abbildung ist sesquiliniear, das heißt
@@ -2462,24 +2470,24 @@ Diese Abbildung ist sesquiliniear, das heißt
J_x (y_1 + y_2) = J_x (y_1) + J_x(y_2), \quad y_1, y_2 ∈ X,
\]
\[
- J_x(αy) = \cl{\alpha} J_x(y), \quad α ∈ \K,
+ J_x(\alpha y) = \cl{\alpha} J_x(y), \quad \alpha ∈ \K,
\]
denn
\[
- \langle \langle J_x(αy),x \rangle \rangle = \langle αy, x \rangle = \cl \alpha \langle y, x \rangle = \cl \alpha J_x(y) [x] = \cl \alpha \langle \langle J_x(y), x \rangle \rangle
- \langle \langle \cl \alpha J_x(y), x \rangle \rangle,
+ \lAngle J_x(\alpha y),x \rAngle = \langle \alpha y, x \rangle = \cl \alpha \langle y, x \rangle = \cl \alpha J_x(y) [x] = \cl \alpha \lAngle J_x(y), x \rAngle
+ \lAngle \cl \alpha J_x(y), x \rAngle,
\]
also $X \cong X'$ sesquilinear isomorph.
Gilt da sauch topologisch?
-Die Topologie von $X'$ sei hierbei die von $\L(X, \K)$, also die von der Norm $\norm{y'}_{X',N} = \sup_{\norm{x} ≤ 1}|y'[x]|$ erzeugte.
+Die Topologie von $X'$ sei hierbei die von $\L(X, \K)$, also die von der Norm $\norm{y'}_{X',N} = \sup_{\norm{x} \le 1}|y'[x]|$ erzeugte.
\begin{satz}
$X$ und $X'$ sind Hilberträume und $J_x: X → X'$ ist kanonischer sesquilinearer Isomorphismus, der die Norm erhält, also eine Isometrie.
Genauer gilt:
\begin{enumerate}
\item
- $\langle y_1', y_2' \rangle_{X'} := \cl{ \langle y_1, y_2 \rangle_X}$, wobei $J_x(y_1) = y_1', J_x(y_2) = y_2'$, macht $X'$ zum Skalarproduktraum.
+ $\langle y_1', y_2' \rangle_{X'} \coloneq \cl{ \langle y_1, y_2 \rangle_X}$, wobei $J_x(y_1) = y_1', J_x(y_2) = y_2'$, macht $X'$ zum Skalarproduktraum.
\item
Die durch $\langle -,- \rangle_{X'}$ induzierte Norm
\[
@@ -2497,7 +2505,7 @@ Die Topologie von $X'$ sei hierbei die von $\L(X, \K)$, also die von der Norm $\
\item
Beispielsweise ist
\[
- \langle α y_1' , y_2' \rangle_{X'} \stackrel{def}{=} \cl{\langle \cl \alpha y_1, y_2 \rangle_X} = \cl{ \alpha \langle y_1, y_2 \rangle_X} = \cl{\alpha} \langle y_1',y_2' \rangle_{X'},
+ \langle \alpha y_1' , y_2' \rangle_{X'} \stackrel{\text{def}}{=} \cl{\langle \cl \alpha y_1, y_2 \rangle_X} = \cl{ \alpha \langle y_1, y_2 \rangle_X} = \cl{\alpha} \langle y_1',y_2' \rangle_{X'},
\]
die anderen Eigenschaften folgen analog.
\item
@@ -2505,7 +2513,7 @@ Die Topologie von $X'$ sei hierbei die von $\L(X, \K)$, also die von der Norm $\
\[
\norm{y'}_{X',N} = \sup_{\norm x \le 1} |y'[x]| = \norm{y}_{X} \quad \text{für alle $y ∈ X$}.
\]
- hierbei ist aber „$\le$“ gerade die Cauchy-Schwarzsche Ungleichung, für „$\ge$“ wähle $x = \frac y {\norm y _{X}}$ für $y \ne 0$ ($y=0$ ist sowieso klar).
+ hierbei ist aber „$\le$“ gerade die Cauchy"=Schwarzsche Ungleichung, für „$\ge$“ wähle $x = \frac y {\norm y _{X}}$ für $y \ne 0$ ($y=0$ ist sowieso klar).
\item
nichts zu zeigen.
\item
@@ -2513,6 +2521,38 @@ Die Topologie von $X'$ sei hierbei die von $\L(X, \K)$, also die von der Norm $\
\end{enumerate}
\end{proof}
+\section{Separable Hilberträume}
+\begin{definition}
+ Ein metrischer Raum $(X,d)$ heißt \emph{separabel}, wenn es $U ⊂ X$ dicht
+ und abzählbar gibt.
+\end{definition}
+
+\begin{beispiele}
+ $ℝ^n, ℂ^n, \ell^2, L^2(\Omega)$ für $\Omega ⊂ ℝ^n$ offen sind separable Hilberträume.
+\end{beispiele}
+
+\begin{satz}
+ In einem separablen unendlich"=dimensionalen Hilbertraum $(X,\langle -,- \rangle)$ gilt
+ \begin{enumerate}
+ \item Jedes ONS in $X$ ist höchstens abzählbar.
+ \item
+ Sei $S = (\hat e_i)_{i ∈ ℕ}$ ein VONS in $X$. Dann existiert zu jeder
+ Folge $\alpha = (\alpha _k)_{k ∈ ℕ} ∈ \ell^2$ genau ein $x ∈ X$ mit $\langle \hat
+ e_k, x \rangle = \alpha _k, k ∈ ℕ$ (Satz von \emph{Riesz-Fischer}).
+ \item
+ $X$ ist isometrisch isomorph zum $\ell^2$. Insbesondere sind
+ $L^2(\Omega)$ und $\ell^2$ isometrisch isomorph.
+ \end{enumerate}
+\end{satz}
+
+
+\section{Riesz'scher Darstellungssatz und Lax-Milgram}
+Für einen topologischen linearen Raum $X$ ist der Dualraum $X' = \{x': X → \K, x' $ linear und stetig $\}$ definiert.
+Im Allgemeinen kann auch $X' = \{0\}$ gelten.
+Ist $X$ jedoch ein Hilbertraum, so ist stets $X' \ne \{0\}$, denn zu $y ∈ X$ ist durch $y'[x] \coloneq \langle y,x \rangle, x ∈ X$ jeweils ein $y' ∈ X'$ erklärt.
+Tatsächlich bekommt man dadurch sogar schon alle Elemente des Dualraums:
+
+
\chapter{Der Satz von Hahn-Banach und seine Konsequenzen}
\section{Fortsetzbarkeit linearer Funktionale}
@@ -2538,49 +2578,49 @@ Wir fragen uns, ob sich Abbildungen so erweitern, dass gewisse Eigenschaften (wi
\end{satz}
\begin{proof}
Zeigen wir zunächst die Existenz der Fortsetzung.
- Da $X_0$ dicht in $X$ ist, existiert zu jedem $x ∈ X$ eine Folge $(x_n)_{n ≥1}$, die ganz in $X_0$ liegt und gegen $x$ konvergiert.
+ Da $X_0$ dicht in $X$ ist, existiert zu jedem $x ∈ X$ eine Folge $(x_n)_{n \ge1}$, die ganz in $X_0$ liegt und gegen $x$ konvergiert.
Wir behaupten, dass $(A_0x_n)_{n ∈ ℕ}$ eine Cauchy-Folge in $Y$ ist.
Dazu beachte, dass
\[
- \norm{A_0 x_n - A_0 x_m}_{Y} \le \norm{A_0}_{\L(X_0,Y)} \norm{x_n-x_m} \xrightarrow[n,m → ∞]{} 0.
- \]
- Da $Y$ ein Banachraum ist, ist $(A_0x_n)_{n≥1}$ konvergiert, etwa gegen $y$.
- Wir setzen $Ax := y$.
- Zunächst ist $A$ wohldefiniert, denn wenn $(z_n)_{n ≥ 1}$ eine weitere Folge mit $\lim_{n → ∞} z_n = x$ ist, dann gilt
- $z_n - x_n \xrightarrow[n→∞]{} 0$ und
- \[
- \norm{A_0 z_n - y} \le \norm{A_0 z_n - A_0 x_n} + \norm{A_0 x_n - y}
- \le
- \norm{A_0} \norm{z_n - x_n} + \norm{A_0 x_n - y} \xrightarrow[n→∞]{} 0.
+ \norm{A_0 x_n - A_0 x_m}_{Y} \le \norm{A_0}_{\L(X_0,Y)} \norm{x_n-x_m} \xrightarrow[n,m → \infty ]{} 0.
\]
+ Da $Y$ ein Banachraum ist, ist $(A_0x_n)_{n\ge1}$ konvergiert, etwa gegen $y$.
+ Wir setzen $Ax \coloneq y$.
+ Zunächst ist $A$ wohldefiniert, denn wenn $(z_n)_{n \ge 1}$ eine weitere Folge mit $\lim_{n → \infty } z_n = x$ ist, dann gilt
+ $z_n - x_n \xrightarrow[n→\infty ]{} 0$ und
+ \begin{align*}
+ \norm{A_0 z_n - y} &\le \norm{A_0 z_n - A_0 x_n} + \norm{A_0 x_n - y} \\
+ & \le
+ \norm{A_0} \norm{z_n - x_n} + \norm{A_0 x_n - y} \xrightarrow[n→\infty ]{} 0.
+ \end{align*}
Offensichtlich ist $A$ eine Fortsetzung von $A_0$.
Dass $A$ linear ist, ist ebenfalls klar.
Zur Stetigkeit ist
- \[
- \norm{Ax}_Y = \norm{\lim_{n → ∞} A_0 x_n}_Y = \lim_{n → ∞} \norm{A_0 x_n}_{Y}
- \le
- \lim_{n → ∞} \norm{A_0}_{\L(X_0,Y)} \norm{x_n}_X = \norm{A_0} \norm{x}.
- \]
+ \begin{align*}
+ \norm{Ax}_Y &= \norm{\lim_{n → \infty } A_0 x_n}_Y = \lim_{n → \infty } \norm{A_0 x_n}_{Y} \\
+ &\le
+ \lim_{n → \infty } \norm{A_0}_{\L(X_0,Y)} \norm{x_n}_X = \norm{A_0} \norm{x}.
+ \end{align*}
Damit ist $A$ beschränkt, also auch stetig.
Es gilt $\norm{A_0}_{\L(X_0,Y)} = \norm{A}_{\L(X,Y)}$:
„$\ge$“ ist aus dem Vorherigen klar. Für die andere Ungleichung ist
\[
\norm{A}_{L(X,Y)} =
- \sup_{\norm{x ≤ 1}, x ∈ X} \norm{Ax}_{Y}
- ≥
- \sup_{\norm{x ≤ 1}, x ∈ X_0} \norm{Ax}_{Y} = \norm{A_0}_{\L(X_0,Y)}.
+ \sup_{\norm{x \le 1}, x ∈ X} \norm{Ax}_{Y}
+ \ge
+ \sup_{\norm{x \le 1}, x ∈ X_0} \norm{Ax}_{Y} = \norm{A_0}_{\L(X_0,Y)}.
\]
Für die Eindeutigkeit sei $B: X → Y$ eine weitere stetige, lineare Fortsetzung von $A_0$.
- Wie oben existiert zu jedem $x ∈ X$ eine Folge $(x_n)_{n ∈ ℕ} ⊂ X$ mit $\lim_{n → ∞} x_n = x$.
+ Wie oben existiert zu jedem $x ∈ X$ eine Folge $(x_n)_{n ∈ ℕ} ⊂ X$ mit $\lim_{n → \infty } x_n = x$.
Dann ist
\[
Ax_n = A_0 x_n = Bx_n \quad ∀ n ∈ ℕ
\]
und für $x ∈ X$
\[
- \norm{B_x - A_x} ≤ \norm{B_x - Bx_n} + \norm{Bx_n - Ax_n} + \norm{Ax_n - Ax} \xrightarrow[n→∞]{} 0,
+ \norm{B_x - A_x} \le \norm{B_x - Bx_n} + \norm{Bx_n - Ax_n} + \norm{Ax_n - Ax} \xrightarrow[n→\infty ]{} 0,
\]
da $A$ und $B$ stetig sind. Also $Bx = Ax$ für alle $x ∈ X$ und damit $B = A$.
\end{proof}
@@ -2600,18 +2640,18 @@ Ist $X_0$ nicht dicht in $X$, wird die Fortsetzung schwieriger.
Auf dem linearen Raum $X$ über $ℝ$ gebe es eine Abbildung $p: X → ℝ$ mit:
\begin{enumerate}[label=(\roman*)]
\item
- $p(αx) = αp(x)$ für alle $α ≥ 0, x ∈ X$ (positiv homogen)
+ $p(\alpha x) = \alpha p(x)$ für alle $\alpha \ge 0, x ∈ X$ (positiv homogen)
\item
- $p(x+y) ≤ p(x) + p(y)$ für alle $x, y ∈ X$ (subadditiv)
+ $p(x+y) \le p(x) + p(y)$ für alle $x, y ∈ X$ (subadditiv)
\end{enumerate}
Weiter seine $X_0$ ein linearer Teilraum von $X$ und $f_0 : X_0 → ℝ$ eine lineare Abbildung mit
\[
- ∀x ∈ X_0 : f_0(x) ≤ p(x).
+ ∀x ∈ X_0 : f_0(x) \le p(x).
\]
Dann gibt es eine lineare Fortsetzung $f: X → ℝ$ von $f_0$, welche die Ungleichung respektiert, das heißt
\[
- f|_{X_0} = f_0 \quad \text{und} \quad ∀x ∈ X: f(x) ≤ p(x).
+ f|_{X_0} = f_0 \quad \text{und} \quad ∀x ∈ X: f(x) \le p(x).
\]
\end{satz}
\begin{bemerkung-nn}
@@ -2619,30 +2659,30 @@ Ist $X_0$ nicht dicht in $X$, wird die Fortsetzung schwieriger.
\end{bemerkung-nn}
\begin{proof}
Schritt 1.
- Wir setzen $f_0$ auf $X_1 := X_0 \oplus \lspan{x_1}$ für ein $x_1 \not\in X$ (existiert immer solange $X_0 \subsetneqq X$).
+ Wir setzen $f_0$ auf $X_1 \coloneq X_0 \oplus \lspan{x_1}$ für ein $x_1 \not\in X$ (existiert immer solange $X_0 \subsetneqq X$).
Offenbar hat jedes $x ∈X_1$ eine eindeutig Darstellung als
- $ y = y + \alpha x_1 $, mit $y ∈ X_0$, $α ∈ ℝ$.
+ $ y = y + \alpha x_1 $, mit $y ∈ X_0$, $\alpha ∈ ℝ$.
Dann ist mit $c ∈ ℝ$ beliebig
\[
- f(x) = f(y + α(x_1)) := f_0(y) + αc
+ f(x) = f(y + \alpha (x_1)) \coloneq f_0(y) + \alpha c
\]
eine lineare Abbildung $X_1 → ℝ$, die $f_0$ fortsetzt.
- Wir müssen $c$ so wählen, dass $f(x) ≤ p(x)$ für alle $x ∈ X_1$, also $f_0(y) + αc \le p(y+αx_1)$ für alle $y ∈ X_0, α ∈ ℝ$.
+ Wir müssen $c$ so wählen, dass $f(x) \le p(x)$ für alle $x ∈ X_1$, also $f_0(y) + \alpha c \le p(y+\alpha x_1)$ für alle $y ∈ X_0, \alpha ∈ ℝ$.
Mit (i) ist diese Bedingung äquivalent zu zwei anderen Bedingungen:
\begin{enumerate}
\item
- Für $a > 0$: $f_0(y/α) + c ≤ p(y/α + x_1)$.
+ Für $a > 0$: $f_0(y/\alpha ) + c \le p(y/\alpha + x_1)$.
\item
- Für $α < 0$: $f_0(-y/α) - c ≤ p(-y/α - x_1)$
+ Für $\alpha < 0$: $f_0(-y/\alpha ) - c \le p(-y/\alpha - x_1)$
\end{enumerate}
- für alle $y ∈ X_0$. Der Fall $α = 0$ ist nach Annahme ohnehin klar.
+ für alle $y ∈ X_0$. Der Fall $\alpha = 0$ ist nach Annahme ohnehin klar.
Um diese Bedingungen erfüllen zu können, muss $c ∈ ℝ$ so gewählt werden, dass
\[
- ∀y_1, y_2 ∈ X_0: f_0(y_1) - p(y_1 - x_1) ≤ c ≤ p(y_2 + x_2) - f_0(y_2).
+ ∀y_1, y_2 ∈ X_0: f_0(y_1) - p(y_1 - x_1) \le c \le p(y_2 + x_2) - f_0(y_2).
\]
Das ist möglich, da
\[
- f_0(y_1) + f_0(y_2) = f_0(y_1+y_2) ≤ p(y_1 + y_2) = p(y_1 - x_1 + y_2 + x_1) ≤ p(y_1 - x_1)+p(y_2+x_1).
+ f_0(y_1) + f_0(y_2) = f_0(y_1+y_2) \le p(y_1 + y_2) = p(y_1 - x_1 + y_2 + x_1) \le p(y_1 - x_1)+p(y_2+x_1).
\]
Folglich gilt
\[
@@ -2654,13 +2694,13 @@ Ist $X_0$ nicht dicht in $X$, wird die Fortsetzung schwieriger.
Finde eine maximale Fortsetzung mit dem Lemma von Zorn.
Betrachte dazu
\[
- \{: X \supset D_g \supset X_0 → ℝ\}: g|_{X_0} = f_0 ∧ ∀x ∈ D_g: g(x) ≤ p(x) \}.
+ \{: X \supset D_g \supset X_0 → ℝ\}: g|_{X_0} = f_0 ∧ ∀x ∈ D_g: g(x) \le p(x) \}.
\]
Diese Menge ordnen wir mit $\succeq$ definiert durch
\[
h \succeq g \gdw h \text{ ist Fortsetzung von $g$}.
\]
- Nach dem Lemma von Zorn existiert eine maximale Fortsetzung $g^*$ von $f_0$ mit $g^*(x) ≤ p(x)$ für alle $x ∈ X$.
+ Nach dem Lemma von Zorn existiert eine maximale Fortsetzung $g^*$ von $f_0$ mit $g^*(x) \le p(x)$ für alle $x ∈ X$.
Wäre $D_{g^*}$ nicht $X$, so verfahre wie in Schritt 1 im Widerspruch zur Maximalität.
Damit hat $g^*$ die gewünschten Eigenschaften.
\end{proof}
@@ -2668,7 +2708,7 @@ Ist $X_0$ nicht dicht in $X$, wird die Fortsetzung schwieriger.
\begin{bemerkung-nn}
\begin{enumerate}
\item
- Ohne die Zusatzforderung $f(x) ≤ p(x)$ für alle $x ∈X$ ist die lineare Fortsetzbarkeit trivial.
+ Ohne die Zusatzforderung $f(x) \le p(x)$ für alle $x ∈X$ ist die lineare Fortsetzbarkeit trivial.
\item
Eine Fortsetzung für lineare Funktionale $f_0: X_0 → \K = ℂ$ ist analog möglich. % yos IV 4
\end{enumerate}
@@ -2681,53 +2721,53 @@ Ist $X_0$ nicht dicht in $X$, wird die Fortsetzung schwieriger.
Dann existiert zu jedem $x_0 \not\in M$ ein $f ∈ X'$ mit
\[
- f(x_0) > 1 ∧ ∀ x ∈ M: f(x) ≤ 1.
+ f(x_0) > 1 ∧ ∀ x ∈ M: f(x) \le 1.
\]
\end{satz}
-Die Hyperebene $H = \{ x ∈ X: f(x) = 1 + ε\}$ für $0 < ε < f(x_0) < 1$ trennt also $x_0$ und $M$.
+Die Hyperebene $H = \{ x ∈ X: f(x) = 1 + \epsilon \}$ für $0 < \epsilon < f(x_0) < 1$ trennt also $x_0$ und $M$.
\begin{proof}
- Setze $2r := \inf_{y ∈ M} \norm{y - x_0}$ (positiv, da $M$ abgeschlossen).
- Sei $N := \cl{M + \cl{B_r(0)}} = \cl{\{ z = y + u: y ∈ M, u ∈ \cl{B_r(0)}\}} ⊂ X$.
+ Setze $2r \coloneq \inf_{y ∈ M} \norm{y - x_0}$ (positiv, da $M$ abgeschlossen).
+ Sei $N \coloneq \cl{M + \cl{B_r(0)}} = \cl{\{ z = y + u: y ∈ M, u ∈ \cl{B_r(0)}\}} ⊂ X$.
Dann ist (i) $N$ abgeschlossen und (ii) $\cl{B_r(0)} ⊂ N$, da $0 ∈ M$, insbesondere ist $0 ∈ N^\circ$.
(iii) ist $N$ konvex: Es genügt, zu zeigen, dass $A = M + B_r(0)$ konvex ist, denn dann ist auch $\cl A$ konvex.
- Sei $x _i = y_i + v_i, y_i ∈ M, v_i ∈ \cl{B_r(0)}, i=1,2$ und $α ∈ (0,1)$. Dann ist
+ Sei $x _i = y_i + v_i, y_i ∈ M, v_i ∈ \cl{B_r(0)}, i=1,2$ und $\alpha ∈ (0,1)$. Dann ist
\[
- αx_1 + (1-α)x_2 = \underbrace{[αy_1 + (1-α)y_2]}_{∈ M} + \underbrace{[αu_1+(1-α)v_2]}_{∈ \cl{B_r(0)}}.
+ \alpha x_1 + (1-\alpha )x_2 = \underbrace{[\alpha y_1 + (1-\alpha )y_2]}_{∈ M} + \underbrace{[\alpha u_1+(1-\alpha )v_2]}_{∈ \cl{B_r(0)}}.
\]
(iv) ist $x_0 \not\in N$.
- Angenommen, $x_0 ∈ N$. Dann existiert eine Folge $z_n = y_n + u_n$ in $A$ mit $z_n → x_0 (n→∞)$.
+ Angenommen, $x_0 ∈ N$. Dann existiert eine Folge $z_n = y_n + u_n$ in $A$ mit $z_n → x_0 (n→\infty )$.
Dann ist für $n_0$ hinreichend groß
\[
- \frac r 2 > \norm{z_{n_0} - x_0} = \norm{y_{n_0 - x_0} + u_{n_0}} ≥ |\underbrace{\norm{y_{n_0-x_0}}}_{≥ 2r} - \underbrace{\norm{u_{n_0}}}_{≤ r}| ≥ r.
+ \frac r 2 > \norm{z_{n_0} - x_0} = \norm{y_{n_0 - x_0} + u_{n_0}} \ge |\underbrace{\norm{y_{n_0-x_0}}}_{\ge 2r} - \underbrace{\norm{u_{n_0}}}_{\le r}| \ge r.
\]
Verwende nun das Minkowski-Funktional
\[
- p_N(x) := \inf \{ρ > 0: ρ^{-1} x ∈ N\}, \quad x ∈ X.
+ p_N(x) \coloneq \inf \{ρ > 0: ρ^{-1} x ∈ N\}, \quad x ∈ X.
\]
Dieses hat die Eigenschaften
\begin{enumerate}
\item
- $p_N(αx) = αp_n(x),\quad α ≥ 0, x ∈ X$ (positiv homogen)
+ $p_N(\alpha x) = \alpha p_n(x),\quad \alpha \ge 0, x ∈ X$ (positiv homogen)
\item
- $p_N(x+y) ≤ p_N(x) + p_N(y), \quad x, y ∈ X$ (subadditiv)
+ $p_N(x+y) \le p_N(x) + p_N(y), \quad x, y ∈ X$ (subadditiv)
\item
- $p_N(x) ≤ 1 \iff x ∈ N$
+ $p_N(x) \le 1 \iff x ∈ N$
\item
- Ist zusätzlich $\cl{B_r(0)} ⊂ N$, so gilt $p_nNx) ≤ r^{-1}\norm x$ für alle $x ∈ X$.
+ Ist zusätzlich $\cl{B_r(0)} ⊂ N$, so gilt $p_nNx) \le r^{-1}\norm x$ für alle $x ∈ X$.
\end{enumerate}
- Sei nun $X_0 := \lspan\{x_0\}$ und $f_0 : X_0 → ℝ$ linear definiert durch $f_0(x_0) := p_N(x_0)$.
- Wir behauptung, dass $f_0 (x) ≤ p_N(x)$ für alle $x = λx_0 ∈ X_0$.
- Falls $λ ≥ 0$, so ist $f_0(x) = f_0(λx_0) = λp_N(x_0) = p_N(λx_0) = p_N(x)$.
- Falls $λ < 0$, so ist wegen $p_n ≥ 0$ ohnehin $f_0(λx_0) = λp_N(x_0) ≤ 0 ≤ p_N(λx_0)$.
+ Sei nun $X_0 \coloneq \lspan\{x_0\}$ und $f_0 : X_0 → ℝ$ linear definiert durch $f_0(x_0) \coloneq p_N(x_0)$.
+ Wir behauptung, dass $f_0 (x) \le p_N(x)$ für alle $x = \lambda x_0 ∈ X_0$.
+ Falls $\lambda \ge 0$, so ist $f_0(x) = f_0(\lambda x_0) = \lambda p_N(x_0) = p_N(\lambda x_0) = p_N(x)$.
+ Falls $\lambda < 0$, so ist wegen $p_n \ge 0$ ohnehin $f_0(\lambda x_0) = \lambda p_N(x_0) \le 0 \le p_N(\lambda x_0)$.
Da $p_N$ die Bedingungen (i) und (ii) aus Hahn-Banach erfüllt,
- gibt es eine lineare Fortsetzung $f$ von $f_0$ mit $f(x) ≤ p_N(x)$ für alle $x ∈ X$.
+ gibt es eine lineare Fortsetzung $f$ von $f_0$ mit $f(x) \le p_N(x)$ für alle $x ∈ X$.
Nun ist $f$ stetig, also $f ∈ X'$, denn für alle $x ∈ X$ gilt
\begin{multline*}
- |f(x) = \max\{f(x), -f(x)\} = \max\{f(x),f(-x)\} ≤ \max\{p_N(x),p_N(-x)\} \\
- ≤ \max\left\{\frac{\norm{x}}{r},\frac{\norm{-x}}{r}\right\} = \frac{\norm x}{r}.
+ |f(x) = \max\{f(x), -f(x)\} = \max\{f(x),f(-x)\} \le \max\{p_N(x),p_N(-x)\} \\
+ \le \max\left\{\frac{\norm{x}}{r},\frac{\norm{-x}}{r}\right\} = \frac{\norm x}{r}.
\end{multline*}
Außerdem erfüllt $f$ die Gleichung 3.1 (?), denn
@@ -2736,27 +2776,27 @@ Die Hyperebene $H = \{ x ∈ X: f(x) = 1 + ε\}$ für $0 < ε < f(x_0) < 1$ tren
\]
und für $x ∈ M ⊂ N$ gilt
\[
- f(x) ≤ p_N(x) ≤ 1.
+ f(x) \le p_N(x) \le 1.
\]
\end{proof}
\section{Einbettung von $X$ in seinen Bidualraum}
Zunächst zur Motivation: Sei $X$ ein normierter linearer Raum.
Dann existiert $X'$ und ist ein Banachraum.
-Aber dann existiert auch $X'' := (X')'$ und ist ebenfalls ein Banachraum.
-Unser ziel wird es nun sein, $X$ in $X''$ einzubetten.
+Aber dann existiert auch $X'' \coloneq (X')'$ und ist ebenfalls ein Banachraum.
+Unser Ziel wird es nun sein, $X$ in $X''$ einzubetten.
\begin{definition}
Die kanonische Abbildung $J_0: X → X''$ ist definiert durch
\[
- J_0(x) [x'] = \langle \langle J_0(x), x' \rangle \rangle_{X''×X'} := \langle \langle x', x \rangle \rangle_{X'×X} = x'[x] ∈ \K
+ J_0(x) [x'] = \lAngle J_0(x), x' \rAngle_{X''×X'} \coloneq \lAngle x', x \rAngle_{X'×X} = x'[x] ∈ \K
\]
für $x ∈ X, x' ∈ X'$.
Offensichtlich gilt für $x ∈ X$ fest $J_0(x): X' → \K$ linear, aber $J_0(x)$ ist auch stetig bzw beschränkt:
Dazu ist
\[
- |J_0(x)[x']| = | \langle \langle x',x \rangle \rangle ≤ \norm{x'}_{X'} \underbrace{\norm{x}_X}_{=: M}.
+ |J_0(x)[x']| = | \langle \langle x',x \rAngle \le \norm{x'}_{X'} \underbrace{\norm{x}_X}_{=: M}.
\]
Also ist $J_0(x) ∈ X''$, also insbesondere $J_0$ wohldefiniert.
Wegen der linearität von $J_0$ in $x$ schreiben wir statt $J_0(x)$ auch $J_0 x$.
@@ -2774,29 +2814,28 @@ Unser ziel wird es nun sein, $X$ in $X''$ einzubetten.
Zur Injektivität: Seien $x_1, x_2 ∈ X$ mit $J_0x_1 = J_0x_2$.
Dann ist für jedes $x' ∈ X'$
\[
- \langle \langle x',x_1 \rangle \rangle = J_0 x_1[x'] = J_0x_2[x'] = \langle \langle x', x_2 \rangle \rangle,
+ \lAngle x',x_1 \rAngle = J_0 x_1[x'] = J_0x_2[x'] = \lAngle x', x_2 \rAngle,
\]
also wegen Linearität von $x'$
\[
- \langle \langle x', x_1-x_2 \rangle \rangle = 0.
+ \lAngle x', x_1-x_2 \rAngle = 0.
\]
Mit Folgerung 2.3(1) folgt $x_1-x_2 = 0$.
Zur Isometrieeigenschaft bleibt zu zeigen: $\norm{J_0x} = \norm{x}$ für alle $x ∈ X''$.
- „≤“: Aus (4.1) folgt bereits
+ „$\le$“: Aus (4.1) folgt bereits
\[
- \norm{J_0(x)}_{X''} = \sup_{\norm{x'} ≤ 1} |J_0(x)[x'] ≤ \norm{x}_X.
+ \norm{J_0(x)}_{X''} = \sup_{\norm{x'} \le 1} |J_0(x)[x'] \le \norm{x}_X.
\]
- „≥“: Zu $x_0 ∈ X$ existiert nach Korollar 2.1 ein $x_0' ∈ X'$ mit
+ „$\ge$“: Zu $x_0 ∈ X$ existiert nach Korollar 2.1 ein $x_0' ∈ X'$ mit
$\norm{x_0'}_{X'} = 1$ und $x_0'[x_0]= \norm{x_0}$.
Also folgt
\[
- \underbrace{|J_0x_0[x_0']|}_{≤ \norm{J_0x_0}_{X''}} = \langle \langle x_0', x_0 \rangle \rangle = \norm{x_0}.
+ \underbrace{|J_0x_0[x_0']|}_{\le \norm{J_0x_0}_{X''}} = \lAngle x_0', x_0 \rAngle = \norm{x_0}.
\]
- Da $x_0$ beliebig war, gilt $\norm{J_0x}_{X''} ≥ \norm{x}$.
+ Da $x_0$ beliebig war, gilt $\norm{J_0x}_{X''} \ge \norm{x}$.
\end{proof}
-
\begin{definition}
Ein Banachraum $X$ heißt \emph{reflexiv}, wenn $J_0$ surjektiv ist, also $X$ und $X''$ isomorph sind vermöge $J_0$.
\end{definition}
@@ -2831,9 +2870,9 @@ Unser ziel wird es nun sein, $X$ in $X''$ einzubetten.
\end{bemerkung-nn}
\begin{definition}
- Eine Folge $(x_n)_{n ∈ ℕ}$ in einem normierten Raum $X$ heißt \emph{schwach konvergent} gegen $x ∈ X$ (in Zeichen: $x_n \rightharpoonup x$ für $n → ∞$), wenn
+ Eine Folge $(x_n)_{n ∈ ℕ}$ in einem normierten Raum $X$ heißt \emph{schwach konvergent} gegen $x ∈ X$ (in Zeichen: $x_n \xrightharpoonup[n → \infty ]{} x$), wenn
\[
- \lim_{n → ∞} x'[x_n] = x'[x]
+ \lim_{n → \infty } x'[x_n] = x'[x]
\]
für alle $x' ∈ X'$ gilt.
\end{definition}
@@ -2845,12 +2884,12 @@ Unser ziel wird es nun sein, $X$ in $X''$ einzubetten.
\begin{beispiel-nn}
Für $(\hat e_i)_{i ∈ ℕ}$ Hilbertraumbasis in einem separablem Hilbertraum $X$ gilt
\[
- \hat e_i \rightharpoonup 0 ∈ X (i → ∞)
+ \hat e_i \rightharpoonup 0 ∈ X (i → \infty )
\]
\end{beispiel-nn}
\begin{bemerkung-nn}
- $(\hat e_i)_{i ∈ ℕ}$ ist nicht konvergent in der Normtopologie, die Folge ist noch nicht mal Cauchy, insbesondere ist $\norm{\hat e_i - 0} \not \to 0 (i → ∞)$.
+ $(\hat e_i)_{i ∈ ℕ}$ ist nicht konvergent in der Normtopologie, die Folge ist noch nicht mal Cauchy, insbesondere ist $\norm{\hat e_i - 0} \not\rightarrow 0 (i → \infty )$.
\end{bemerkung-nn}
\begin{proof}
@@ -2858,12 +2897,13 @@ Unser ziel wird es nun sein, $X$ in $X''$ einzubetten.
\[
X' = \{ x' : x' ∈ X'\} = \{ J_X(y) : y ∈ X\}.
\]
- Zu zeigen ist $\lim\limits_{i → ∞}x'[\hat e_i] = x'[0]$ für alle $x' ∈ X'$, also äquivalent
- $\lim\limits_{i → ∞} J_x(y)[\hat e_i] = J_x(y)[0]$ für alle $y ∈ X$ bzw. $\lim\limits_{i → ∞} \langle y, \hat e_i \rangle = \langle y, 0 \rangle$ für alle $y ∈ X$.
+ Zu zeigen ist $\lim\limits_{i → \infty }x'[\hat e_i] = x'[0]$ für alle $x' ∈ X'$, also äquivalent
+ $\lim\limits_{i → \infty } J_x(y)[\hat e_i] = J_x(y)[0]$ für alle $y ∈ X$ bzw. $\lim\limits_{i → \infty } \langle y, \hat e_i \rangle = \langle y, 0 \rangle$ für alle $y ∈ X$.
- Sei also $y ∈ X$ fest gewählt. Dann ist $y = \sum_{i=1}^∞α_i \hat e_i$ mit $α_i = \langle \hat e_i, y \rangle$.
- Es gilt $\sum_{i=1}^∞ |α_i|^2 < ∞$ (vgl Def 4.2.12).
- Damit folgt $α_i = \langle \hat e_i, y \rangle → 0 (i → ∞)$, weil $α ∈ \ell^2$. Damit folgt die Schache Konvergenz von $(\hat e_i)_{i ∈ ℕ}$.
+ Sei also $y ∈ X$ fest gewählt. Dann ist $y = \sum_{i=1}^\infty \alpha _i \hat e_i$ mit $\alpha _i = \langle \hat e_i, y \rangle$.
+ Es gilt $\sum_{i=1}^\infty |\alpha _i|^2 < \infty $ (vgl Def 4.2.12).
+ Damit folgt $\alpha _i = \langle \hat e_i, y \rangle → 0 (i → \infty )$, weil $\alpha ∈ \ell^2$.
+ Damit folgt die Schwache Konvergenz von $(\hat e_i)_{i ∈ ℕ}$.
\end{proof}
@@ -2879,11 +2919,7 @@ Unser ziel wird es nun sein, $X$ in $X''$ einzubetten.
\end{satz}
-
-
-
-\end{document}
%%% Local Variables:
%%% mode: latex
-%%% TeX-master: "funkana"
+%%% TeX-master: "funkana-ebook"
%%% End: \ No newline at end of file
diff --git a/latexmkrc b/latexmkrc
index 8354309..9dbd136 100644
--- a/latexmkrc
+++ b/latexmkrc
@@ -1,4 +1,4 @@
-#$pdflatex = 'lualatex --recorder --synctex=1 --shell-escape -halt-on-error %O %S && cp %D ./pdf/%R.pdf';
-$pdflatex = 'pdflatex --recorder --synctex=1 --shell-escape -halt-on-error %O %S && cp %D ./pdf/%R.pdf';
+$pdflatex = 'xelatex --recorder --synctex=1 --shell-escape -halt-on-error %O %S && cp %D ./pdf/%R.pdf';
+#$pdflatex = 'pdflatex --recorder --synctex=1 --shell-escape -halt-on-error %O %S && cp %D ./pdf/%R.pdf';
$pdf_mode = 1;
$out_dir = "build"; \ No newline at end of file
diff --git a/motivation.tex b/motivation.tex
new file mode 100644
index 0000000..b7c631b
--- /dev/null
+++ b/motivation.tex
@@ -0,0 +1,68 @@
+\section*{Motivation} \markboth{}{Motivation}
+In der klassischen Analyis haben wir Funktionen im $\K^n$, wobei $\K$ entweder $ℝ$ oder $ℂ$ ist, untersucht.
+Dabei war das Betrachten von Eigenschaften wie Konvergenz, Stetigkeit und Differenzierbarkeit sehr nützlich.
+Die Funktionalanalysis beschäftigt sich nun mit vergleichbaren Problemen in üblicherweise unendlich"=dimensionalen Funktionenräumen.
+Hierfür werden wir versuchen, die aus der klassischen Analysis bekannten Untersuchungsmethoden zu verallgemeinern.
+Doch zunächst ein paar Probleme, für deren Lösung man die Funktionalanalysis benötigt.
+
+\begin{problem-nn}
+ Ein klassisches Beispiel aus der Variationsrechnung:
+ Wir wollen die Funktion
+ \[
+ f(u) = \int_0^\pi |u'(x)|^2 dx
+ \]
+ unter den Nebenbedingungungen $u(0) = u(\pi ) = 0$ und $\int_0^\pi |u(x)|^2 dx = 1$ minimieren.
+ In der klassischen Analysis haben wir für Minimierungsprobleme mit Nebenbedingungungen Lagrange-Multiplikatoren genutzt.
+ Im unendlich"=dimensionalen Fall ist das jedoch nicht so einfach.
+ Wir betrachten $f : Y → ℝ$ wie oben, wobei $Y$ eine Teilmenge des unendlich"=dimensionalen Funktionenraums
+ \[
+ X = \left\{ u ∈ C^1[0,\pi ]: u(0) = u(\pi ) = 0 \right\}
+ \]
+ ist, die durch
+ \[
+ Y = \left\{ u ∈ X: \int_0^\pi |u(x)|^2 dx = 1 \right\}
+ \]
+ gegeben ist.
+ Zwar ist $Y$ (in der $\L^2([0,\pi ])$-Metrik) beschränkt und abgeschlossen, jedoch nicht kompakt.
+\end{problem-nn}
+\begin{problem-nn}
+ Sei $\mathcal T = \{ 1, \cos t, \sin t, \cos (2t), \sin (2t), … \} =
+ \{\phi_i\}_{i ∈ ℕ}$. Dann ist bekanntlich
+ \[
+ \langle \phi_i, \phi_j \rangle = ∫_0^{2\pi } φ_i(t) φ_j(t) dt = 2\pi \delta _{i,j},
+ \]
+ wobei $\delta _{i,j}$ das Kronecker-Delta bezeichne.
+ Also lässt sich durch Normierung ein Orthonormalsystem aus $\mathcal T$ gewinnen.
+ Jetzt fragen wir uns, ob sich jede $2\pi $-periodische Funktion $u$ bezüglich eines geeigneten Konvergenzbegriffs in eine Reihe $u = \sum_{i ∈ ℕ} \alpha _i φ_i$ mit $\alpha _i ∈ ℝ$ entwickeln können.
+ Bereits bekannt ist, dass das für das entsprechende endlich-dimensionale Problem geht: Sei $T = \{ e_1,…,e_n\}$ die kanonische Standardbasis des $ℝ^n$
+ Dann gilt bekanntlich
+ \[
+ \langle e_i, e_j \rangle_{ℝ^n} = \delta _{i,j}
+ \]
+ und für jedes $x ∈ ℝ^n$ ist
+ \[
+ x = \sum_{i=1}^n \alpha _i e_i, \quad \alpha _i = \langle x, e_i \rangle_{ℝ^n}.
+ \]
+ Wir fragen uns nach den Zusammenhängen zwischen den Problemen im endlich- und unendlich"=dimensionalen.
+\end{problem-nn}
+\begin{problem-nn}
+ Das Biegemoment eines Trägers kann man als Randwertaufgabe (gesucht ist $u: [0,1] → ℝ$, gegeben sind $p,r: [0,1] → ℝ$)
+ \[
+ u''(t) + p(t) u(t) = r(t), \quad u(0) = u(1) = 0
+ \]
+ bestimmen. Mit Hilfte der sogenannten Green'schen Funktion lässt sich diese Randwertaufgabe in eine Integralgleichung
+ \[
+ (T_u)(t) \coloneq ∫_0^1 G(t,s) \big(r(s)-p(s)u(s)\big) ds = u
+ \]
+ umwandeln. Das heißt, man sucht einen Fixpunkt eines Integraloperators $T$ in einer geeigneten Menge von Funktionen.
+\end{problem-nn}
+
+Diese Probleme lassen sich mit der klassischen Analysis nicht mehr behandeln.
+In der Funktionalanalysis behandeln wir nun im Wesentlichen „Analysis in $\infty$-dimensionalen Räumen“ (meist Funktionenräume).
+Das heißt, wir wollen jetzt anstelle des $\K^n$ allgemeinere Räume betrachten, die jodoch immer noch folgende beide Charakteristika aufweisen:
+\begin{enumerate}
+\item Die lineare Struktur (das heißt, Elemente lassen sich addieren und mit einem Skalar multiplizieren)
+\item Die topologische Struktur (also insbesondere ein Konvergenzbegriff)
+\end{enumerate}
+
+Unser Ziel ist es zunächst, die beiden Strukturen zu erarbeiten. \ No newline at end of file
diff --git a/pdf/funkana.pdf b/pdf/funkana.pdf
deleted file mode 100644
index c8046a5..0000000
--- a/pdf/funkana.pdf
+++ /dev/null
Binary files differ
diff --git a/ref.bib b/ref.bib
new file mode 100644
index 0000000..7507e59
--- /dev/null
+++ b/ref.bib
@@ -0,0 +1,153 @@
+@book {MR0450957,
+ AUTHOR = {Adams, Robert A.},
+ TITLE = {Sobolev spaces},
+% NOTE = {Pure and Applied Mathematics, Vol. 65},
+ PUBLISHER = {Academic Press, New York-London},
+ YEAR = {1975},
+% PAGES = {xviii+268},
+ MRCLASS = {46E35},
+ MRNUMBER = {0450957},
+MRREVIEWER = {A. Kufner},
+}
+@book{alt2002lineare,
+ title={Lineare Funktionalanalysis: Eine anwendungsorientierte Einf{\"u}hrung},
+ author={Alt, H.W.},
+ isbn={9783540439479},
+ series={Springer Lehrbuch},
+ year={2002},
+ publisher={Springer}
+}
+@book {MR0217549,
+ AUTHOR = {Bachman, George and Narici, Lawrence},
+ TITLE = {Functional analysis},
+ PUBLISHER = {Academic Press, New York-London},
+ YEAR = {1966},
+% PAGES = {xiv+530},
+ MRCLASS = {46.00 (47.00)},
+ MRNUMBER = {0217549},
+MRREVIEWER = {R. G. Bartle},
+}
+@book {MR787404,
+ AUTHOR = {Deimling, Klaus},
+ TITLE = {Nonlinear functional analysis},
+ PUBLISHER = {Springer-Verlag, Berlin},
+ YEAR = {1985},
+% PAGES = {xiv+450},
+ ISBN = {3-540-13928-1},
+ MRCLASS = {47-01 (47Hxx 55M25 58-01 58Cxx)},
+ MRNUMBER = {787404},
+MRREVIEWER = {Joachim Naumann},
+ URL = {https://doi.org/10.1007/978-3-662-00547-7},
+}
+
+@book {MR1787146,
+ AUTHOR = {Werner, Dirk},
+ TITLE = {Funktionalanalysis},
+ PUBLISHER = {Springer-Verlag, Berlin},
+ YEAR = {2000},
+% PAGES = {xii+501},
+ ISBN = {3-540-67645-7},
+ MRCLASS = {46-01 (47-01)},
+ MRNUMBER = {1787146},
+MRREVIEWER = {Manfred Wolff},
+}
+@book {MR617913,
+ AUTHOR = {Yosida, K\^osaku},
+ TITLE = {Functional analysis},
+ SERIES = {Grundlehren der Mathematischen Wissenschaften},
+ VOLUME = {123},
+ PUBLISHER = {Springer-Verlag, Berlin-New York},
+ YEAR = {1980},
+% PAGES = {xii+501},
+ ISBN = {3-540-10210-8},
+ MRCLASS = {46-01 (47-01)},
+ MRNUMBER = {617913},
+}
+@book {MR2380292,
+ AUTHOR = {Heuser, Harro},
+ TITLE = {Funktionalanalysis},
+ SERIES = {Mathematische Leitf\"aden},
+ NOTE = {Theorie und Anwendung},
+ PUBLISHER = {B. G. Teubner, Stuttgart},
+ YEAR = {2006},
+% PAGES = {696},
+ ISBN = {978-3-8351-0026-8; 3-8351-0026-2},
+ MRCLASS = {46-01 (47-01)},
+ MRNUMBER = {2380292},
+ URL = {https://doi.org/10.1007/978-3-8351-9072-6},
+}
+@book {MR0365062,
+ AUTHOR = {Rudin, Walter},
+ TITLE = {Functional analysis},
+ NOTE = {McGraw-Hill Series in Higher Mathematics},
+ PUBLISHER = {McGraw-Hill Book Co., New York-D\"usseldorf-Johannesburg},
+ YEAR = {1973},
+% PAGES = {xiii+397},
+ MRCLASS = {46-01},
+ MRNUMBER = {0365062},
+MRREVIEWER = {F. Smithies},
+}
+@book {MR2953760,
+ AUTHOR = {Meise, Reinhold and Vogt, Dietmar},
+ TITLE = {Einf\"uhrung in die {F}unktionalanalysis},
+ PUBLISHER = {Vieweg + Teubner Verlag, Wiesbaden},
+ YEAR = {2011},
+% PAGES = {x+273},
+ ISBN = {978-3-8348-1872-0},
+ MRCLASS = {46-02 (46-01 46A04 46A45 46B99 47-01)},
+ MRNUMBER = {2953760},
+ URL = {https://doi.org/10.1007/978-3-8348-8654-5},
+}
+@book {MR564653,
+ AUTHOR = {Taylor, Angus Ellis and Lay, David C.},
+ TITLE = {Introduction to functional analysis},
+ PUBLISHER = {John Wiley \&\ Sons, New York-Chichester-Brisbane},
+ YEAR = {1980},
+% PAGES = {xi+467},
+ ISBN = {0-471-84646-5},
+ MRCLASS = {46-01 (47-01)},
+ MRNUMBER = {564653},
+}
+
+@book {MR1347692,
+ AUTHOR = {Zeidler, Eberhard},
+ TITLE = {Applied functional analysis},
+ SERIES = {Applied Mathematical Sciences},
+ VOLUME = {109},
+% NOTE = {Main principles and their applications},
+ PUBLISHER = {Springer-Verlag, New York},
+ YEAR = {1995},
+% PAGES = {xvi+404},
+ ISBN = {0-387-94422-2},
+ MRCLASS = {00A05 (35-01 46-01 47-01 49-01 58-01)},
+ MRNUMBER = {1347692},
+MRREVIEWER = {Jean Mawhin},
+}
+@book {MR0467224,
+ AUTHOR = {Wloka, Josef},
+ TITLE = {Funktionalanalysis und {A}nwendungen},
+% NOTE = {de Gruyter Lehrbuch},
+ PUBLISHER = {Walter de Gruyter, Berlin-New York},
+ YEAR = {1971},
+% PAGES = {291},
+ MRCLASS = {46-01},
+ MRNUMBER = {0467224},
+}
+@book {MR969367,
+ AUTHOR = {Dautray, Robert and Lions, Jacques-Louis},
+ TITLE = {Mathematical analysis and numerical methods for science and
+ technology. {V}ol. 2},
+% NOTE = {Functional and variational methods,
+% With the collaboration of Michel Artola, Marc Authier,
+% Philippe B\'enilan, Michel Cessenat, Jean Michel Combes, H\'el\`ene
+% Lanchon, Bertrand Mercier, Claude Wild and Claude Zuily,
+% Translated from the French by Ian N. Sneddon},
+ PUBLISHER = {Springer-Verlag, Berlin},
+ YEAR = {1988},
+% PAGES = {xvi+561},
+ ISBN = {3-540-19045-7},
+ MRCLASS = {00A05},
+ MRNUMBER = {969367},
+ URL = {https://doi.org/10.1007/978-3-642-61566-5},
+}
+ \ No newline at end of file
diff --git a/skript.cls b/skript.cls
index 1eddd77..cb2247b 100644
--- a/skript.cls
+++ b/skript.cls
@@ -9,27 +9,35 @@
\RequirePackage{tikz-cd}
\tikzcdset{arrow style=tikz, diagrams={>=stealth}}
-\RequirePackage{ifluatex}
+\RequirePackage{ifxetex,ifluatex}
+\newif\ifxetexorluatex
+\ifxetex
+ \xetexorluatextrue
+\else
+ \ifluatex
+ \xetexorluatextrue
+ \else
+ \xetexorluatexfalse
+ \fi
+\fi
-\RequirePackage{csquotes}
-\RequirePackage{hyphenat}
-\RequirePackage{titlesec}
+\RequirePackage{scrlayer-scrpage}
\RequirePackage{gitinfo}
-\RequirePackage{mathtools}
-\RequirePackage{amsmath, amssymb}
+\RequirePackage{mathtools}
+\RequirePackage{amsmath, amssymb}
-\ifluatex
- \RequirePackage{polyglossia}
+\ifxetexorluatex
+ \RequirePackage[babelshorthands]{polyglossia}
\setdefaultlanguage{german}
- \RequirePackage{libertine}
+ \setmainfont{Libertinus Serif}
\RequirePackage[warnings-off={mathtools-colon,mathtools-overbracket}]{unicode-math}
- \setmathfont{TeX Gyre Pagella Math}
- \setmathfont[range=\setminus]{XITS Math}
- \setmathfont[range={\sum}]{TeX Gyre Termes Math}
- \setmathfont[range={\int}]{XITS Math}
- \setmathfont{Latin Modern Math}[range={cal,bfcal},StylisticSet=1]
+ \setmathfont[AutoFakeBold]{Libertinus Math}
+ \setmathfont[AutoFakeBold,range={\rightarrow,\leftarrow,\rightharpoonup,\rightharpoondown,\leftharpoondown,\leftharpoonup}]{texgyretermes-math.otf}
+ \setmathfont[AutoFakeBold,range={\Vert,\lbrace,\rbrace,\vert}]{xits-math.otf}
+ \setmathfont[AutoFakeBold,range={\BbbN,\BbbR,\BbbC,\BbbZ,\mathbb,\int}]{XITS Math}
+ % \setmathfont{latinmodern-math.otf}[range={cal,bfcal}]
\else
\RequirePackage[ngerman]{babel}
\RequirePackage[utf8]{inputenc}
@@ -38,37 +46,39 @@
\RequirePackage[libertine]{newtxmath}
\RequirePackage[cal=zapfc,bb=boondox]{mathalfa}
\RequirePackage[T1]{fontenc}
+ \def\lAngle{\langle\langle}
+ \def\rAngle{\rangle\rangle}
\fi
% fonts
\setkomafont{disposition}{\rmfamily}
-\RequirePackage{mathtools}
-\RequirePackage{amsmath, amsfonts, amssymb}
-
% fonts
-\RequirePackage{textcomp} % für erweiterten Text-Symbolvorrat
\RequirePackage{setspace}
-\setstretch{1.05}
+\setstretch{1.1}
\setlength\parskip{4pt}
\setlength\parindent{0pt}
\RequirePackage[amsmath, thmmarks, framed]{ntheorem}
\RequirePackage[framemethod=tikz]{mdframed}
-\RequirePackage{versions}
-\RequirePackage{hyperref}
+\RequirePackage[unicode,colorlinks]{hyperref}
\RequirePackage[capitalise, nameinlink]{cleveref}
-\RequirePackage{scrpage2}
-\RequirePackage{authoraftertitle}
-
\pagestyle{scrheadings}
\clearscrheadfoot
\ohead{\headmark}
-\cfoot{-- \pagemark~--}
+\cfoot{\pagemark}
+\ifoot{\tiny Revision\gitVtags: \gitAbbrevHash{} (\gitAuthorDate)}
\automark{section}
+\RequirePackage[
+ backend=biber,
+ style=alphabetic,
+ giveninits,
+ url=true,
+ ]{biblatex}
+
\usepackage{enumitem}
\setenumerate{label=(\alph*)}
@@ -118,8 +128,8 @@
\def\newdef#1#2{\newtheorem{#1}[defsatzusw]{#2}\newtheorem*{#1-nn}{#2}}
\theoremseparator{.}
\numberwithin{defsatzusw}{section}
-% \theoremsymbol{\ensuremath{\diamond}}
% kursive schrift
+\theorembodyfont{\normalfont}
\theoremstyle{mychange}
\newthm{satz}{Satz}
\newthm{lemma}{Lemma}
@@ -148,12 +158,11 @@
\theorembodyfont{\normalfont}
\theoremseparator{.}
% \theoremsymbol{\scalebox{0.8}{\ensuremath{\blacksquare}}}
-\theoremsymbol{\nolinebreak[1]\hspace*{.5em plus 1fill}\ensuremath{\blacksquare}}
+\theoremsymbol{\nolinebreak[1]\hspace*{.5em plus 1fill}\scalebox{0.8}{\ensuremath{\blacksquare}}}
\newtheorem{proof}{Beweis}
\newtheorem{beweis}{Beweis}
-\titleformat{\section}{\titlefont\Large}%
- {\S\,\thesection}{.66em}{}
\renewcommand{\thesection}{\arabic{section}}
+\renewcommand\sectionformat{\S \thesection\autodot \hspace{1ex}}
\endinput \ No newline at end of file